SlideShare a Scribd company logo
1 of 201
Download to read offline
Tutorial Section: Screen 10 of 10 National Board of Medical Examiners®
Tutorial
Time Remaining:
14 min 18 sec
This concludes the tutorial.
Click "Next" to begin your assessment.
L J
7
Previous Next Help Pause
Exam Section 1: Item 1 of 50 National Board of Medical Examiners^
Comprehensive Basic Science Self-Assessment
Time Remaining:
1 hr 14 min 58 sec
9 Mark
1. A 25-year-old man is brought to the emergency department because of severe abdominal pain, nausea, and vomiting for 1 hour. The pain originates in the left flank and radiates to his groin. His putse is 100/min, respirations are 18/min, and blood pressure is
150/100 mm Hg. Physical examination shows tenderness of the left flank and the left lower quadrant of the abdomen. Bowel sounds are mildly hypoactive. Test of the stoof for occult blood is negative. Which of the following best explains these findings?
. A ) Colon neoplasm
B) Diverticulitis
O C) Epididymitis
0 D) Renalinfarction
. E) Torsion of the testis
F) Ureteral calculus
9 0 0
Next Lab Values Calculator Review Help Pause
Exam Section 1: Item 2 of 50 National Board of Medical Examiners^
Comprehensive Basic Science Self-Assessment
Time Remaining:
1 hr 13 min 57 sec
9 Mark
2. Which of the following types of sensory information is compromised by lesions of the structure at site X in the photograph shown?
A ) Conscious proprioception
B) Pain sensation
0 C) Two-point discrimination
D ) Unconscious proprioception
E:
) Vibration sense
& * f*
Previous Next Lab Values Calculator Review Help Pause
Exam Section 1: Item 3 of 50 National Board of Medical Examiners^
Comprehensive Basic Science Self-Assessment
Time Remaining:
1 hr 13 min 51 sec
9 Mark
3. Moving the forearm against resistance from palm-down to palm-up (supination) position requires the use of which of the following muscles?
3 A ) Biceps brachii
B) Brachialis
O C) Triceps
D) Flexor carpi radialis
! J E) Pronator teres
9 0 0
Previous Next Lab Values Calculator Review Help Pause
Exam Section 1: Item 4 of 50 National Board of Medical Examiners^
Comprehensive Basic Science Self-Assessment
Time Remaining:
1 hr 13 min 42 sec
® Mark
4. A 35-year-old man is admitted to the hospital because of a 5-day history of fever and dyspnea He underwent a bone marrow transplantation G months ago: the procedure was complicated by severe graft-versus-host disease. His temperature is 38- C
(100.4T). and respirations are 30/min. Scattered crackles are heard on auscultation of the chest. A chest x-ray shows patchy infiltrates. A transbronchial biopsy specimen shows findings consistent with cytomegalovirus infection. Intravenous administration of
ganciclovir is begun. This drug interferes with the function of which of the following enzymes?
A ) DNApolymerase
3 B) Integrase
o C) Reverse transcriptase
3 0 ) RNA polymerase
J E } Thymidine kinase
* m O m
Previous Next Lab Values Calculator Review Help Pause
Exam Section 1: Item 5 of 50 National Board of Medical Examiners^
Comprehensive Basic Science Self-Assessment
Time Remaining:
1 hr 13 min 36 sec
9 Mark
o
200-
T>
» £
£ S 100-
<D
co
0
25 27 1
Cycle (day)
M - menstruation
5. The graph shows changes in serum estradiol concentration during a normal menstrual cycle. Which of the following ovarian cells is primarily responsible for the aromatizaiion of androstenedione to estradiol at the time indicated by the arrow?
A ) Granulosa
'
B) Luteaf
. ) G) Stromal
D ) Theca externa
E) Theca interna
O *
r
- o o o
Previous Next Lab Values Calculator Review Help Pause
Exam Section 1: Item 6 of 50 National Board of Medical Examiners^
Comprehensive Basic Science Self-Assessment
Time Remaining:
1 hr 13 min 32 sec
® Mark
6. A 14-year-old girl is brought to the physician by her parents because of a 1-month history of a rash that appears with sun exposure, Her parents tell the physician that she has been eating fiittle food. Physical examination shows a pruritic rash on the exposed
areas of the body. Her serum tryptophan concentration is decreased Urine studies show increased excretion of amino acids: predominantly alanine, isoleucine, leucine, phenylalanine: tryptophan, and valine. Production of which of the following vitamins is
most likely impaired in this patient?
A ) Niacin
0 B) Vitamin B ,(thiamine)
0 G) Vitamin B2 (riboflavin)
0 0 ) Vitamin B5 (pantothenic add)
E) Vitamin C
* m O m
Previous Next Lab Values Calculator Review Help Pause
Exam Section 1
: Item 7 of 50 National Board of Medical Examiners^
Comprehensive Basic Science Self-Assessment
Time Remaining:
1 hr 13 min 29 sec
® Mark
7. Aminoglycoside antibiotics are used for their synergistic action against bacteria, in combination with other agents. These antibiotics demonstrate in vitro synergy for several bacterial species when combined with which of the following classes of antibiotics?
3 A ) Fluoroquinolones
S) Macrolides
O C) Penicillins
D) Rifamycrns
! J E) Tetracyclines
* m O m
Previous Next Lab Values Calculator Review Help Pause
Exam Section 1: Item 8 of 50 National Board of Medical Examiners^
Comprehensive Basic Science Self-Assessment
Time Remaining:
1 hr 13 min 26 sec
® Mark
8. A 16-year-old boy with moderate intellectual disability is brought to the physician for a routine examination. There is a family history of mild and moderate intellectual disability in his mother and brother, respectively. Physical examination shows a long face;
prominent ears, and moderately enlarged testicles. Which of the following best describes the genetic mechanism of this patient's disorder?
. A ) Mutation in a mitochondrial gene
8) Presence of an extra sex chromosome
. J C) Translocation of a portion of an autosome
'
3 D) Trinucleotide repeat mutation on the X chromosome
. E } Trisomy of an autosome
O
* 9 0 0
Previous Next Lab Values Calculator Review Help Pause
Exam Section 1: Item 8 of 50 National Board of Medical Examiners^
Comprehensive Basic Science Self-Assessment MeaseWait
® Mark
9. A 75-year-old woman comes to the physician because of a 3-month history of an enlarging lesion on her face. Physical examination shows a 1, 5-cm, brown-black, mottled, scaly lesion with irregular borders. Microscopic examination of a biopsy specimen of the
lesion shows atypical melanocytes spread along the basilar layer of the epidermis. Which of the following is the most likely cause of these findings?
A ) Acanthosis nigricans
8) Actinic keratosis
C) Compound nevus
D) Lentigo maligna
E) Seborrheic keratosis
O * m O m
Previous Next Lab Values Calculator Review Help Pause
Exam Section 1: Item 10 ol 50 National Board of Medical Examiners^
Comprehensive Basic Science Self-Assessment
Time Remaining:
1 hr 13 min 15 sec
9 Mark
10. A physician wishes to determine the proportion of newborns delivered at a local hospital who had a diagnosis of congenital heart disease within the past year. Which of the following statistical measurements best describes these data?
Q A ) Attributable risk
S) Incidence
O C) Odds ratio
D) Prevalence
' J E) Relative risk
* m O m
Previous Next Lab Values Calculator Review Help Pause
Exam Section 1: Item 11 of 50 National Board of Medical Examiners^
Comprehensive Basic Science Self-Assessment
Time Remaining:
1 hr 13 min 11 sec
® Mark
11. A 25-year-old woman comes to the physician after her blood pressure was found to be 130/105 mm Hg at a health fair. She takes no medications. There is no family history of hypertension. Her last menstrual period was 1 week ago. Her blood pressure today
is 180/110 mm Hg. Bilateral abdominal bruits are heard. Treatment with an angiotensin-converting enzyme (ACE) inhibitor will most likely have which of the following acute effects on this patient's renal function?
. A ) Decreased concentrating ability secondary to renal angioedema
B) Decreased glomerular filtration rate secondary to dilation of efferent arterioles
. 5 C) Decreased renal blood flow secondary to dilation of afferent arterioles
'
! D) increased concentrating ability secondary to a change in permeability of the collecting duct
. E) Interstitial nephritis secondary to allergic drug reaction
* m O m
Previous Next Lab Values Calculator Review Help Pause
Exam Section 1: Item 12 ol 50 National Board of Medical Examiners^
Comprehensive Basic Science Self-Assessment
Time Remaining:
1 hr 12 min 57 sec
9 Mark
12 A 16-year-ofd boy is brought to the physician because of a 3-month history of shortness of breath while playing sports. He has no shortness of breath at rest. He says7
"Whenever I run around I cough, so i don't want to be on the basketball team anymore. " He
takes no medications and has no known allergies. There is a family history of hypertension and asthma. He is 165 cm (5 ft 5 in) tall and weighs 68 kg (150 lb): BMI is 25 kg/m2. His respirations are 12/min, and blood pressure is 115/75 mm Hg. Cardiac
examination shows no abnormalities except for a midsystolic click at the apex. The lungs are clear to auscultation of the chest. Which of the following best explains this patient's symptoms?
A ) Deconditioning
3 B) Exercise-induced asthma
o G) Malingering
3 0 ) Mitral valve prolapse
J E } Thyroid disease
O m O m
Previous Next Lab Values Calculator Review Help Pause
Exam Section 1: Item 13 of 50 National Board of Medical Examiners^
Comprehensive Basic Science Self-Assessment
Time Remaining:
1 hr 12 min 53 sec
9 Mark
13. A 25-year-ofd man comes to the physician because of a 3-day history of pain and swelling of his right leg. He has no history of major medical illness or recent trauma. Examination of the right lower extremity shows edema and tenderness. Duplex
ultrasonography of the right lower extremity shows a thrombus extending into the superficial femoral vein. Further studies show protein C deficiency. Inactivation of which of the foliowing coagulation factors is most likely as a result of this deficiency in this
patient?
A ) Factors V (proaccelebn) and VIII (antihemophilic factor)
3 B) Factors V (proaccelebn) and IX (plasma thromboplastin component)
O C) Factors V (proaccelebn) and XI (plasma thromboplastin antecedent)
3 D ) Factors VIII (antihemophilic factor) and IX (plasma thromboplastin component)
Q E) Factors VIII (antihemophilic factor) and XI (plasma thromboplastin antecedent)
( J F) Factors IX (plasma thromboplastin component) and XI (plasma thromboplastin antecedent)
* m O m
Previous Next Lab Values Calculator Review Help Pause
Exam Section 1: Item 14 ol 50 National Board of Medical Examiners^
Comprehensive Basic Science Self-Assessment
Time Remaining:
1 hr 12 min 50 sec
® Mark
14. Which of the following is most directly responsible for concentrating testosterone in the lumen of the seminiferous tubules?
3 A ) Androgen-binding protein
^ S) Follicle-stimulating hormone (FSH)
O G) FSH/gonadotropin-releasing hormone
o Q) Irthibm
' J E) Luteinizing hormone
O m O m
Previous Next Lab Values Calculator Review Help Pause
Exam Section 1: Item 15 ol 50 National Board of Medical Examiners^
Comprehensive Basic Science Self-Assessment
Time Remaining:
1 hr 12 min 47 sec
0Mar
*
15. A 23-year-old woman has had the lesions shown In her mouth for 3 days. She has had frequent similar episodes over the past 15 years. The lesions are exacerbated by spicy, salty: and acidic food and drinks. They last approximately 1 week and resolve
spontaneously. Visits to the dentist seem to trigger the development of the sores. Which of the following is the most likely diagnosis?
A ) Aphthous ulcers
3 B) Candidiasis
0 C) Geographic tongue
O 0 ) Koplik spots
^ E) Leukoplakia
F) Lichen planus
'
G) Psoriasis
o 9 0 0
Previous Next Lab Values Calculator Review Help Pause
Exam Section 1: Item 16 ol 50 National Board of Medical Examiners^
Comprehensive Basic Science Self-Assessment
Time Remaining:
1 hr 12 min 41 sec
® Mark
16. A 20-year-old woman comes to the emergency department 30 minutes after slippmg on ice and extending her hand to break her fall. Palpation of the anatomic snuff-box produces pain. A wrist x-ray is most likely to show a fracture of which of the following
carpal bones?
. A ) Scaphoid
8) Lunate
. ) C) Triquetrum
D) Pisiform
. . E) Trapezium
F) Trapezoid
. ) G ) Capitate
H) Hamate
O * m O m
Previous Next Lab Values Calculator Review Help Pause
Exam Section 1: Item 17 of 50 National Board of Medical Examiners^
Comprehensive Basic Science Self-Assessment
Time Remaining:
1 hr 12 min 38 sec
® Mark
17. A 7G-year-ofd man undergoes laparotomy for resection of an abdominal aortic aneurysm. During the procedure, an incidental finding of acquired colonic diverticula is made. The diverticula in this patient are most likely present in which of the following?
A ) Ascending coion
i S) Cecum
Q G) Descending colon
D) Sigmoid colon
! J E) Transverse colon
* m O m
Previous Next Lab Values Calculator Review Help Pause
Exam Section 1: Item 13 ol 50 National Board of Medical Examiners^
Comprehensive Basic Science Self-Assessment
Time Remaining:
1 hr 12 min 19 sec
9 Mark
18. During a period of 36 hours, an 80-year-old woman has increasingly severe abdominal pain followed by fever chills, tachycardia: hypotension and. finally, shock. Blood cultures grow Escherichia coti. Her condition worsens and. despite supportive therapy
and antibiotics; she dies 4 days after the onset of the illness. Which of the following is the most likely cause of the initial hypotension?
. A ) Excessive production of nitric oxide
B) Generation of hydrogen peroxide
o C) Hemorrhage
! D ) Induction of endothelial adhesion molecules
. E } Platelet aggregation
* m O m
Previous Next Lab Values Calculator Review Help Pause
Exam Section 1: Item 19 of 50 National Board of Medical Examiners^
Comprehensive Basic Science Self-Assessment
Time Remaining:
1 hr 12 min 16 sec
® Mark
19. A 65-year-old woman comes to the physician because of a 3-month history of headache, weakness of her arms, and left flank pain: she also has had a 14-kg (31-Eb) weight loss during this period. Physical examination shows weakness of the proximal upper
and lower extremity muscles. There is augmentation of strength with repetitive testing of the deltoid muscles. An MRI of the brain shows a single well-demarcated mass surrounded by edema in the right frontal lobe. A stereotactic biopsy specimen of the lesion
shows a malignant; smalt blue cell neoplasm that expresses cytokeratin, chromogranin, and synaptophysin. Which of the following is the most likely diagnosis?
A ) Anaplastic ependymoma
3 B) ExtranodaE primary central nervous system lymphoma
o C) Glioblastoma multiforme
3 0 ) Primary cerebral neuroblastoma
E } Pulmonary small cell carcinoma metastatic to the brain
® 0 0 0 0
Previous Next Lab Values Calculator Review Help Pause
Exam Section 1: Item 20 ol 50 National Board of Medical Examiners^
Comprehensive Basic Science Self-Assessment
Time Remaining:
1 hr 12 min 12 sec
® Mark
20. Failure of normal differentiation of the endoderm in the embryonic lung bud is most likely to affect the development of which of the following?
3 A ) Capillary patterns
3 S) Cartilage in bronchi
O Gj Smooth muscle on the bronchi
J D) Surfaetant secretion
3 E) Tracheal rings
o o o
Previous Next Lab Values Calculator Review Help Pause
Exam Section 1: Item 21 ol 50 National Board of Medical Examiners^
Comprehensive Basic Science Self-Assessment
Time Remaining:
1 hr 12 min 9 sec
® Mark
21. A 67-year-old woman comes to the physician for a health maintenance examination. Her brother and mother have a history of colon cancer. The physician recommends colonoscopy: but the patient says that she would prefer only for her stool to be tested for
biood. The physician explains that testing the stool for occult blood is not appropriate in this case. The physician is most likely concerned about which of the following regarding this test?
. A ) Low sensitivity
8) Low specificity
. J C) Potential for a false-positive result
3 D ) Uncertain negative predictive value
. E) Uncertain positive predictive value
* m O m
Previous Next Lab Values Calculator Review Help Pause
Exam Section 1: Item 22 ol 50 National Board of Medical Examiners^
Comprehensive Basic Science Self-Assessment
Time Remaining:
1 hr 12 min 5 sec
® Mark
22 A 5-year-old boy is brought to the emergency department after ingesting 10 oz of a household cleaning solvent He is treated for acute hepatic and renal failure for 1 week and then discharged. During the next montfr regeneration of this boy's mature
hepatocytes and renal tubular epithelial cells will be accomplished mostly by which of the following mechanisms?
. A ) Activation of stem cells to enter G1 phase of the cell cycle
8) Decreased apoptosis at GrM transition of the cell cycle
. J G) Recruitment of cells from G0 into the cell cycle
’
3 D) Shortened time for progression of cells through the cell cycle
. E) Terminal differentiation by celts exiting from the cell cycle
O 0 I* 0 0 0
Previous Next Lab Values Calculator Review Help Pause
Exam Section 1: Item 23 of 50 National Board of Medical Examiners^
Comprehensive Basic Science Self-Assessment
Time Remaining:
1 hr 12 min 4 sec
® Mark
23. A 17-year-old boy is brought to the physician by his mother because she is concerned that his puberty is delayed. The mother states. "He is so short. His father is 6 feet 5 inches talk I don't understand why he has not had his growth spurt." When the mother
leaves the room, the patient states:
'
I'm fine. I don't know what's the matter with her. She wants me to be tall like my dad." The patient is 175 cm (5 ft 9 in) tali and weighs 70 kg (155 lb): BMI is 23 kg/m2 Sexual development is Tanner stage 4. in addition to
reassuring the mother that her son is fine, which of the following is the most appropriate initial statement by the physician to the mother?
A ) "Since your son is fine with his height you should try to accept him as he is. "
3 B) "Tell me more about your concerns about your son's height."
o C) "Well do some blood tests just to be sure that all your son's hormone levels are okay."
3 O ) "Your son is average for his height and weight."
J E) "Your son is not going to be any taller."
* m O m
Previous Next Lab Values Calculator Review Help Pause
Exam Section 1: Item 24 ol 50 National Board of Medical Examiners^
Comprehensive Basic Science Self-Assessment
Time Remaining:
1 hr 12 min 2 sec
® Mark
24. A 34-year-old woman is admitted to the hospital for treatment of pulmonary tuberculosis. Infliximab therapy was initiated 6 months ago for severe Crohn disease. This pharmacotherapy most likely inhibited which of the following immunologic functions in this
patient?
. A ) Activation of nuclear factor K8 to induce expression of interleukin-10 (IL-10)
8) Direct toxicity to the causal organism
. J C) Maintenance of granulomas
'
3 D) Recruitment of segmented neutrophils to ingest and kill the bacteria
(_y E) Stimulation of B lymphocytes to produce neutralizing antibodies against the causal organism
* m O m
Previous Next Lab Values Calculator Review Help Pause
Exam Section 1: Item 25 ol 50 National Board of Medical Examiners^
Comprehensive Basic Science Self-Assessment
Time Remaining:
1 hr 11 min 53 sec
® Mark
25. An 8-year-old boy continues to bleed excessively after tooth extraction. Prothrombin time, bleeding time, and platelet count are within the reference range. Partial thromboplastin time is prolonged but corrects after addition to the assay chamber of plasma
from a patient with hemophilia A. Which of the following is the most likely diagnosis?
. A ) Acute disseminated intravascular coagulation
8) Pactor V (proaccelerin) deficiency
. J C) Factor VII (proconvertin) deficiency
D) Hemophilia A
. . E) Hemophilia B
F) Immune thrombocytopenic purpura
. J. G ) von Willebrand disease
O * m O m
Previous Next Lab Values Calculator Review Help Pause
Exam Section 1: Item 26 ol 50 National Board of Medical Examiners^
Comprehensive Basic Science Self-Assessment
Time Remaining:
1 hr 11 min 56 sec
® Mark
26. The diagram shows the major factors that determine blood pressure. Which of the following labeled factors is affected most by an aradrenergic antagonist?
^Centralnervous system
m®
Peripheral resistance
farteridfat)
Cardiac output x
Blood pressure
©
Stroke volume
^Heort rale
J
"Contractility j
^Venous return
Capacitance vessel tone
(v&nutar)
Blood volume
©
O A)
O 8)
O C)
O D)
OE)
m O m
Previous Next Lab Values Calculator Review Help Pause
Exam Section 1: Item 27 of 50 National Board of Medical Examiners^
Comprehensive Basic Science Self-Assessment
Time Remaining:
1 hr 11 min 54 sec
® Mark
r
f
..Jl
;
<
A /
1
/ 
^
|
f
5 iVtt VI
II
05
v i
uVL V2 VJ
R
V? v
*
r
*
m nVF
4
—
V~- I -T-
—I V
_ ^ H
-
II
^ .
4ir4 J
^
A
vs
27. A previously healthy 21-year-old woman comes to the office because of a 2-month,
history of shortness of breath and fatigue. Her most recent menstrual period was 3 months ago. Menses previously had occurred at regular 28-day intervals. She tells the
physician that she thinks she may be pregnant. She takes no medications and has not seen a physician for several years. She appears healthy. She is 160 cm (5 ft 3 in) tall and weighs 54 kg (120 ib); BMI is 21 kg/m2 Vital signs are within normal limits. The
lungs are dear. Cardiac examination shows a normal S(f
a widely split S2 that does not change with respiration, and a grade 3/6 holosystoMc murmur that is loudest at the lower left sternal border and radiates to the upper left sternal border. EGG is shown.
The most likely cause of these ffndings is dysfunction of which of the following structures?
A ) Atrial septum
3 B) Ductus arteriosus
Q C) Interventricular septum
D ) Pulmonic valve
E) Tricuspid valve
* m O m
Previous Next Lab Values Calculator Review Help Pause
Exam Section 1: Item 28 ol 50 National Board of Medical Examiners^
Comprehensive Basic Science Self-Assessment
Time Remaining:
1 hr 11 min 52 sec
® Mark
ft
*’v
-i - H
‘-v
: 8 ..
T 7-
-
-v /. y
^
:mrnm
r
*
aw
k
J %
J VJ
-
b
fi* * " . i
. J -i
» - WWHB
^dS?^
'
^d>
V
I
£ V-p
B
a
% -rt -kr
.
r - -
w.
v- .
i
*_
.
it
J
:
>S-
:
^
r’
v - ,.v
.
V ’ &
11
-J
"H H
_*
IS
n
T
* IT
* “ S
"V P
I
i
-J.
1 P
s i"3
1
r
ii
p
*
K i
i
a
i
l
.
i
f s t
•J~ r b
'
- b f a J
9 " P
J
r* In
J
n
X
« ]31
P
9
+
28. A 53-year-old man has had progressive difficulty swallowing for the past 3 months. He has a 10-year history of heartburn with esophageal regurgitation of gastric contents. Tissue obtained on biopsy of the lower third of the esophagus is shown. Which of the
following best describes the nature of this lesion?
A } Basal zone hyperplasia of submucosal glands
! J B) Intestinal metaplasia of squamous epithelium
O G) Malignant transformation of epithelium into squamous carcinoma
' 0 ) Squamous metaplasia of submucosal glands
«1 0 r r r»;
(IS
Lab Values Calculator Review
Previous Next Help Pause
Exam Section 1: Item 29 of 50 National Board of Medical Examiners^
Comprehensive Basic Science Self-Assessment
Time Remaining:
1 hr 11 min 45 sec
® Mark
29. A 50-year-old man comes to the emergency department because of a 2-week history of progressive shortness of breath. His pufse is 90/min, respirations are 26/min: and blood pressure is 120/S0 mm Hg. Physical examination shows no other abnormalities.
Laboratory studies show:
Arterial Pco2
Arterial Po2
Arterial 02content
Mixed venous Po2
Mixed venous 02 content
30 mm Hg
96 mm Hg
12 vol% (N=17%-21%)
36 mm Hg
8 vol% (N=10%-
1
:6%)
Which of the following is the most likely explanation for these findings?
A ) Anemia
B) Drug-induced alveolar hypoventilation
C) Residence at a high altitude
D ) Severe regional mismatching of alveolar ventilation and pulmonary capillary perfusion
E) Voluntary hyperventilation
* m O m
Previous Next Lab Values Calculator Review Help Pause
Exam Section 1: Item 30 ol 50 National Board of Medical Examiners^
Comprehensive Basic Science Self-Assessment
Time Remaining:
1 hr 11 min 46 sec
® Mark
30. An investigator is studying the effects of triiodothyronine (T
^
) and thyroxine (TJin hepatocytes in an experimental animal model. Which of the following best describes the action of these thyroid hormones on this target tissue?
3 A ) Both T3 and T 4 bind to the melanocortin 2 receptor on the cell surface
J B) Both T3 and T 4 enter the nucleus
( G) T3 is converted to T4 in the cytosol
J D) Thyroid hormone receptors preferentially bind T 4 over T3
O m O m
Previous Next Lab Values Calculator Review Help Pause
Exam Section 1: Item 31 ol 50 National Board of Medical Examiners^
Comprehensive Basic Science Self-Assessment
Time Remaining:
1 hr 11 min 42 sec
® Mark
31. A female newborn delivered at 38 weeks' gestation to a 28-year-old woman; gravida 1, para 1 develops respiratory distress. Pregnancy and delivery were uncomplicated'
amniotic fluid was clear, and the placenta was normal. Fetal ultrasonography and MRf
at 34 weeks’ gestation showed a congenital diaphragmatic hernia: with evidence of small bowel and stomach herniation into left hemithorax. She is 52 cm (20.4 in) long and weighs 3500 g (7 lb 11 oz). Her temperature is 37.5°C (99.ST), pulse is 138/min.
respirations are 50/min, and blood pressure is 70/55 mm Hg. Physical examination shows peripheral cyanosis that improves after administration of oxygen via endotracheal intubation. Breath sounds are decreased on the left. Cardiac examination shows
normal heart sounds without murmurs. Which of the following complications of the described pathology is most likely to be life threatening to this newborn?
A ) Active pneumonia
. B) Alveolar edema
0 C) Amniotic embolism
0 0 ) Inadequate surfactant synthesis
E) Pulmonary hypoplasia
* m O m
Previous Next Lab Values Calculator Review Help Pause
Exam Section 1: Item 32 ol 50 National Board of Medical Examiners^
Comprehensive Basic Science Self-Assessment
Time Remaining:
1 hr 11 min 35 sec
® Mark
32 An 18-hour-old male newborn Is 61 cm (24 in) long and weighs 5443 g (12 lb). His mother has type 1 diabetes mellitus. His serum glucose concentration is 20 rrig/dL Which of the following fetal conditions immediately prior to birth most likely precipitated the
newborn's postnatal hypoglycemia?
. A ) Decreased gluconeogenesis
8) Decreased glycogen concentration
. J C) Decreased glycogen synthetase activity
3 D ) Decreased serum insuEin concentration
. E } Increased serum insulin-like growth factor
O * m O m
Previous Next Lab Values Calculator Review Help Pause
Exam Section 1: Item 33 of 50 National Board of Medical Examiners^
Comprehensive Basic Science Self-Assessment
Time Remaining:
1 hr 11 min 31 sec
® Mark
33. The synthesis of the enzymes necessary for the replication of the genome occurs during which of the following phases of the cell cycle?
O A)
O B)
O C)
O D)
O E)
* m O m
Previous Next Lab Values Calculator Review Help Pause
Exam Section 1: Item 34 ol 50 National Board of Medical Examiners^
Comprehensive Basic Science Self-Assessment
Time Remaining:
1 hr 11 min 29 sec
® Mark
34. A 22-year-old woman is brought to the emergency department in a semicomatose condition after collapsing near the end of running a marathon. Her prerace weight was 47 kg (103 lb). She now weighs 50 kg (110 lb). Her pulse is 115/min, respirations are
15/min, and blood pressure is 90/50 mm Hg. Physical examination shows cool, dry skin. She is responsive to painful stimuli. Laboratory studies show:
Serum
Na+
116 mEq/L
4.8 mEq/L
89 mEq/L
22 mEq/L
22 mg/dL
101 mg/dL
1 mg/dL
K-
ci-
HCO3
-
Urea nitrogen
Glucose
Creatinine
This patient's condition is most likely due to which of the following?
o A ) Decreased ADH (vasopressin)
6 } Decreased aldosterone
Q C) Excessive fluid intake
! _ D) Inadequate fluid intake
O E) Increased aldosterone
* m O m
Previous Next Lab Values Calculator Review Help Pause
Exam Section 1: Item 35 ol 50 National Board of Medical Examiners^
Comprehensive Basic Science Self-Assessment
Time Remaining:
1 hr 11 min 26 sec
® Mark
35. A 36-year-old woman with type 2 diabetes mellitus comes to the physician for a follow-up examination. Current medications include a sulfonylurea. She is 173 cm (5 ft 3 in) tall and weighs 95 kg (210 lb): BMI is 32 kg/m2 Physical examination shows
acanthosis nigricans. Treatment with metformin is most likely to produce which of the following effects in this patient?
. A ) Decrease intestinal carbohydrate digestion
B) increase beta-cell insulin secretion
. J C) Increase deposition of adipocyte fat
'
3 D) increase hepatic triglyceride synthesis
. E } Inhibit hepatic gluconeogenesis
O * m O m
Previous Next Lab Values Calculator Review Help Pause
Exam Section 1: Item 36 ol 50 National Board of Medical Examiners^
Comprehensive Basic Science Self-Assessment
Time Remaining:
1 hr 11 min 23 sec
® Mark
36. A 23-year-old man in the emergency department has apnea and pinpoint pupils. Needle tracks are present on his arms. Activation of which of the following opioid receptors in the central nervous system is most likely to be responsible for the apnea?
O A) 5
O B) K
O C) u
O D) a
O * m O m
Previous Next Lab Values Calculator Review Help Pause
Exam Section 1: Item 37 of 50 National Board of Medical Examiners^
Comprehensive Basic Science Self-Assessment
Time Remaining:
1 hr 11 min 21 sec
® Mark
37. A 5-year-old girl with premature sexual development is diagnosed with precocious puberty. Pelvic examination shows a mass consistent with an ovarian tumor. Laboratory studies show decreased serum concentrations of gonadotropins and a marked
increase in circulating estrogens. The ovarian tumor is most likely derived from which of the following cell types?
. A ) Endothelial cells
8) Germinal epithelium
. ) G) Granulosa cells
0 D) Stromal fibroblasts
E) Thecal cells
* m O m
Previous Next Lab Values Calculator Review Help Pause
Exam Section 1: Item 33 ol 50 National Board of Medical Examiners^
Comprehensive Basic Science Self-Assessment
Time Remaining:
1 hr 11 min 19 sec
® Mark
38. A healthy 22-year-old woman undergoes testing to determine whether she is a suitable kidney donor for her 25-year-old brother with end-stage renal disease caused by type 1 diabetes meilitus. Immunologic studies show that she is human leukocyte antigen
(HLAJ-DR3/DR6 positive; and her brother is HLA-DR3/DR4 positive. To determine the extent of alioreactivity: a mixed lymphocyte reaction is done using irradiated stimulator cells isolated from the donor and responder cells isolated from the recipient. The T
lymphocytes that proliferate in these cultures will most likely react with which of the following HLA types?
A) DR3 only
O B) OR3 and DR4
o C) DR3 and DR6
o 0) 0R4 only
E) DR4 and DR6
O F) 0R6 only
O * I* m O m
Previous Next Lab Values Calculator Review Help Pause
Exam Section 1: Item 39 of 50 National Board of Medical Examiners^
Comprehensive Basic Science Self-Assessment
Time Remaining:
1 hr 11 min 17 sec
9 Mark
39. A 67-year-old man with poorty controlled unstable angina is about to undergo coronary angiography with stent placement Prior to the procedure, treatment is initiated with aspirin and a drug that inhibits platelet interaction with fibrinogen. This drug is most
likely which of the following?
. A ) Abciximab
8) Celecoxib
0 C) Cilostazol
_ D) Cloptdogrel
. E } Dipyridamole
* m O m
Previous Next Lab Values Calculator Review Help Pause
Exam Section 1: Item 40 ol 50 National Board of Medical Examiners^
Comprehensive Basic Science Self-Assessment
Time Remaining:
1 hr 11 min 15 sec
® Mark
40. Lesch-Nyhan syndrome, an X-linked recessive disease; is seen in approximately 1/100.000 males. Which of the following is the expected prevalence of heterozygous females?
O A) 1/1000
c B) 1/10,000
o C) 1/50,000
'
0) 1/200,000
o E) 1/10,000,000
o m O m
Previous Next Lab Values Calculator Review Help Pause
Exam Section 1: Item 41 ol 50 National Board of Medical Examiners^
Comprehensive Basic Science Self-Assessment
Time Remaining:
1 hr 11 min 12 sec
® Mark
41. The specimen shown is from a 65-year-old man. Which of the following is the most likely diagnosis?
O A ) Acute leukemia
B) Colonic carcinoma
0 C) Hepatic cell carcinoma
D ) Leiomyosarcoma
E) Liposarcoma
o o o
Previous Next Lab Values Calculator Review Help Pause
Exam Section 1: Item 42 ol 50 National Board of Medical Examiners^
Comprehensive Basic Science Self-Assessment
Time Remaining:
1 hr 11 min 10 sec
® Mark
42 A 45-year-old man who is able to bicycle 4 5 minutes a day has switched to a rowing machine. After 5 minutes on the machine, he experiences vertigo, lightheadedness. and fatigue of the left upper extremity. Within a few minutes of stopping the rowing
exercise, all symptoms resolve.. Which of the following findings is most likely on physical examination?
O A ) Diastolic murmur at the cardiac apex
8) increased jugular venous pressure
. J C) Pansystolic murmur at the cardiac apex
D) Right carotid bruit
. E) Supraclavicular bruit
O * m O m
Previous Next Lab Values Calculator Review Help Pause
Exam Section 1: Item 43 of 50 National Board of Medical Examiners^
Comprehensive Basic Science Self-Assessment
Time Remaining:
1 hr 11 min 8 sec
9 Mark
43. A 55-year-old man comes to the physician because of a 2-month history of increasing difficulty swallowing and regurgitation of undigested food. He also has noticed unusual rumbling sounds in his voice that he feels originate in his neck. Physical examination
shows halitosis. A videofluoroscopic swallowing study shows a 4-cm, posterior midline pouch protruding between the thyropharyngeus and cricopharyngeus portions of the inferior pharyngeal constrictor muscle. These muscles are most likely innervated by
which of the following nerves?
A ) Glossopharyngeal nerve
0 B) Hypoglossal nerve
0 G) Motor fibers from the vagus nerve
0 0 ) Parasympathetic fibers from the vagus nerve
O & ) Sympathetic fibers from the superior cervical ganglion
* m O m
Previous Next Lab Values Calculator Review Help Pause
Exam Section 1: Item 44 ol 50 National Board of Medical Examiners^
Comprehensive Basic Science Self-Assessment
Time Remaining:
1 hr 11 min 6 sec
9 Mark
44. A 5-year-old girl is brought to the physician because of list;essness: fatigue, and dulf pain in the right upper quadrant of the abdomen. Her height and weight are below the 25th percentile. Laboratory findings indicate that the content of her p-globin chain is
15% to 20% of normal. Sequencing of the (3-globin gene shows a point mutation in a sequence 3' to the coding region in which AATAAA is converted to AACAAA. Consequently, the amount of mRNA for (3-globin is decreased to 10% of normal. Which of the
following functions in rriRMA synthesis and processing is most fikefy encoded by the sequence AATAAA?
A ) Capping with GTP
3 B) Cleavage and pofyadenylation
o C) Silencing of the promoter
3 0 ) Splicing of the initial mRNA transcript in the nucleus
J E } Transport of the mRNA out of the nucleus
* m O m
Previous Next Lab Values Calculator Review Help Pause
Exam Section 1: Item 45 ol 50 National Board of Medical Examiners^
Comprehensive Basic Science Self-Assessment
Time Remaining:
1 hr 11 min 3 sec
9 Mark
45. A 7G-year-old man comes to the physician for a follow-up examination. He has hypertension treated with a p-adrenergic antagonist He lives on a farm in central California and says he has always distilled his own liquor Before retiring f 0 years ago, he
worked in a hat factory and subsequently in a textile factory. He has smoked 2 packs of cigarettes daily for the past 55 years. He tells the physician that he has had several episodes of painful swelling of his right great toe. Physical examination shows several
lesions consistent with gouty tophi over the elbows bilaterally. Laboratory studies show:
Hemoglobin A
Serum
Glucose
Creatinine
Uric acid
5.6%
1c
93 mg/dL
3.2 mg/dL
7.9 mg/dL
The most likely cause of this patient's condition is which ofthe following?
A ) Cigarette smoking
^ B) Drinking home-distilled liquor
C) Farming in central California
Q D ) Working in a hat factory
E } Working in a textile factory
o * 9 0 0
Previous Next Lab Values Calculator Review Help Pause
Exam Section 1: Item 46 ol 50 National Board of Medical Examiners^
Comprehensive Basic Science Self-Assessment
Time Remaining:
1 hr 11 min 1 sec
9 Mark
46. A 22-year-old woman comes to the physician because of a 6-month history of difficulty swallowing. She says that she feels like she is choking on both solids and liquids. She has no pain with
swallowing. She has had a 4.5-kg f10-lb) weight loss during this time. There is no history of fever or chills. She is not sexualfy active. She does not smoke cigarettes or use illicit drugs. She is 170 cm
(5 1
!7 in) tall and weighs 59 kg (130 lb); EM is 20 kg/m2 Her vital signs are within normal limits. Physical examination shows no abnormalities. An x-ray of the esophagus is shown. Which of the
following is the most likely explanation for this patient's symptoms?
O A ) Acid reflux into the lower esophagus
Q B } Atrophy of the smooth muscle in the esophagus
o C ) Inflammatory degeneration of esophageal waif neurons
D ) Longitudinal mucosal tear at the esophagogastric junction
E) Perforation of the esophageal wall
«r
* m O m
Previous Next Lab Values Calculator Review Help Pause
Exam Section 1: Item 47 ol 50 National Board of Medical Examiners^
Comprehensive Basic Science Self-Assessment
Time Remaining:
1 hr 10 min 59 sec
® Mark
47. A 4S-year-old woman comes to the office because of a 4-month history of headaches, itchy skim difficulty swallowing, heartburn, chest tightness, pain in her arms and legs; and a burning sensation with urination. She has a history of simitar symptoms since
the age of 14 years but previous examinations showed no abnormalities. Her vital signs are within normal limits. Physical examination and laboratory studies today show no abnormalities. Which of the following is the most likely diagnosis?
A ) Conversion disorder
8) Factitious disorder
. J C) Illness anxiety disorder (hypochondriasis)
D) Malingering
. E } Somatic symptom disorder
O * m O m
Previous Next Lab Values Calculator Review Help Pause
Exam Section 1: Item 43 ol 50 National Board of Medical Examiners^
Comprehensive Basic Science Self-Assessment
Time Remaining:
1 hr 10 min 56 sec
® Mark
48. Based on the graph of ju-aminohippurate (PAM) concentration versus PAH secretion, which of the following is lower at point Y than at point X?
A ) Glomerular filtration rate
B) PAH clearance
Q C) PAH excretion rate
'
D ) PAH filtered bad
E) Renal blood flow
Y
80-
“O
at
c
£
X
Q
OJ &
<n
l 40 —
x
<
0 i
T T
0 20 40 60 80 100
Plasma p-aminohippurate
(PAH) concentration
mg/dL
o o o
Previous Next Lab Values Calculator Review Help Pause
Exam Section 1: Item 49 of 50 National Board of Medical Examiners^
Comprehensive Basic Science Self-Assessment
Time Remaining:
1 hr 10 min 52 sec
® Mark
49. A 41-year-old woman is evaluated because of increasingly severe headaches for 6 weeks. Her blood pressure is 160/100 mm Hg while standing and supine. A bruit is heard over the left costovertebral angle. Urinalysis shows no abnormalities. An angiogram
of the left renal artery shows alternating areas of stenosis and aneurysmal dilatation ("string of beads" sign). Which of the following conditions of the renal artery is the most fikely diagnosis?
. A ) Fibromuscular dysplasia
8) Hyaline arteriolosclerosis
. J C) Intimal fibroplasia
3 0 ) Periarterial fibroplasia
. E } Perimedial hyperplasia
O
* 9 0 0
Previous Next Lab Values Calculator Review Help Pause
Exam Section 1: Item 50 ol 50 National Board of Medical Examiners^
Comprehensive Basic Science Self-Assessment
Time Remaining:
1 hr 10 min 51 sec
® Mark
50. A cohort study assessing risk factors for acquisition of infection with a newly identified agent is performed. Only newly diagnosed subjects are eligible, and controls are selected on the basis of age. The results of this study are shown:
Infection Present Infection Absent
Turtle exposure
No turtle exposure
60 20
40 80
Which of the following is the relative risk, for the exposure variable?
O A) 0-7
O B) 1.0
O C) 1.7
O P) 2.2
O E) 3.1
* m O m
Previous Next Lab Values Calculator Review Help Pause
Exam Section 2: Item 1 of 50 National Board of Medical Examiners^
Comprehensive Basic Science Self-Assessment
Time Remaining:
1 hr 14 min 55 sec
9 Mark
1. A 30-year-old man with HIV infection has been treated with a combination of antiretroviral drugs, including zidovudine (AZT); for 3 years. Laboratory studies show a marked increase in his plasma HIV viral load during the past 3 months. Viral resistance to
zidovudine is suspected. A mutation in which of the following is most likely to explain the resistance to zidovudine in this patient?
A ) Integrase
8) Neuraminidase
. C) Protease
! D ) RNA-dependent DNA polymerase
. E } Thymidine kinase
9 0 0
Next Lab Values Calculator Review Help Pause
Exam Section 2: Item 2 of 50 National Board of Medical Examiners^
Comprehensive Basic Science Self-Assessment
Time Remaining:
1 hr 14 min 51 sec
9 Mark
2. An investigator is studying [
^adrenoreceptors in female experimental animals. During the experiment, epinephrine is injected intramuscularly into each animal, and the effects on p^
-adrenoreceptors are then observed. Which of the following physiologic effects
is most likely to be observed in these animals?
. A ) Increased myocardial contractility
8) internal urethral sphincter contraction
O C) Lipolysis
3 D) Pilomotor contraction
. . E) Pupillary dilation
F) Uterine relaxation
* m O m
Previous Next Lab Values Calculator Review Help Pause
Exam Section 2: Item 3 of 50 National Board of Medical Examiners^
Comprehensive Basic Science Self-Assessment
Time Remaining:
1 hr 14 min 43 sec
9 Mark
3. A 16-year-old boy is brought to the emergency department because of a 2-day history of increasingly severe abdominal pain. His temperature is 39X (182:2X1pulse is 86,'min, respirations are 18/min, and blood pressure is 120/60 mm Hg. Abdominal
examination shows exquisite tenderness of the right lower quadrant. His leukocyte count is 16: 000/mma. An appendectomy is done; the appendix is swollen with a tan exudate on the serosal surface. Which of the fofiowing best characterizes the leukocytosis in
this patient?
A ) Basophilia
0 B) Eosinophiiia
0 G) Lymphocytosis
0 0 ) Monocytosis
J E } Neutrophilia
* m O m
Previous Next Lab Values Calculator Review Help Pause
Exam Section 2: Item 4 of 50 National Board of Medical Examiners^
Comprehensive Basic Science Self-Assessment
Time Remaining:
1 hr 14 min 46 sec
® Mark
4. A 32-year-old primigravid woman delivers a heafthy 3402-g (7-lb 8-oz) male newborn after an uncomplicated cesarean delivery because of a nonreassuring fetal stress test. Two days prior to discharge from the hospital she has persistent numbness of the
area surrounding the abdominal incision. The physician assures the patient that sensation will gradually return as the nerves regenerate. Which of the following best describes the rate-limiting step in this patient's return to normal sensation?
. A ) Dorsal root ganglion cel! proliferation
8) Fast anterograde axonal transport
0 C) Fibroblast proliferation
0 D ) Retrograde axonal transport
. E) Slow anterograde axonal transport
* m O m
Previous Next Lab Values Calculator Review Help Pause
Exam Section 2: Item 5 of 50 National Board of Medical Examiners^
Comprehensive Basic Science Self-Assessment
Time Remaining:
1 hr 14 min 44 sec
® Mark
5. An 18-year-old woman comes to the physician because of progressive fever general malaise, and blood in her urine since she began oral antibiotic therapy for a urinary tract infection 5 days ago. She also has a 3-day history of a rash.. Her temperature is
38°C (1DG. 43F), pulse is 75/min, respirations are 12/mrn, and blood pressure is 125/80 mm Hg. Physical examination shows a petechial rash over the chest. back: and upper and lower extremities. Urinalysis shows:
Blood
Protein
Leukocytes
Eosinophils
3+
1+
150/hpf
30%
Which of the following is the most likely diagnosis?
o A ) Acute tubular necrosis
6 } Glomerulonephritis
Q C) IgA nephropathy
D) Interstitial nephritis
O E) Papillary necrosis
* m O m
Previous Next Lab Values Calculator Review Help Pause
Exam Section 2: Item 6 of 50 National Board of Medical Examiners^
Comprehensive Basic Science Self-Assessment
Time Remaining:
1 hr 14 min 33 sec
9 Mark
6. A 33-year-old woman has had weakness of the right lower two thirds of the face for the past 2 months. Which of the following labeled regions in the normal brain shown is the most likely site of the lesion causing this symptom?
Central sulcus
E F
C
/
G
B  r
r
£$
I
> J
(h : kM
A-
5
r cd- -i
I
.--
Ll
m
-wr
r m
- —
.
-J 3«
» r.
j
i
H .
^ .
ip
O A)
O B)
O C)
O D)
OE)
OF)
O G)
O H)
O n
O J)
0 0 # 0 0
Previous Next Lab Values Calculator Review Help Pause
Exam Section 2: Item 7 of 50 National Board of Medical Examiners^
Comprehensive Basic Science Self-Assessment
Time Remaining:
1 hr 14 min 36 sec
® Mark
7. A 4G-year-old woman comes to the physician because of temperatures of 38 to 39X (100.4 to 102.2aF) and malaise for the past 2 days. She has chronic alcohol dependence. There are several spider angiomas on the face, chest and back; she is not
jaundiced. The abdomen is protuberant and nontender. There is shifting dullness to percussion. A firm liver edge is ballotable 3 cm below the right costal margin. Routine laboratory studies show miid anemia; mildly increased hepatic transaminase activities:
and a decreased serum albumin concentration. Peritoneal aspiration yields serous fluid with 1100 leukocytes/mm3 {80 neutrophils) and 100 erythrocytes/mm3 Which of the following processes best accounts for the patient's febrife ilEness?
A ) Acute cholecystitis
3 B) Chronic pancreatitis
o C) Exacerbation of autoimmune hepatitis
3 0 ) Pelvic infEammatory disease
E } Spontaneous bacterial peritonitis
O
* 9 0 0
Previous Next Lab Values Calculator Review Help Pause
Exam Section 2: Item 8 of 50 National Board of Medical Examiners^
Comprehensive Basic Science Self-Assessment
Time Remaining:
1 hr 14 min 34 sec
9 Mark
8. A patient with a 6-month history of heat intolerance. fatigue; episodes of tachycardia: and weight loss has a diffusely enlarged thyroid gland Serum concentrations of triiodothyronine (T3) and thyroxine (TJ are increased; thyroid-stimulating hormone is
decreased. Which of the following is the most likely diagnosis?
O A ) Autoimmune thyroid hyperplasia
8) Pituitary neoplasm
O G) Surreptitious ingestion of T4
D) Thyroid neoplasm
O * m O m
Previous Next Lab Values Calculator Review Help Pause
Exam Section 2: Item 9 of 50 National Board of Medical Examiners^
Comprehensive Basic Science Self-Assessment
Time Remaining:
1 hr 14 min 31 sec
9 Mark
9. A 19-year-old man who is a college student is brought to the emergency department because of the sudden onset of right-sided chest pain and difficulty breathing after an accident in which he was thrown from his bicycle. He has difficulty walking and cannot
climb stairs because of pain and shortness of breath. He is slightly cyanotic, afebrile: and tachypneic. Which of the following is most suggestive that fractured ribs caused the respiratory problem?
A ) Bronchophony
B) Expiratory stridor
0 C) Inspiratory stridor
0 D) Subcutaneous crepitus
. E) Succussion splash
* m O m
Previous Next Lab Values Calculator Review Help Pause
Exam Section 2: Item 10 of 50 National Board of Medical Examiners^
Comprehensive Basic Science Self-Assessment
Time Remaining:
1 hr 14 min 29 sec
® Mark
10. A 62-year-ofd woman develops difficulty breathing. Pulmonary function tests before and after branchediiator therapy show no changes. Predicted and patient values are:
Predicted
Test Patient
FVC (L)
FEV1 (L)
FEV/FVC
Total Eung capacity (L)
Residual volume (L)
5.0 4.0
4.0 2.4
0.8 0.6
6.0 7.2
1.6 2.7
Which of the following is the most likely explanation for these findings?
Airway Resistance Lung Compliance
O A)
O B)
O C)
O 0)
OE)
T T
normal
T
Normal 1
I T
4
* m O m
Previous Next Lab Values Calculator Review Help Pause
Exam Section 2: Item 11 of 50 National Board of Medical Examiners^
Comprehensive Basic Science Self-Assessment
Time Remaining:
1 hr 14 min 26 sec
® Mark
11. A 3-year-old girl is brought to the physician because of a 2-week history of diarrhea. Her temperature is 37.6CC (99.8aF}: pulse is 70/min, respirations are Wmjn; and blood pressure is 110/70 mm Hg. Physical examination shows generalized
[ymphadenopathy. A CT scan of the chest and abdomen shows enlarged lymph nodes in the mesentery and para-aortic region. Examination of a lymph node biopsy specimen shows marked proliferation of histiocytes and numerous segmented neutrophils.
Granulomata are absent and special stains show numerous acid-fast bacilli. which are subsequently identified as Mycobacterium avium-intracellulare. Serum studies show normal concentrations of IgA, lgG; IgM, B lymphocytes, T lymphocytes, and CD4? and
CD8+- T lymphocytes. This patient most likely has defective function or expression of which of the following proteins?
A ) Class I MHG molecules
. 8) Interferon-gamma receptor
0 G) interleukin-2 (IL-2) receptor
) 0) Leukocyte function-associated antigen-1
'
E) MADPH oxidase
* m O m
Previous Next Lab Values Calculator Review Help Pause
Exam Section 2: Item 12 of 50 National Board of Medical Examiners^
Comprehensive Basic Science Self-Assessment
Time Remaining:
1 hr 14 min 24 sec
® Mark
12 Drug X is used to treat pain associated with rheumatoid arthritis. The drug is a weak acid, with a pKa of 4.4; it is absorbed principally through the stomach. It is determined that Drug X is absorbed efficiently by the body because of the ionization conditions
under which it exists at gastric and blood pH. Which of the following sets of physical chemical states of the drug is most fikely?
At Gastric pH
Ionized
Ionized
Nonionized
At Blood pH
ionized
O A)
O B)
O C)
O D)
nonionized
ioniized
Nonionized nonionized
* m O m
Previous Next Lab Values Calculator Review Help Pause
Exam Section 2: Item 13 of 50 National Board of Medical Examiners^
Comprehensive Basic Science Self-Assessment
Time Remaining:
1 hr 14 min 21 sec
® Mark
13. A previously healthy 10-year-old girl is brought to the physician by her mother because of a 6-week history of headache, nausea, and difficulty walking. An MR] of the brain shows a mass iin the posterior fossa that is found to be an astrocytoma, i his tumor
developed from cells that normally serve which of the following functions in the brain?
. A ) Formation of myelin sheaths in the central nervous system
8) Phagocytosis of recycled synaptic terminal membrane
. C) Production and secretion of cerebrospinal fluid
Q D) Termination of action potentials
. E) Transport of hormones from the cerebral spinal fluid to capillaries
F) Uptake of amino acid neuretransmitters
* m O m
Previous Next Lab Values Calculator Review Help Pause
Exam Section 2: Item 14 of 50 National Board of Medical Examiners^
Comprehensive Basic Science Self-Assessment
Time Remaining:
1 hr 14 min 19 sec
9 Mark
14. A 26-year-old woman comes to the physician because of a 1-week history of rectal pain that is made more severe by defecation, and occasional blood on the toilet tissue after a bowel movement. She says that her
stools appear normal and that she has not had any trauma. She has a history of chronic constipation. Visual rectal examination shows the findings in the photograph. Which of the following is the most likely diagnosis?
A ) Anal fissure
3 B) Bowen carcinoma
C) Condyloma acuminatum
O D) Perianal abscess
E) Prolapsed internai hemorrhoid
O * & * C
Previous Next Lab Values Calculator Review Help Pause
Exam Section 2: Item 15 of 50 National Board of Medical Examiners^
Comprehensive Basic Science Self-Assessment
Time Remaining:
1 hr 14 min 16 sec
9 Mark
15. A moderately obese 55-year-old man is brought to the emergency department because of a 10-hour history of severe chest pain. He has a 5-year history of exercise-induced angina. His pulse is 109/min, respirations are 15/min, and blood pressure is
132/92 mm Hg. Physical examination shows diaphoresis. A blood sample obtained 2 hours after admission shows increased serum activity of creatine kinase ME. Which of the following is the most likely cause of this aboratory finding?
. A ) Increased GoEgi complex activity
8) increased permeability of the plasma membrane
O G) Mitochondria!swelling
D) Muclear lysis
. E) Proliferation of the endoplasmic reticulum
O * 9 0 0
Previous Next Lab Values Calculator Review Help Pause
Exam Section 2: Item 16 of 50 National Board of Medical Examiners^
Comprehensive Basic Science Self-Assessment
Time Remaining:
1 hr 14 min 12 sec
9 Mark
16. AGG-year-ofd woman comes to the physician because of a 6-month history of pain in her hips and knees. Physical examination shows findings consistent with osteoarthritis; and the physician recommends iibuprofen. The patient refuses and asks about taking
glucosamine. Which of the following responses by the physician is most appropriate?
. A ) "Glucosamine hasn't been studied well enough for me to recommend it."
8) "Glucosamine's side effects aren't listed. It may be more dangerous than we realize."
. G) "Ibuprofen has been proven effective for your condition."
'
! D) "What have you heard about using glucosamine to treat arthritis?"
. E) "Why did you come to me if you don’t want to take what I recommend?11
F) "You should really see a naturopathic doctor."
* m O m
Previous Next Lab Values Calculator Review Help Pause
Exam Section 2: Item 17 of 50 National Board of Medical Examiners^
Comprehensive Basic Science Self-Assessment
Time Remaining:
1 hr 14 min 10 sec
9 Mark
17. An 88-year-old man who lives alone is brought to the physician by his daughter because she is concerned that he has not been eating a well-balanced diet for 9 months. He is 170 cm (5 ft 7 in) tall and weighs 50 kg (110 fb); BMI is 17 kg/m2 Physical
examination shows multiple ecchymoses on the upper and lower extremities. Laboratory studies show:
Platelet count
Prothrombin time
Serum
Vitamin B6 (pyridoxine)
Vitamin C (ascorbic acid)
Folic acid
160,000/mm3
12 sec (INR=1}
9 ng/dL (N=5—30)
0.1 mg/dL (N=0.4-2)
5 ng/dL (N=2—20)
The ecchymoses in this patient are most likely due to a disorder of which of the following?
A ) Arachidonic add production
Q B) Binding of carboxyglutamic acid to phospholipid
C C) Carboxylation of factor fl (prothrombin)
J D ) Proline hydroxylafion
E } Transfer of methyl groups to organic acids
* m O m
Previous Next Lab Values Calculator Review Help Pause
Exam Section 2: Item 13 of 50 National Board of Medical Examiners^
Comprehensive Basic Science Self-Assessment
Time Remaining:
1 hr 14 min 3 sec
® Mark
18. A 14-year-ofd boy has persistent leukocytosis and neutrophilia without evidence of a current infection. He has a history of recurrent infections of the skin: upper and lower airways, and perirectal area. Gram-negative and gram-positive rods have been isolated.
The number and function of B and T lymphocytes are normal. Production of hypochlorous acid by neutrophils and the nitroblue tetrazolium reduction test are normal;. Neutrophil chemotactic response to the formyl-MetLeuPhe (fMLP) peptide is diminished.
Which of the following disorders of neutrophils is the most likely diagnosis?
A ) Chronic granulomatous disease
3 B) Cyclic neutropenia
o C) Leukocyte adhesion deficiency
3 0 ) Myeloperoxidase deficiency
E } Neutrophil-specific granule deficiency
O * m O m
Previous Next Lab Values Calculator Review Help Pause
Exam Section 2: Item 19 of 50 National Board of Medical Examiners^
Comprehensive Basic Science Self-Assessment
Time Remaining:
1 hr 14 min 5 sec
® Mark
0-r0
1
x
2
I
> 4
^0
2
^
3 6
1 1
n 2 £
3 3
3
1 2
1 1 1 1
o
4
HI 2 2
3 3
5 2
6 3
1 2 3
1 1 1
IV 2 5
3 6
2
3
2 3
1 4
Affected male
# Affected female
Unaffected male
O Unaffected female
0 Affected male, deceased
0 Unaffected female, deceased
19. An investigator is studying a large family with many members who are affected by a disorder caused by a fully penetrant autosomal dominant inherited gene mutation. A pedigree is shown. Most affected members also have a rare allele at a focus thought to
be closely linked to the disease focus. A father (individual 111-3) and his daughter (individual iV-3) have the disorder, but they have the wild-type allele at the linked locus. Which of the following is the most likely cause of these findings?
( J A ) Insertion of a LINE sequence
8 } Random segregation
Q G) Recombination
D) Single nucleotide polymorphism
E) Transduction
® 0 0 0 0
Previous Next Lab Values Calculator Review Help Pause
Exam Section 2: Item 20 ol 50 National Board of Medical Examiners^
Comprehensive Basic Science Self-Assessment
Time Remaining:
1 hr 14 min 3 sec
® Mark
20. A 45-year-old woman comes to the physician because of a 6-month history of shortness of breath with exeition and a nonproductive cough. She sometimes has difficulty swallowing and often has heartburn, especially if she fies down after a meal. She adds
that her fingers have swollen, and she has had to get her wedding ring resized. Her fingers also become white and painful in cold weather or cotd water. Her pulse is 75/min: respirations are 20/min, and blood pressure is 150/100 mm Hg. Physical
examination shows tight, smooth facial skin without wrinkles. Additional testing is most likely to show which of the following sets of cardiovascular changes in this patient?
Left Ventricular
Diastolic Compliance
Mean Pulmonary
Artery Pressure
Coronary Vascular
Resistance
O A)
O B)
O C)
O D)
O E)
O F)
O G)
O H)
f f f
T T I
T *
V
4 4
t t
v
'
l i
A.
1
4 V V
© 15
fE
Lab Values Calculator Review
Previous Next Help Pause
Exam Section 2: Item 21 ol 50 National Board of Medical Examiners^
Comprehensive Basic Science Self-Assessment
Time Remaining:
1 hr 13 min 45 sec
9 Mark
21. A 25-year-old woman comes to the physician for a routine health maintenance examination. She is currently preparing for an 8-km (5-mile) race, and she sprint trains twice weekly. During this training: she runs 200-meter sprints in two groups of 10 sprints
separated by a 30-second rest between each sprint in a group: and a 2-minute rest between each group of 10. After the workout her legs feel weak, and her muscles burn and sometimes cramp. Which of the following best explains her symptoms?
A ) Decreased activity of the sodium-proton antiporter, resulting in an acidic sarcoplasm
8) increased activity of the sodium-proton antiporter resulting in an acidic sarcoplasm
O C) increased oxygen delivery to the muscle: leading to increased metabolism and acid production
'
D) Regeneration of NAD+ from NADH; which produces acid
* m O m
Previous Next Lab Values Calculator Review Help Pause
Exam Section 2: Item 22 ol 50 National Board of Medical Examiners^
Comprehensive Basic Science Self-Assessment
Time Remaining:
1 hr 13 min 42 sec
® Mark
22 A 14-year-old girl is brought to the physician because of a 1-month history of migraine-like headaches, vomifing: and multiple left-sided focal seizures. She has had hearing foss since the age of 11 years. Her mother and maternal grandmother have high-tone
deafness. Physical examination shows Joss of vision in one half of the visual field of the right eye and weakness of the right upper and tower extremities. Serum and cerebrospinal fluid concentrations of lactic acid are increased. This patient most likely has a
mutation of which of the fallowing?
O A ) Endoplasmic reticulum glycosyItransferase
3 B) Lysosomal a-gfucosidase
o C) Mitochondrial tRNALeu
3 0 ) Nuclear proteasome activator
J E } Peroxisomal catalase
O m O m
Previous Next Lab Values Calculator Review Help Pause
Exam Section 2: Item 23 of 50 National Board of Medical Examiners^
Comprehensive Basic Science Self-Assessment
Time Remaining:
1 hr 13 min 40 sec
9 Mark
23. A case-control study is conducted to determine if obesity is a risk factor for gastroesophageal reflux disease (GERD). A questionnaire is used to select subjects with severe symptoms of GERD and subjects with no symptoms. A BMf is calculated for each
subject. The results (in kg/m2) are shown:
BMI<25 25<BMK30 30<BMk35 BMI>35
Subjects with GERD symptoms
Subjects with no symptoms
303 900 200 50
700 900 200 30
Which of the following represents the odds ratio for GERD symptoms in subjects with BMfs greater than 35 compared with subjects with BMIs less than 25?
A) (50
*30) / (300
*700)
o B) (50
*700) I (30x300)
o cj [50/(30+50)31[300/(300+700)]
: D) [50/(50+300)31[30/(30+700)3
.
^ E) [50/(50+200+900+300)3 '[30/(30+200+900+700)3
* m O m
Previous Next Lab Values Calculator Review Help Pause
Exam Section 2: Item 24 ol 50 National Board of Medical Examiners^
Comprehensive Basic Science Self-Assessment
Time Remaining:
1 hr 13 min 33 sec
0Mar
*
24. A 14-year-old boy is brought to the physician by his mother because of daily headaches for 2 months. The headaches are described as a bilateral aching in the tempfes. His mother states that he also Tias not been himself' for the past few months. He seems
more confused, often forgetting names, dates, and places: and he is clumsy with frequent falls. His school performance also has declined over the past quarter. Physical examination shows a broad-based; ataxic gait. He is alert and oriented to person: place,
and time: but he is slow to answer questions. Chronic abuse of which of the following substances is the most likely cause of this patient's condition?
A ) Cocaine
C B) Ethanol
0 G) Inhaled gfue
0 0 ) Methamphetamines
0 E } PCP (phencyclidine)
* m O m
Previous Next Lab Values Calculator Review Help Pause
Exam Section 2: Item 25 ol 50 National Board of Medical Examiners^
Comprehensive Basic Science Self-Assessment
Time Remaining:
1 hr 13 min 36 sec
9 Mark
25. A study is designed to measure the impact of exercise on the incidence of myocardial infarction. Subjects are enrolled in the study and divided into two groups based on their self-reported exercise habits. At the end of the study; subjects who reported
exercising have half the incidence of myocardial infarction compared with the subjects who did not exercise. Which of the following best describes this study design?
A ) Case-control
8) Case series
. C) Cohort
o D) Cross-sectional
. E } Randomized clinical trial
* m O m
Previous Next Lab Values Calculator Review Help Pause
Exam Section 2: Item 26 ol 50 National Board of Medical Examiners^
Comprehensive Basic Science Self-Assessment
Time Remaining:
1 hr 13 min 33 sec
9 Mark
26. An 18-year-old woman is being evaluated for amenorrhea. She has never had a menstrual period. She is 183 cm (6 ft) tall. Breast development and external genitalia are normal. There is no axillary or pubic hair. Which of the following karyotypes is most
likely?
O A) 45.X
O 0) 46,xx
O C) 46,XY
O D) 46.X,i(Xq)
O E) 47,XXX
* m O m
Previous Next Lab Values Calculator Review Help Pause
Exam Section 2: Item 27 of 50 National Board of Medical Examiners^
Comprehensive Basic Science Self-Assessment
Time Remaining:
1 hr 13 min 31 sec
® Mark
27. A 50-year-old man comes to the physician because of a 2-month history of pain of his wrists; changes in skin color, and progressive fatigue. His brother has type 2 diabetes mefliitus and cirrhosis. Physical examination shows bronze-colored skin, tenderness
of the metacarpophalangeal joints in both hands, and hepatosplenomegaly. Serum studies show:
AST
AIT
Ferritin
100 U/L
110 U/L
1200 ng/mL
Total iron-binding capacity 200 pg/dL (N=25G-400)
Transferrin saturation 80% (N=20—50}
Analysis of a liver biopsy specimen shows a markedly increased iron concentration and cirrhosis. Which of the following is the most likely cause of the fmdings in this patient?
A ) Increased erythropoietin action
Q B) Increased intestinal iron absorption
Q C) Increased oral iron intake
! D ) Decreased erythropoiesis
) E) Decreased iron excretion
F) Decreased serum transferrin concentration
m O m
Previous Next Lab Values Calculator Review Help Pause
Exam Section 2: Item 28 ol 50 National Board of Medical Examiners^
Comprehensive Basic Science Self-Assessment
Time Remaining:
1 hr 13 min 29 sec
® Mark
28. The presence of argininosuccinate in the urine indicates a defect in the conversion of which of the following?
0 A ) Ammonia to urea
J S) Lysine to glutaryl CoA
0 G) Methionine to succinyl CoA
J D) Phenylalanine to fumarate
! J E) Tryptophan to indole
o o o
Previous Next Lab Values Calculator Review Help Pause
Exam Section 2: Item 29 of 50 National Board of Medical Examiners^
Comprehensive Basic Science Self-Assessment
Time Remaining:
1 hr 13 min 26 sec
® Mark
29. A 55-year-old woman is brought to the emergency department after being injured in a motor vehicle collision. She has an injury of the soft tissue of the face that prevents her from drinking from a glass without
spilling the contents. Several days later she stilt has this problem. The photograph shows her attempting to purse her lips to whistle. Which of the following nerves is most likely damaged?
A ) Buccal branch of the facial nerve
B) Inferior alveolar branch of the mandibular division of the trigeminal nerve
O Infraorbital branch of the maxillary division of the trigeminal nerve
D ) Mandibular branch of the facial nerve
E) Pharyngeal branches of the vagus nerve
* m O m
Previous Next Lab Values Calculator Review Help Pause
Exam Section 2: Item 30 ol 50 National Board of Medical Examiners^
Comprehensive Basic Science Self-Assessment
Time Remaining:
1 hr 13 min 24 sec
® Mark
30. A previously healthy 6-month-ofd boy is brought to the physician because of a cough for 1 week. Initially he had a low-grade fever sneezing, congestion, and runny nose. He then developed a dry intermittent cough. The parents now note that with any startle
the baby chokes and gasps. He has not had any immunizations. Physical examination shows paroxysms of '‘machine gun" -like coughing with a forced expiratory grunt at the end of coughing. Leukocyte count is 30; 000/mm3 (70% lymphocytes). Neutrophil
chemotaxis and oxidative metabolism are defective due to increased activity of which of the following enzymes?
A ) Adenyly! cyclase
3 B) Myeloperoxidase
o C) NADPH oxidase
O 0 ) Phospholipase C
J E } Protein kinase C
* m O m
Previous Next Lab Values Calculator Review Help Pause
Exam Section 2: Item 31 of 50 National Board of Medical Examiners^
Comprehensive Basic Science Self-Assessment
Time Remaining:
1 hr 13 min 22 sec
® Mark
31. A 22-year-old woman comes to the office because of a 3-day history of nonproductive cough. She also has a 1-week history of fatigue: progressive shortness of breath with exertion and white lying down and swelling of her legs and feet. She delivered a male
newborn via uncomplicated vaginal delivery 1 month ago. She has no history of major medical illness and takes no medications. Her temperature is 37.7aC (99.8aF): pulse is 104/min, respirations are 20/miinP and blood pressure is 126/80 mm Hg. Bilateral
basilar crackles are heard. There is 1+ edema of the lower extremities bilaterally. Which of the following is the most likely diagnosis?
A ) Amniotic fluid embolism
3 B) Cardiomyopathy
o C) Major depressive disorder
3 0 ) Pneumonia
J E } Pulmonary embolism
' F) Pulmonary fibrosis
o * 9 0 0
Previous Next Lab Values Calculator Review Help Pause
Exam Section 2: Item 32 ol 50 National Board of Medical Examiners^
Comprehensive Basic Science Self-Assessment
Time Remaining:
1 hr 13 min 20 sec
® Mark
32 A 63-year-old man is scheduled to undergo coronary artery bypass grafting with a portion of the great saphenous vein. An incision to remove a portion of the vein should begin in which of the following locations?
3 A ) Along the lateral surface of the leg
J S) Along the medial side of the ankle joint
(3 G) Along the plantar surface of the foot
0 } Anterior to the knee joint
' J E) Posterior to the hip joint
* m O m
Previous Next Lab Values Calculator Review Help Pause
Exam Section 2: Item 33 of 50 National Board of Medical Examiners^
Comprehensive Basic Science Self-Assessment
Time Remaining:
1 hr 13 min 18 sec
9 Mark
33. A 62-year-old man is brought to the emergency department 2 hours after the sudden onset of pain and coolness of his right leg. He is otherwise healthy except for mild hyperthyroidism treated with propylthiouracil. Examination of the tower extremities shows
normal skim naiis: and hair growth patterns. Pulses are absent in the right lower extremity and normal on the left. Which of the following is the most likely diagnosis?
A ) Cellulitis
8) Deep venous thrombosis
O C) Embolic arterial occlusion
3 D ) Lumbar cisc herniation
. E } Rhabdomyolysis
* m O m
Previous Next Lab Values Calculator Review Help Pause
Exam Section 2: Item 34 ol 50 National Board of Medical Examiners^
Comprehensive Basic Science Self-Assessment
Time Remaining:
1 hr 13 min 16 sec
9 Mark
34. A 32-year-old woman comes to the physician for a follow-up examination after atypical cells were noted on a recent Pap smear. Physical examination shows a 1 x 1-cm area of leukoplakia on the cervix A biopsy specimen of the lesion shows invasive
squamous cell carcinoma. Malignant cells from this site will most likely drain first to which of the following lymph nodes in this patient?
. A ) Femoral
8) inferior mesenteric
O C) Internal iliac
D) Lumbar
. E) Superficial inguinal
* m O m
Previous Next Lab Values Calculator Review Help Pause
Exam Section 2: Item 35 ol 50 National Board of Medical Examiners^
Comprehensive Basic Science Self-Assessment
Time Remaining:
1 hr 13 min 14 sec
9 Mark
35. A 70-year-old man dies in a motor vehicle collision. He had been undergoing evaluation for occult blood in the stool A photograph of a section of the transverse colon obtained at autopsy is shown. Which of the following is
the most likely diagnosis?
.. A ) Hyperplastic polyp
Q B) Inflammatory pseudopolyp
C) Juvenile polyp
O D ) Peutz-Jeghers syndrome
E) Tubular adenoma
& * f*
Previous Next Lab Values Calculator Review Help Pause
Exam Section 2: Item 36 ol 50 National Board of Medical Examiners^
Comprehensive Basic Science Self-Assessment
Time Remaining:
1 hr 13 min 12 sec
9 Mark
36. A 61-year-old man has erectile dysfunction due to spinal cord injury at L-2. Sildenafil is likely to markedly correct the dysfunction by acting at which of the following labeled structures in the transverse section of the penis?
O A)
O B)
O C)
O Pi
0 0 © 0 0
ns
Lab Values Calculator Review
Previous Next Help Pause
Exam Section 2: Item 37 of 50 National Board of Medical Examiners^
Comprehensive Basic Science Self-Assessment
Time Remaining:
1 hr 13 min 10 sec
9 Mark
40-i
37. Which of the following is the mean number of episodes of urinary tract infections for children (n=100} in the sample shown in the graph?
O A ) 1
c B) 1.55
o C) 2.07
D ) Cannot be determined from this graph
g 30- I
J iBUBn
o
8 2
°-
_Q
I 10- «
3
0 1 2
Number of urinary tract
infections
m O m
Previous Next Lab Values Calculator Review Help Pause
Exam Section 2: Item 33 ol 50 National Board of Medical Examiners^
Comprehensive Basic Science Self-Assessment
Time Remaining:
1 hr 13 min 3 sec
® Mark
38. A 3-year-old boy is brought to the physician because of a 1-month history of pale skin. His parents are of European descent. He has no personal or family history of major medical illness. Physical
examination shows pallor. Laboratory studies show:
Hemoglobin
Hematocrit
Mean corpuscular hemoglobin concentration
Mean corpuscular volume
3 g.'dL (N=11-15)
24% (N=28-45)
34% Hb/cell (IM=31-36)
90 pm3 (N=77-98)
A photomicrograph of a peripheral blood smear is shown. Genetic testing is most likely to show which of the following findings in this patient?
A ) Heterozygous mutation in the ankyrin gene
B) Heterozygous mutation in the a-globin gene
O -
P) Heterozygous mutation in the p-giobin gene
D ) Homozygous mutation in the ankyrin gene
O E) Homozygous mutation in the a-globin gene
F) Homozygous mutation in the P-globin gene
* m O m
Previous Next Lab Values Calculator Review Help Pause
Exam Section 2: Item 39 of 50 National Board of Medical Examiners^
Comprehensive Basic Science Self-Assessment
Time Remaining:
1 hr 13 min 5 sec
® Mark
39. A 24-year-old woman is brought to the emergency department 15 minutes after the sudden onset of shortness of breath following a bee sting. Her pulse is 130/min, and blood pressure is 70/30 mm Hg. Three hours later, her blood pressure returns to normal
following administration of intravenous fluids, corticosteroids. antihistamines: and epinephrine. The next day. she has minimal urine output. Which of the following areas of the kidney is likely to be most affected with this patient's prolonged hypotension?
O A ) Glomerular epithelial cells
8) Loop of Henle
0 G) Medullary interstifium
D) Mesangial cells
. E) Proximal tubules
* m O m
Previous Next Lab Values Calculator Review Help Pause
Exam Section 2: Item 40 ol 50 National Board of Medical Examiners^
Comprehensive Basic Science Self-Assessment
Time Remaining:
1 hr 13 min 3 sec
® Mark
40. A 65-year-old man develops urinary incontinence immediately after an operation for prostate cancer. The most likely cause of his condition is damage to the prostatic nerve plexus that resulted in denervation of the internal urethrai sphincter. The function of
which of the following types of tissue is most likely impaired as a result of this damage?
. A ) Dense irregular connective tissue
8) Dense regular connective tissue
. C) Skeletal muscle
D) Smooth muscle
. E } Transitional epithelium
* m O m
Previous Next Lab Values Calculator Review Help Pause
Exam Section 2: Item 41 ol 50 National Board of Medical Examiners^
Comprehensive Basic Science Self-Assessment
Time Remaining:
1 hr 13 min 0 sec
® Mark
41. A 10-year-old boy who has begun chemotherapy for acute myelogenous leukemia awakens at night with fever and severe pain in the ankles. Treatment with over-the-counter analgesics does not resolve the pain. The next morning, he has pain with urination
and blood in the urine. An increased serum concentration of which of the following compounds is the most likely cause of this patient's symptoms?
A ) Cystine
8) Glycine
. ; C) Magnesium
V D) Urea
. . E) Uric acid
* m O m
Previous Next Lab Values Calculator Review Help Pause
Exam Section 2: Item 42 ol 50 National Board of Medical Examiners^
Comprehensive Basic Science Self-Assessment
Time Remaining:
1 hr 12 min 53 sec
9 Mark
42 A 27-year-old woman is brought to the physician by her family because she has been progressively lethargic and unwilling to leave her apartment over the past week. She has been receiving treatment in a mental health center for 10 years but missed her last
appointment 13 days ago because of a snowstorm. She is now reluctant to return because she believes the staff is involved in an extraterrestrial plot. Three years ago; she had similar symptoms treated with electroconvulsive therapy. She appears
disheveled. She is having auditory hallucinations of several people talking about her. Physical examination shows normal findings. She has poor eye contact, a flat affect and slow speech. She describes an elaborate delusional system about the plot at the
mental health center. Thought content is otherwise impoverished. Which of the following is the most likely diagnosis?
A ) Bipolar disorder depressed
. B) Borderline personality disorder
0 C) Delusional disorder
) D ) Schizophrenia
E) Schizotypal personality disorder
* m O m
Previous Next Lab Values Calculator Review Help Pause
Exam Section 2: Item 43 of 50 National Board of Medical Examiners^
Comprehensive Basic Science Self-Assessment
Time Remaining:
1 hr 12 min 56 sec
® Mark
43. A viral gene product is found to decrease expression of class i MHC molecules on the surfaces of infected cells. A mutant strain of virus is isolated that has a nonfunctional form of this gene. Which of the following types of cells are likely to contribute more
effectively to control of the parental strain of the virus than to control of the mutant virus?
0 A ) CD4+- 7 lymphocytes
8) CD8+- T lymphocytes
. ) C) Follicular dendritic cells
'
_ D) Natural killer cells
. . E } Plasma cells
* m O m
Previous Next Lab Values Calculator Review Help Pause
Exam Section 2: Item 44 ol 50 National Board of Medical Examiners^
Comprehensive Basic Science Self-Assessment
Time Remaining:
1 hr 12 min 54 sec
® Mark
44. A previously healthy 40-year-old man is brought to the emergency department 1 hour after the sudden onset of severe pain in his left leg while playing tennis. He is found to have ruptured the left Achilles tendon and undergoes operative repair and long leg
cast immobilization. Six months later, the left calf shows a 2-cm decrease in circumference compared with the right calf. Which of the following is the most likely cause of this decrease?
. A ) Decreased glycogen synthesis
8) Decreased myosin light chain phosphatase activity
. J C) Increased phosphatidyl degradation
'
j D) increased protein degradation
. E) Mitochondria damage
F) Necrosis of muscle fibers
* m O m
Previous Next Lab Values Calculator Review Help Pause
Exam Section 2: Item 45 ol 50 National Board of Medical Examiners^
Comprehensive Basic Science Self-Assessment
Time Remaining:
1 hr 12 min 50 sec
® Mark
45. A 5G-year-old woman has recently diagnosed carcinoma of the breast. An x-ray of the chest shows a tumor next to the right side of the heart. An enhanced CT scan with the tumor invading the
pericardium is shown. Which of the following structures is most likely involved?
A) Coronary sinus
B) Greater splanchnic vein
C) Right phrenic nerve
D ) Right vagus nerve
E) Thoracic duct
O * 9 0 0
Previous Next Lab Values Calculator Review Help Pause
Exam Section 2: Item 46 ol 50 National Board of Medical Examiners^
Comprehensive Basic Science Self-Assessment
Time Remaining:
1 hr 12 min 4£ sec
® Mark
46. A 25-year-old woman with a history of rheumatic fever and mitral valve dysfunction comes to the physician because of a 2-week history of fever and fatigue. She underwent a root canal procedure 1 month ago;
before which she had taken a single dose of amoxicillin. Her temperature is 38.4*0 (101 29F). A grade 4/6 blowing murmur is heard on auscultation under the left axilla. A photomicrograph of a Gram stain of the
organism recovered from a blood culture specimen is shown. On blood agar plates; the organism shows alpha hemolysis. Which of the following is the most likely causal organism?
A ) Enterococcus faecalis
B) Group A beta-hemolytic streptococci
0 C) Staphylococcus aureus
D) Streptococcus mitts
E) Streptococcus pneumoniae
O * m O m
Previous Next Lab Values Calculator Review Help Pause
Exam Section 2: Item 47 ol 50 National Board of Medical Examiners^
Comprehensive Basic Science Self-Assessment
Time Remaining:
1 hr 12 min 46 sec
® Mark
47. A 24-year-old woman comes to the physician for advice about contraceptive methods. She is recently married and is not interested in having children until her mid 30s. Which of the following contraceptives carries the highest risk for interference with fertility in
this patient Eater in life?
. A ) Cervical cap with spermicidal jelly
8) Condoms and spermicidal foam
o C) JUD
o D) Oral contraceptive
. . E) Progestin implant
O m O m
Previous Next Lab Values Calculator Review Help Pause
Exam Section 2: Item 43 ol 50 National Board of Medical Examiners^
Comprehensive Basic Science Self-Assessment
Time Remaining:
1 hr 12 min 44 sec
® Mark
48. Topical corticosteroid creams and phototherapy are often effective in the treatment of psoriasis. This effectiveness suggests a role for metabolism or nuclear binding of which of the following vitamins in the treatment of psoriatic lesions?
A) Niacin
^ S) Vitamin B1 (thiamine)
0 G) Vitamin B2 (riboflavin)
J 0 } Vitamin B6 (pyridoxine)
0 E) Vitamin C
.
'
F) Vitamin D
0 G) Vitamin E
H } Vitamin K
* m O m
Previous Next Lab Values Calculator Review Help Pause
Exam Section 2: Item 49 of 50 National Board of Medical Examiners^
Comprehensive Basic Science Self-Assessment
Time Remaining:
1 hr 12 min 42 sec
® Mark
49. A 21-year-old woman of Japanese descent comes to the emergency department because of a 3-hour history of facial flushing. Her symptoms began shortly after she drank a glass of champagne for the first time at a wedding reception. Physical examination
shows profound erythema over the face. The most likely cause of these findings is a mutation in the gene for which of the following enzymes?
. A ) Acetyl-CoAreductase
8) Alcohol catalase
. ) C) Alcohol dehydrogenase
'
_ D) Alcohol reductase
. E) Aldehyde dehydrogenase
O * 9 0 0
Previous Next Lab Values Calculator Review Help Pause
Exam Section 2: Item 50 ol 50 National Board of Medical Examiners^
Comprehensive Basic Science Self-Assessment
Time Remaining:
1 hr 12 min 39 sec
® Mark
50. A 35-year-old woman is brought to the emergency department after she sustains a fracture of the neck of the fibula of her right leg. She was struck by a car while crossing the street. Which of the following findings is most likely on examination of the affected
leg?
Pain Over
Proximal Fibula
Pain Over
Distal Fibula
absent
present
absent
Dorsiflexion Plantar Flexion Achilles Reflex
O A)
O B)
O C)
O D)
O E)
Absent 4/5 1/5 1+
Absent 0/S 4/5 1+
Present 1/5 4/5 2+
Present absent
present
5/5 0/5 absent
Present 4/5 4/5 2+
* m O m
Previous Next Lab Values Calculator Review Help Pause
Exam Section 3: Item 1 of 50 National Board of Medical Examiners^
Comprehensive Basic Science Self-Assessment
Time Remaining:
1 hr 14 min 57 sec
9 Mark
1. AG5-year-old man comes to the physician because of fever and a worsening cough productive of approximatefy % cup of blood-tinged sputum daily. He has smoked VA packs of cigarettes daily for 45 years. His temperature is 38.4X (101 2aF); respirations
are 20/min,and blood pressure is 140/90 mm Hg. Physical examination shows bilateral clubbing of the digits. Pulmonary examination shows egophony. whispered pedoriloquy, and dullness to percussion in the area overlying the 2nd and 3rd ribs on the right
anteriorly just inferior to the right clavicle. Which of the following structures is the most likely site of an obstructing carcinoma?
A ) Main carina
0 B) Right lower lobe bronchus
0 G) Right main bronchus
0 0 ) Right middle lobe bronchus
J E } Right upper lobe bronchus
m O m
Next Lab Values Calculator Review Help Pause
Exam Section 3: Item 2 of 50 National Board of Medical Examiners^
Comprehensive Basic Science Self-Assessment
Time Remaining:
1 hr 14 min 54 sec
9 Mark
2. A 43-year-old woman comes to the physician because of a 4-week history of intermittent episodes of severe abdominal pain: especially after eating fatty foods. Physical examination shows no abnormalities. Abdominal ultrasonography shows several
3- to 4-mm gallstones contained within the gallbladder. The patient declines surgery. A drug with which of the following mechanisms of action is most appropriate for this patient?
. A ) Decreased gallbladder muscle contractility
8) increased gastrin production
. J C) increased metabolism of bite salts
o D) inhibition of biliary cholesterol secretion
. E } Inhibition of cytochrome P450 activity
O m O m
Previous Next Lab Values Calculator Review Help Pause
Exam Section 3: Item 3 of 50 National Board of Medical Examiners^
Comprehensive Basic Science Self-Assessment
Time Remaining:
1 hr 14 min 52 sec
® Mark
3. A 24-year-old woman comes to the physician for an examination prior to employment. Physical examination shows no abnormalities. An ECG shows a sinus rhythm of 70/min; PR interval of 152 msec and QRS complex of 84 msec: there are no ectopic beats.
Which of the following best represents the longest conduction time in this patient?
A ) Anterior left bundle branch
8) intra-atrial
G) Low right atrium to bundle of His
D) Proximal bundle of His to ventricular myocardium
E) Right bundle branch
O * m O m
Previous Next Lab Values Calculator Review Help Pause
Exam Section 3: Item 4 of 50 National Board of Medical Examiners®
Comprehensive Basic Science Self-Assessment
Time Remaining:
1 hr 14 min 46 sec
® Mark
4. An 18-year-old woman with mild intellectual disability is brought to the physician because of a 3-day history of decreased ability to see in reduced light. She has a lifelong history of chronic diarrhea. Two years ago; she developed a lack of muscle control of her
arms and legs: and generalized weakness. Her 16-year-old brother has had similar symptoms. Ophthalmologic examination shows bilateral retinitis pigmentosa. There is ataxia and loss of deep tendon reflexes. Laboratory studies show erythrocytes with spiny
projections and a serum total cholesterol concentration of 40 mg/dL. Which of the following apolipoproteirs is most likely deficient in this patient?
O A ) ApoA-l
O B) Apo A-l
o C} Apo B
O D } Apo C
O E) Apo E
o o & r #
Previous Next Lab Values Calculator Review Help Pause
Exam Section 3: Item 5 of 50 National Board of Medical Examiners^
Comprehensive Basic Science Self-Assessment
Time Remaining:
1 hr 14 min 41 sec
® Mark
5. An investigator is studying obesity in an adolescent population. Five thousand normal-weight patients are followed from the age of 10 years to the age of 15 years. At the conclusion of the study, 1100 patients meet the criteria for obesity. Which of the following
best represents the incidence per 1000 patient-years of obesity during the course of this study?
O A) 10
O 8) 25
O C) 44
o D) 50
O E) 64
O * m O m
Previous Next Lab Values Calculator Review Help Pause
Exam Section 3: Item 6 of 50 National Board of Medical Examiners^
Comprehensive Basic Science Self-Assessment
Time Remaining:
1 hr 14 min 33 sec
9 Mark
t "
*y?vrG>H v

* .>v- - . -.
-v
^ asf i
i
u
A d a£F £
6. A 2S-year-old woman at 32 weeks' gestation comes to the physician because of a 4-day history of fever and back pain. She says:hat during this time she afso has been crying frequentfy. Her temperature is 38X
(100.4°F). Physical examination shows costophrenic angle tenderness. The photomicrograph shown represents her disease. Which of the following mechanisms iis the most likely cause?
Q A ) Chorioamnionitis
3 B) Endometritis
Q G) Glomerulonephritis
O D) Hematogenous infection
Q E } Obstructive uropathy
F) Pelvic infEammatory disease
* _ c*
4 A
*
i
i Hi
^
* _
S
'L
r i
~
- Sr
I
^ |
I
- i L
.* * J
T' a
 i
F
V I
*
»i
: *1 *
*
- •j
'*' t*
i ** •
' f *> * %
*
* ¥
V - - k* li
* !r
;&3r
V 41
. “
P u
n -i
n ill r *-
j
rrn* •
• 'S-
. L * ..
.
4
ii
1
,
I ' >
t ' -r
*
. i
*
r /
P L U
I
-t
*»*
.
0%
^ -
l/v »
I
*
. :
!
M
*
A- * h
,vF rf
-I
1 *1
*
+
l*
§
wf
I
--
1
K
-;
JI
'V
-.’ -F
„ r'rt
H W
*
s ,
% »
**
V l!
twit
|V
,1*
n J
w
:
4
m
f
=.
«1
!
© ©
Previous Next Lab Values Calculator Review Help Pause
Exam Section 3: Item 7 of 50 National Board of Medical Examiners^
Comprehensive Basic Science Self-Assessment
Time Remaining:
1 hr 14 min 35 sec
® Mark
7. A 54-year-old man with a 1-year history of multiple sclerosis has increasingly painful muscle spasms. An agonist that acts at which of the following receptors is most likely to alleviate the increased extensor tone and clonus?
3 A ) y-Aminobutyricacid
^ S) Histamine-1 (Ht)
o G) Histamine-2 (H
^
i D) Muscarinic-1 (M,)
3 E) Muscarinic-2
3 F) Nicotinic
E
o m O m
Previous Next Lab Values Calculator Review Help Pause
Exam Section 3: Item 8 of 50 National Board of Medical Examiners^
Comprehensive Basic Science Self-Assessment
Time Remaining:
1 hr 14 min 32 sec
® Mark
8. A 27-year-old primigravid woman at 36 weeks' gestation comes to the physician for a prenatal visit. Physical examination shows a uterus consistent in size with a 33-week gestation, so the patient is referred for ultrasonography, which shows normal fetal
measurements. The ultrasonography showrn is an image of the male fetal scrotum; the testes are indicated by the arrows. Which of the following is the most likely underlying cause of this finding?
A } High attachment of the tunica albuginea
. 8) Lack of scrotal attachment of the guberrtaculum
'
G) Obstruction of the posterior urethra
J 0 ) Patent processus vaginalis
E } Twisting of the spermatic cord
* m O m
Previous Next Lab Values Calculator Review Help Pause
Exam Section 3: Item 9 of 50 National Board of Medical Examiners^
Comprehensive Basic Science Self-Assessment
Time Remaining:
1 hr 14 min 28 sec
® Mark
9. A 98-year-old woman dies of respiratory failure. At autopsy: gross examination shows a small heart with a brown discoloration. Histologic examination of the cardiac tissue by light microscopy shows fight brown pigment within the cardiac myocytes, particularly
in the perinuclear region. This material does not stain positively for iron. Which of the following best describes this pigment?
. A ) Anthracotic pigment
8) Free fatty acids
. C) Hemosiderin
D) Laminin
. . E) Lipofuscin
O * m O m
Previous Next Lab Values Calculator Review Help Pause
Exam Section 3: Item 10 ol 50 National Board of Medical Examiners^
Comprehensive Basic Science Self-Assessment
Time Remaining:
1 hr 14 min 24 sec
® Mark
10. A 43-year-old woman has had increasing fatigue, intolerance to cold: and dry thickened skin for 7 weeks. The thyroid gland is firm, nodular and diffusely enlarged. Examination of tissue obtained on biopsy of the thyroid gland shows a diffuse infiltration of
lymphocytes with occasional lymphoid follicles. Which of the following is the most likely diagnosis?
. A ) Chronic autoimmune (Hashimoto) thyroiditis
8) Diffuse toxic goiter (Graves disease)
0 C) Granulomatous thyroiditis
0 D) Papillary carcinoma
. E) Toxic multinodular goiter
F) Viral thyroiditis
* m O m
Previous Next Lab Values Calculator Review Help Pause
Exam Section 3: Item 11 of 50 National Board of Medical Examiners^
Comprehensive Basic Science Self-Assessment
Time Remaining:
1 hr 14 min 20 sec
® Mark
11. A 3-year-old girl has a history of recurrent infections, fn vitro, neutrophils isolated from this patient are capable of phagocytosis and can kill Lactobacillus species but not Staphylococcus aureus. This patient most likely has a defect involving which of the
following enzymes?
. A ) Catalase
8) Elastase
. C) Myeloperoxidase
o D) NADPH oxidase
. E } Superoxide dismutase
O m O m
Previous Next Lab Values Calculator Review Help Pause
Exam Section 3: Item 12 ol 50 National Board of Medical Examiners^
Comprehensive Basic Science Self-Assessment
Time Remaining:
1 hr 14 min 17 sec
9 Mark
12 A 7-year-old girl is brought to the physician because of a 2-week history of painful swelling under her right ami which has become increasingly severe during the past 3 days. Physical examination shows a 2 x 1-cm, tender right axillary lymph node and a
small papule on the dorsum of the right hand. A silver-stained biopsy specimen of the papule shows pleomorphic bacilli. Which of the following is the most likely causal organism?
O A } Bartonella henselae
8) Brucella melitensis
o C) Burkholder!a mallei
0 D) Franciselta tuiarensis
. E) Streptobacitlus moniliformis
* m O m
Previous Next Lab Values Calculator Review Help Pause
Exam Section 3: Item 13 of 50 National Board of Medical Examiners^
Comprehensive Basic Science Self-Assessment
Time Remaining:
1 hr 14 min 14 sec
9 Mark
13. A newborn born at 3u weeks' gestation dies 3 days later The photograph shown is of a section of brain as seen at autopsy. Which of the following is the most likely underlying disease?
A ) Biliary atresia
B) Cyanotic congenital heart disease
O ®) Hemolytic disease of the newborn
D ) Neonatal meningitis
O E) Respiratory distress syndrome
o o o
Previous Next Lab Values Calculator Review Help Pause
Exam Section 3: Item 14 of 50 National Board of Medical Examiners^
Comprehensive Basic Science Self-Assessment
Time Remaining:
1 hr 14 min 10 sec
9 Mark
14. A 50-year-ofd woman is brought to the emergency department 20 minutes after the sudden onset of pain in her epigastrium that radiates to her back and is associated with nausea. She has a history of cholelithiasis. She appears restless. Her temperature is
37.6X (99.7aF): pulse is 120/min, and btood pressure is 115/60 mm Hg. Abdominal examination shows distention with rebound tenderness and guarding. The pain is partially relieved when the patient bends forward. ACT scan of the abdomen shows fluid
surrounding the pancreas. Obstruction of which of the following is the most likely cause of the findings in this patient?
A ) Ampulla of Vater
0 B) Cisterna chyli
0 G) Common hepatic duct
0 0 ) Cystic duct
J E } Superior mesenteric artery
* m O m
Previous Next Lab Values Calculator Review Help Pause
Exam Section 3: Item 15 ol 50 National Board of Medical Examiners^
Comprehensive Basic Science Self-Assessment
Time Remaining:
1 hr 14 min 7 sec
® Mark
15. A 74-year-ofd man comes to the office because of a 3-week history of progressive shortness of breath with exertion. He has had difficulty climbing one flight of stairs. He also has a 20-year history of poorly controlled hypertension, for which he rarely takes his
prescribed medication and osteoarthritis treated with naproxen daily. His temperature is 37.1CC (98.8QF); puise is 76/min, respira:ions are 20/min, and blood pressure is 180/98 mm Hg. Pulse oximetry on room air shows an oxygen saturation of 92%. Physical
examination shows 10-cm jugular venous distention above the sternal angle. Crackies are heard over both lung bases. An S4 is heard. Echocardiography shows a normal ejection fraction. Which of the following best explains this patient's symptom?
) A ) Decreased myocardiaE oxygen supply
' J B) External compression of the right ventricle
0 C) Impaired left ventricular relaxation
' J D ) Increased turbulent flow across the mitral valve
J E } Obstruction of the left ventricular outflow tract
O 0 I* 0 0 0
Previous Next Lab Values Calculator Review Help Pause
Exam Section 3: Item 16 ol 50 National Board of Medical Examiners^
Comprehensive Basic Science Self-Assessment
Time Remaining:
1 hr 14 min 4 sec
® Mark
25i
16. A study is conducted to assess the effect of a new selective estrogen receptor modulator (SERM) on bone fractures. Two thousand women with a history of vertebral fracture and 5000 women with no
fracture history are randomly assigned to a placebo or to SERM at a dose of 60 mg or 120 mg. The graph shows the percent of women who had a new fracture during 5 years of foJIow-up
(± standard error of the mean). Which of the following is the most accurate conclusion about preventing new fractures based on these data?
A ) High-dose SERM treatment is more effective than low-dose SERM treatment
B) Low-dose SERM treatment is more effective than high-dose SERM treatment
3 C) Placebo treatment is effective only in women with a history of vertebral fracture
D) SERM treatment is effective only in women with a history of vertebral fracture
E) SERM treatment is effective only in women with no history of fracture
T
£ 20
o
15 T T
0)
10
£ T T T
5 -
o
o
0
No fracture history Fracture history
Placebo
SERM 60 mg
SERM 120 mg
O
* 9 0 0
Previous Next Lab Values Calculator Review Help Pause
Exam Section 3: Item 17 of 50 National Board of Medical Examiners^
Comprehensive Basic Science Self-Assessment
Time Remaining:
1 hr 14 min 0 sec
® Mark
17. A 23-year-ofd man cuts his tip inadvertently while shaving. Seconds after the injury the bleeding nearly stops. Which of the following mechanisms is the most likely cause of the early rapid control of blood loss in this man?
3 A ) Activation of antithrombin Hi
^ 8 } Localized secretion of endothelin
Q G) Oxygen-stimulated cleavage of thromboplastin
D) Polymerization of fibrin
«r
* m O m
Previous Next Lab Values Calculator Review Help Pause
Exam Section 3: Item 13 ol 50 National Board of Medical Examiners^
Comprehensive Basic Science Self-Assessment
Time Remaining:
1 hr 13 min 57 sec
9 Mark
18. A 21-year-old woman comes to the physician for a routine health maintenance examination. Her 3-year-old sister was recently diagnosed with cystic fibrosis. Prior to any genetic testing; which of the following best approximates this patient's risk of being a
heterozygote for cystic fibrosis?
O A) 0
O 8) 1/2
O C) 1/4
o D) 2/3
O E} 3/4
O * 9 0 0
Previous Next Lab Values Calculator Review Help Pause
Exam Section 3: Item 19 of 50 National Board of Medical Examiners^
Comprehensive Basic Science Self-Assessment
Time Remaining:
1 hr 13 min 54 sec
9 Mark
19. A 25-year-old woman with a 3-year history of celiac disease comes to the physician because of a markedly itchy rash on her scalp, arms, buttocks: and legs for 5 weeks. Examination of the skin shows many crusted papules surrounded by excoriations over
the upper and lower extremities and a few vesicles on the buttocks. A skin biopsy specimen shows the vesicles to be subepidermal. Direct immunofluorescence of the vesicles shows granular deposits of IgA in the epidermal-dermal junction. Which of the
following is the most likely diagnosis?
A ) Bullous pemphigoid
3 B) Dermatitis herpetiformis
o G) Dyshidrotic eczema
3 D ) Epidermolysis bullosa acquisita
J E) Pemphigus
* m O m
Previous Next Lab Values Calculator Review Help Pause
Exam Section 3: Item 20 ol 50 National Board of Medical Examiners^
Comprehensive Basic Science Self-Assessment
Time Remaining:
1 hr 13 min 50 sec
9 Mark
20. A 1-year-old boy is brought to the physician for a weli-chiid examination. The mother is concerned because her son's fine, pale hair has not changed color since birth. His eyes are blue. During ophthalmologic examination, the patient turns away from the
flashlight and starts crying. Which of the following is the most likely cause of the paie skin color in this patient?
. A ) Aberrant migration of neural crest cells
8) Decreased number of epidermal melanocytes
. J C) immune destruction of melanocytes
'
3 D) inability to produce melanin
. E } Melanin dropout to the dermis
O * m O m
Previous Next Lab Values Calculator Review Help Pause
NBME 28 Q.pdf
NBME 28 Q.pdf
NBME 28 Q.pdf
NBME 28 Q.pdf
NBME 28 Q.pdf
NBME 28 Q.pdf
NBME 28 Q.pdf
NBME 28 Q.pdf
NBME 28 Q.pdf
NBME 28 Q.pdf
NBME 28 Q.pdf
NBME 28 Q.pdf
NBME 28 Q.pdf
NBME 28 Q.pdf
NBME 28 Q.pdf
NBME 28 Q.pdf
NBME 28 Q.pdf
NBME 28 Q.pdf
NBME 28 Q.pdf
NBME 28 Q.pdf
NBME 28 Q.pdf
NBME 28 Q.pdf
NBME 28 Q.pdf
NBME 28 Q.pdf
NBME 28 Q.pdf
NBME 28 Q.pdf
NBME 28 Q.pdf
NBME 28 Q.pdf
NBME 28 Q.pdf
NBME 28 Q.pdf
NBME 28 Q.pdf
NBME 28 Q.pdf
NBME 28 Q.pdf
NBME 28 Q.pdf
NBME 28 Q.pdf
NBME 28 Q.pdf
NBME 28 Q.pdf
NBME 28 Q.pdf
NBME 28 Q.pdf
NBME 28 Q.pdf
NBME 28 Q.pdf
NBME 28 Q.pdf
NBME 28 Q.pdf
NBME 28 Q.pdf
NBME 28 Q.pdf
NBME 28 Q.pdf
NBME 28 Q.pdf
NBME 28 Q.pdf
NBME 28 Q.pdf
NBME 28 Q.pdf
NBME 28 Q.pdf
NBME 28 Q.pdf
NBME 28 Q.pdf
NBME 28 Q.pdf
NBME 28 Q.pdf
NBME 28 Q.pdf
NBME 28 Q.pdf
NBME 28 Q.pdf
NBME 28 Q.pdf
NBME 28 Q.pdf
NBME 28 Q.pdf
NBME 28 Q.pdf
NBME 28 Q.pdf
NBME 28 Q.pdf
NBME 28 Q.pdf
NBME 28 Q.pdf
NBME 28 Q.pdf
NBME 28 Q.pdf
NBME 28 Q.pdf
NBME 28 Q.pdf
NBME 28 Q.pdf
NBME 28 Q.pdf
NBME 28 Q.pdf
NBME 28 Q.pdf
NBME 28 Q.pdf
NBME 28 Q.pdf
NBME 28 Q.pdf
NBME 28 Q.pdf
NBME 28 Q.pdf
NBME 28 Q.pdf

More Related Content

What's hot

Eronhii mergejliin emch jishig soril-2
Eronhii mergejliin emch jishig soril-2Eronhii mergejliin emch jishig soril-2
Eronhii mergejliin emch jishig soril-2Gantulga Nyamdorj
 
антибактериал эмийн тухай, бүлэг, ангилал,
антибактериал эмийн тухай, бүлэг, ангилал,антибактериал эмийн тухай, бүлэг, ангилал,
антибактериал эмийн тухай, бүлэг, ангилал,Bilguun To Gold Or
 
Exercícios de revisão 9º ano-simave- números naturais e inteiros
Exercícios de revisão 9º ano-simave- números naturais e inteirosExercícios de revisão 9º ano-simave- números naturais e inteiros
Exercícios de revisão 9º ano-simave- números naturais e inteirosRenata Alves Xavier
 
Ficha revisões fracções
Ficha revisões fracçõesFicha revisões fracções
Ficha revisões fracçõesJoao Ferreira
 
Teste 1º bimestre ( números inteiros)
Teste 1º bimestre ( números inteiros)Teste 1º bimestre ( números inteiros)
Teste 1º bimestre ( números inteiros)Olicio Silva
 
Noir bulchirhain tsochmog urewsel
Noir bulchirhain tsochmog urewselNoir bulchirhain tsochmog urewsel
Noir bulchirhain tsochmog urewselnytt103103
 
פרק 1.2 מאגר שאלות802 סדרה חשבונית וסדרה הנדסית - פתרונות
פרק 1.2 מאגר שאלות802 סדרה חשבונית וסדרה הנדסית - פתרונותפרק 1.2 מאגר שאלות802 סדרה חשבונית וסדרה הנדסית - פתרונות
פרק 1.2 מאגר שאלות802 סדרה חשבונית וסדרה הנדסית - פתרונותtelnof
 
פרק 2.1 מאגרשאלות: סטטיסטיקה - פתרונות
פרק 2.1 מאגרשאלות: סטטיסטיקה - פתרונותפרק 2.1 מאגרשאלות: סטטיסטיקה - פתרונות
פרק 2.1 מאגרשאלות: סטטיסטיקה - פתרונותtelnof
 
Cruzadinha Matemática 03
Cruzadinha Matemática 03Cruzadinha Matemática 03
Cruzadinha Matemática 03Prof. Materaldo
 

What's hot (20)

Eronhii mergejliin emch jishig soril-2
Eronhii mergejliin emch jishig soril-2Eronhii mergejliin emch jishig soril-2
Eronhii mergejliin emch jishig soril-2
 
Medrel 1
Medrel 1Medrel 1
Medrel 1
 
Medrel sudlal
Medrel sudlalMedrel sudlal
Medrel sudlal
 
антибактериал эмийн тухай, бүлэг, ангилал,
антибактериал эмийн тухай, бүлэг, ангилал,антибактериал эмийн тухай, бүлэг, ангилал,
антибактериал эмийн тухай, бүлэг, ангилал,
 
Exercícios de revisão 9º ano-simave- números naturais e inteiros
Exercícios de revisão 9º ano-simave- números naturais e inteirosExercícios de revisão 9º ano-simave- números naturais e inteiros
Exercícios de revisão 9º ano-simave- números naturais e inteiros
 
Dotor sudlal
Dotor sudlalDotor sudlal
Dotor sudlal
 
Medrel
MedrelMedrel
Medrel
 
Ficha revisões fracções
Ficha revisões fracçõesFicha revisões fracções
Ficha revisões fracções
 
Mes zasal sudlal
Mes zasal sudlalMes zasal sudlal
Mes zasal sudlal
 
área de figuras planas
área de figuras planasárea de figuras planas
área de figuras planas
 
Medrel
MedrelMedrel
Medrel
 
Teste 1º bimestre ( números inteiros)
Teste 1º bimestre ( números inteiros)Teste 1º bimestre ( números inteiros)
Teste 1º bimestre ( números inteiros)
 
Noir bulchirhain tsochmog urewsel
Noir bulchirhain tsochmog urewselNoir bulchirhain tsochmog urewsel
Noir bulchirhain tsochmog urewsel
 
פרק 1.2 מאגר שאלות802 סדרה חשבונית וסדרה הנדסית - פתרונות
פרק 1.2 מאגר שאלות802 סדרה חשבונית וסדרה הנדסית - פתרונותפרק 1.2 מאגר שאלות802 סדרה חשבונית וסדרה הנדסית - פתרונות
פרק 1.2 מאגר שאלות802 סדרה חשבונית וסדרה הנדסית - פתרונות
 
פרק 2.1 מאגרשאלות: סטטיסטיקה - פתרונות
פרק 2.1 מאגרשאלות: סטטיסטיקה - פתרונותפרק 2.1 מאגרשאלות: סטטיסטיקה - פתרונות
פרק 2.1 מאגרשאלות: סטטיסטיקה - פתרונות
 
Cruzadinha Matemática 03
Cruzadinha Matemática 03Cruzadinha Matemática 03
Cruzadinha Matemática 03
 
Buduun gedesni mes zasal
Buduun gedesni mes zasalBuduun gedesni mes zasal
Buduun gedesni mes zasal
 
Produto Notáveis
Produto NotáveisProduto Notáveis
Produto Notáveis
 
Huuhdiin loor
Huuhdiin loorHuuhdiin loor
Huuhdiin loor
 
Emnelzuia 1
Emnelzuia 1Emnelzuia 1
Emnelzuia 1
 

Similar to NBME 28 Q.pdf

UWSA 1 2021 For USMLE Step 1 Exam
UWSA 1 2021 For USMLE Step 1 ExamUWSA 1 2021 For USMLE Step 1 Exam
UWSA 1 2021 For USMLE Step 1 Examusmlematerialsnet
 
Golden book for Medicine OSCE: First View
Golden book for Medicine OSCE: First ViewGolden book for Medicine OSCE: First View
Golden book for Medicine OSCE: First ViewMan B Paudyal
 
History taking a case based discussion
History taking a case based discussionHistory taking a case based discussion
History taking a case based discussionPritom Das
 
Presentation2-1.pptx
Presentation2-1.pptxPresentation2-1.pptx
Presentation2-1.pptxssuser5e81db
 
AIIMS Medicine Quiz
AIIMS Medicine QuizAIIMS Medicine Quiz
AIIMS Medicine QuizUmang Arora
 
Mod 1 case 2022
Mod 1 case 2022Mod 1 case 2022
Mod 1 case 2022vetindex4
 
Krok 2 - 2007 Question Paper (General Medicine)
Krok 2 - 2007 Question Paper (General Medicine)Krok 2 - 2007 Question Paper (General Medicine)
Krok 2 - 2007 Question Paper (General Medicine)Eneutron
 
Book 2006 krok-2
Book 2006 krok-2Book 2006 krok-2
Book 2006 krok-2Raj Twix
 
Book 2011 krok 2
Book 2011 krok 2Book 2011 krok 2
Book 2011 krok 2Raj Twix
 
Case Study Assignment for Unit IIIPurpose The purpose of th.docx
Case Study Assignment for Unit IIIPurpose The purpose of th.docxCase Study Assignment for Unit IIIPurpose The purpose of th.docx
Case Study Assignment for Unit IIIPurpose The purpose of th.docxwendolynhalbert
 
Abnormal uterine bleeding
Abnormal uterine bleeding Abnormal uterine bleeding
Abnormal uterine bleeding BhawanaYadav9
 
Book 2009 krok 2
Book 2009 krok 2Book 2009 krok 2
Book 2009 krok 2Raj Twix
 
Book 2007 krok 2
Book 2007 krok 2Book 2007 krok 2
Book 2007 krok 2Raj Twix
 
Gastroenterology ppt 2
Gastroenterology ppt  2Gastroenterology ppt  2
Gastroenterology ppt 2UPUL UDAYARAJ
 
UCMS: Prelim Medical Quiz2018
UCMS: Prelim Medical Quiz2018 UCMS: Prelim Medical Quiz2018
UCMS: Prelim Medical Quiz2018 Illuminous
 
Krok 2 - 2015 Question Paper (General Medicine)
Krok 2 - 2015 Question Paper (General Medicine)Krok 2 - 2015 Question Paper (General Medicine)
Krok 2 - 2015 Question Paper (General Medicine)Eneutron
 

Similar to NBME 28 Q.pdf (20)

UWSA 1 2021 For USMLE Step 1 Exam
UWSA 1 2021 For USMLE Step 1 ExamUWSA 1 2021 For USMLE Step 1 Exam
UWSA 1 2021 For USMLE Step 1 Exam
 
Golden book for Medicine OSCE: First View
Golden book for Medicine OSCE: First ViewGolden book for Medicine OSCE: First View
Golden book for Medicine OSCE: First View
 
Segundo simulador primera parte
Segundo simulador primera parteSegundo simulador primera parte
Segundo simulador primera parte
 
DNB Pediatrics OSCE June 2013
DNB Pediatrics OSCE June 2013DNB Pediatrics OSCE June 2013
DNB Pediatrics OSCE June 2013
 
History taking a case based discussion
History taking a case based discussionHistory taking a case based discussion
History taking a case based discussion
 
Presentation2-1.pptx
Presentation2-1.pptxPresentation2-1.pptx
Presentation2-1.pptx
 
AIIMS Medicine Quiz
AIIMS Medicine QuizAIIMS Medicine Quiz
AIIMS Medicine Quiz
 
Mod 1 case 2022
Mod 1 case 2022Mod 1 case 2022
Mod 1 case 2022
 
Krok 2 - 2007 Question Paper (General Medicine)
Krok 2 - 2007 Question Paper (General Medicine)Krok 2 - 2007 Question Paper (General Medicine)
Krok 2 - 2007 Question Paper (General Medicine)
 
Book 2006 krok-2
Book 2006 krok-2Book 2006 krok-2
Book 2006 krok-2
 
Book 2011 krok 2
Book 2011 krok 2Book 2011 krok 2
Book 2011 krok 2
 
Case Study Assignment for Unit IIIPurpose The purpose of th.docx
Case Study Assignment for Unit IIIPurpose The purpose of th.docxCase Study Assignment for Unit IIIPurpose The purpose of th.docx
Case Study Assignment for Unit IIIPurpose The purpose of th.docx
 
Principles of Hormone Testing
Principles of Hormone TestingPrinciples of Hormone Testing
Principles of Hormone Testing
 
Senior Medillectuals Mains
Senior Medillectuals MainsSenior Medillectuals Mains
Senior Medillectuals Mains
 
Abnormal uterine bleeding
Abnormal uterine bleeding Abnormal uterine bleeding
Abnormal uterine bleeding
 
Book 2009 krok 2
Book 2009 krok 2Book 2009 krok 2
Book 2009 krok 2
 
Book 2007 krok 2
Book 2007 krok 2Book 2007 krok 2
Book 2007 krok 2
 
Gastroenterology ppt 2
Gastroenterology ppt  2Gastroenterology ppt  2
Gastroenterology ppt 2
 
UCMS: Prelim Medical Quiz2018
UCMS: Prelim Medical Quiz2018 UCMS: Prelim Medical Quiz2018
UCMS: Prelim Medical Quiz2018
 
Krok 2 - 2015 Question Paper (General Medicine)
Krok 2 - 2015 Question Paper (General Medicine)Krok 2 - 2015 Question Paper (General Medicine)
Krok 2 - 2015 Question Paper (General Medicine)
 

Recently uploaded

Bangalore whatsapp Number Just VIP Brookefield 100% Genuine at your Door Step
Bangalore whatsapp Number Just VIP Brookefield 100% Genuine at your Door StepBangalore whatsapp Number Just VIP Brookefield 100% Genuine at your Door Step
Bangalore whatsapp Number Just VIP Brookefield 100% Genuine at your Door Stepdarmandersingh4580
 
Sell 5cladba adbb JWH-018 5FADB in stock
Sell 5cladba adbb JWH-018 5FADB in stockSell 5cladba adbb JWH-018 5FADB in stock
Sell 5cladba adbb JWH-018 5FADB in stocktammysayles9
 
Histology of Epithelium - Dr Muhammad Ali Rabbani - Medicose Academics
Histology of Epithelium - Dr Muhammad Ali Rabbani - Medicose AcademicsHistology of Epithelium - Dr Muhammad Ali Rabbani - Medicose Academics
Histology of Epithelium - Dr Muhammad Ali Rabbani - Medicose AcademicsMedicoseAcademics
 
Young & Hot ℂall Girls Patna 8250077686 WhatsApp Number Best Rates of Patna ℂ...
Young & Hot ℂall Girls Patna 8250077686 WhatsApp Number Best Rates of Patna ℂ...Young & Hot ℂall Girls Patna 8250077686 WhatsApp Number Best Rates of Patna ℂ...
Young & Hot ℂall Girls Patna 8250077686 WhatsApp Number Best Rates of Patna ℂ...Model Neeha Mumbai
 
SEMESTER-V CHILD HEALTH NURSING-UNIT-1-INTRODUCTION.pdf
SEMESTER-V CHILD HEALTH NURSING-UNIT-1-INTRODUCTION.pdfSEMESTER-V CHILD HEALTH NURSING-UNIT-1-INTRODUCTION.pdf
SEMESTER-V CHILD HEALTH NURSING-UNIT-1-INTRODUCTION.pdfSachin Sharma
 
Gallbladder Double-Diverticular: A Case Report المرارة مزدوجة التج: تقرير حالة
Gallbladder Double-Diverticular: A Case Report  المرارة مزدوجة التج: تقرير حالةGallbladder Double-Diverticular: A Case Report  المرارة مزدوجة التج: تقرير حالة
Gallbladder Double-Diverticular: A Case Report المرارة مزدوجة التج: تقرير حالةMohamad محمد Al-Gailani الكيلاني
 
Signs It’s Time for Physiotherapy Sessions Prioritizing Wellness
Signs It’s Time for Physiotherapy Sessions Prioritizing WellnessSigns It’s Time for Physiotherapy Sessions Prioritizing Wellness
Signs It’s Time for Physiotherapy Sessions Prioritizing WellnessGokuldas Hospital
 
Quality control tests of suppository ...
Quality control tests  of suppository ...Quality control tests  of suppository ...
Quality control tests of suppository ...Hasnat Tariq
 
Failure to thrive in neonates and infants + pediatric case.pptx
Failure to thrive in neonates and infants  + pediatric case.pptxFailure to thrive in neonates and infants  + pediatric case.pptx
Failure to thrive in neonates and infants + pediatric case.pptxclaviclebrown44
 
duus neurology.pdf anatomy. phisiology///
duus neurology.pdf anatomy. phisiology///duus neurology.pdf anatomy. phisiology///
duus neurology.pdf anatomy. phisiology///sofia95y
 
Young & Hot Surat ℂall Girls Vesu 8527049040 WhatsApp AnyTime Best Surat ℂall...
Young & Hot Surat ℂall Girls Vesu 8527049040 WhatsApp AnyTime Best Surat ℂall...Young & Hot Surat ℂall Girls Vesu 8527049040 WhatsApp AnyTime Best Surat ℂall...
Young & Hot Surat ℂall Girls Vesu 8527049040 WhatsApp AnyTime Best Surat ℂall...Neelam SharmaI11
 
Mgr university bsc nursing adult health previous question paper with answers
Mgr university  bsc nursing adult health previous question paper with answersMgr university  bsc nursing adult health previous question paper with answers
Mgr university bsc nursing adult health previous question paper with answersShafnaP5
 
Stereochemistry & Asymmetric Synthesis.pptx
Stereochemistry & Asymmetric Synthesis.pptxStereochemistry & Asymmetric Synthesis.pptx
Stereochemistry & Asymmetric Synthesis.pptxAkanshaBhatnagar7
 
Overview on the Automatic pill identifier
Overview on the Automatic pill identifierOverview on the Automatic pill identifier
Overview on the Automatic pill identifierNidhi Joshi
 
Report Back from SGO: What’s the Latest in Ovarian Cancer?
Report Back from SGO: What’s the Latest in Ovarian Cancer?Report Back from SGO: What’s the Latest in Ovarian Cancer?
Report Back from SGO: What’s the Latest in Ovarian Cancer?bkling
 
Young & Hot Surat ℂall Girls Dindoli 8527049040 WhatsApp AnyTime Best Surat ℂ...
Young & Hot Surat ℂall Girls Dindoli 8527049040 WhatsApp AnyTime Best Surat ℂ...Young & Hot Surat ℂall Girls Dindoli 8527049040 WhatsApp AnyTime Best Surat ℂ...
Young & Hot Surat ℂall Girls Dindoli 8527049040 WhatsApp AnyTime Best Surat ℂ...Neelam SharmaI11
 
Vesu + ℂall Girls Serviℂe Surat (Adult Only) 8849756361 Esℂort Serviℂe 24x7 C...
Vesu + ℂall Girls Serviℂe Surat (Adult Only) 8849756361 Esℂort Serviℂe 24x7 C...Vesu + ℂall Girls Serviℂe Surat (Adult Only) 8849756361 Esℂort Serviℂe 24x7 C...
Vesu + ℂall Girls Serviℂe Surat (Adult Only) 8849756361 Esℂort Serviℂe 24x7 C...anushka vermaI11
 
Unveiling Alcohol Withdrawal Syndrome: exploring it's hidden depths
Unveiling Alcohol Withdrawal Syndrome: exploring it's hidden depthsUnveiling Alcohol Withdrawal Syndrome: exploring it's hidden depths
Unveiling Alcohol Withdrawal Syndrome: exploring it's hidden depthsYash Garg
 
Cytoskeleton and Cell Inclusions - Dr Muhammad Ali Rabbani - Medicose Academics
Cytoskeleton and Cell Inclusions - Dr Muhammad Ali Rabbani - Medicose AcademicsCytoskeleton and Cell Inclusions - Dr Muhammad Ali Rabbani - Medicose Academics
Cytoskeleton and Cell Inclusions - Dr Muhammad Ali Rabbani - Medicose AcademicsMedicoseAcademics
 

Recently uploaded (20)

Bangalore whatsapp Number Just VIP Brookefield 100% Genuine at your Door Step
Bangalore whatsapp Number Just VIP Brookefield 100% Genuine at your Door StepBangalore whatsapp Number Just VIP Brookefield 100% Genuine at your Door Step
Bangalore whatsapp Number Just VIP Brookefield 100% Genuine at your Door Step
 
Sell 5cladba adbb JWH-018 5FADB in stock
Sell 5cladba adbb JWH-018 5FADB in stockSell 5cladba adbb JWH-018 5FADB in stock
Sell 5cladba adbb JWH-018 5FADB in stock
 
Histology of Epithelium - Dr Muhammad Ali Rabbani - Medicose Academics
Histology of Epithelium - Dr Muhammad Ali Rabbani - Medicose AcademicsHistology of Epithelium - Dr Muhammad Ali Rabbani - Medicose Academics
Histology of Epithelium - Dr Muhammad Ali Rabbani - Medicose Academics
 
Best medicine 100% Effective&Safe Mifepristion ௵+918133066128௹Abortion pills ...
Best medicine 100% Effective&Safe Mifepristion ௵+918133066128௹Abortion pills ...Best medicine 100% Effective&Safe Mifepristion ௵+918133066128௹Abortion pills ...
Best medicine 100% Effective&Safe Mifepristion ௵+918133066128௹Abortion pills ...
 
Young & Hot ℂall Girls Patna 8250077686 WhatsApp Number Best Rates of Patna ℂ...
Young & Hot ℂall Girls Patna 8250077686 WhatsApp Number Best Rates of Patna ℂ...Young & Hot ℂall Girls Patna 8250077686 WhatsApp Number Best Rates of Patna ℂ...
Young & Hot ℂall Girls Patna 8250077686 WhatsApp Number Best Rates of Patna ℂ...
 
SEMESTER-V CHILD HEALTH NURSING-UNIT-1-INTRODUCTION.pdf
SEMESTER-V CHILD HEALTH NURSING-UNIT-1-INTRODUCTION.pdfSEMESTER-V CHILD HEALTH NURSING-UNIT-1-INTRODUCTION.pdf
SEMESTER-V CHILD HEALTH NURSING-UNIT-1-INTRODUCTION.pdf
 
Gallbladder Double-Diverticular: A Case Report المرارة مزدوجة التج: تقرير حالة
Gallbladder Double-Diverticular: A Case Report  المرارة مزدوجة التج: تقرير حالةGallbladder Double-Diverticular: A Case Report  المرارة مزدوجة التج: تقرير حالة
Gallbladder Double-Diverticular: A Case Report المرارة مزدوجة التج: تقرير حالة
 
Signs It’s Time for Physiotherapy Sessions Prioritizing Wellness
Signs It’s Time for Physiotherapy Sessions Prioritizing WellnessSigns It’s Time for Physiotherapy Sessions Prioritizing Wellness
Signs It’s Time for Physiotherapy Sessions Prioritizing Wellness
 
Quality control tests of suppository ...
Quality control tests  of suppository ...Quality control tests  of suppository ...
Quality control tests of suppository ...
 
Failure to thrive in neonates and infants + pediatric case.pptx
Failure to thrive in neonates and infants  + pediatric case.pptxFailure to thrive in neonates and infants  + pediatric case.pptx
Failure to thrive in neonates and infants + pediatric case.pptx
 
duus neurology.pdf anatomy. phisiology///
duus neurology.pdf anatomy. phisiology///duus neurology.pdf anatomy. phisiology///
duus neurology.pdf anatomy. phisiology///
 
Young & Hot Surat ℂall Girls Vesu 8527049040 WhatsApp AnyTime Best Surat ℂall...
Young & Hot Surat ℂall Girls Vesu 8527049040 WhatsApp AnyTime Best Surat ℂall...Young & Hot Surat ℂall Girls Vesu 8527049040 WhatsApp AnyTime Best Surat ℂall...
Young & Hot Surat ℂall Girls Vesu 8527049040 WhatsApp AnyTime Best Surat ℂall...
 
Mgr university bsc nursing adult health previous question paper with answers
Mgr university  bsc nursing adult health previous question paper with answersMgr university  bsc nursing adult health previous question paper with answers
Mgr university bsc nursing adult health previous question paper with answers
 
Stereochemistry & Asymmetric Synthesis.pptx
Stereochemistry & Asymmetric Synthesis.pptxStereochemistry & Asymmetric Synthesis.pptx
Stereochemistry & Asymmetric Synthesis.pptx
 
Overview on the Automatic pill identifier
Overview on the Automatic pill identifierOverview on the Automatic pill identifier
Overview on the Automatic pill identifier
 
Report Back from SGO: What’s the Latest in Ovarian Cancer?
Report Back from SGO: What’s the Latest in Ovarian Cancer?Report Back from SGO: What’s the Latest in Ovarian Cancer?
Report Back from SGO: What’s the Latest in Ovarian Cancer?
 
Young & Hot Surat ℂall Girls Dindoli 8527049040 WhatsApp AnyTime Best Surat ℂ...
Young & Hot Surat ℂall Girls Dindoli 8527049040 WhatsApp AnyTime Best Surat ℂ...Young & Hot Surat ℂall Girls Dindoli 8527049040 WhatsApp AnyTime Best Surat ℂ...
Young & Hot Surat ℂall Girls Dindoli 8527049040 WhatsApp AnyTime Best Surat ℂ...
 
Vesu + ℂall Girls Serviℂe Surat (Adult Only) 8849756361 Esℂort Serviℂe 24x7 C...
Vesu + ℂall Girls Serviℂe Surat (Adult Only) 8849756361 Esℂort Serviℂe 24x7 C...Vesu + ℂall Girls Serviℂe Surat (Adult Only) 8849756361 Esℂort Serviℂe 24x7 C...
Vesu + ℂall Girls Serviℂe Surat (Adult Only) 8849756361 Esℂort Serviℂe 24x7 C...
 
Unveiling Alcohol Withdrawal Syndrome: exploring it's hidden depths
Unveiling Alcohol Withdrawal Syndrome: exploring it's hidden depthsUnveiling Alcohol Withdrawal Syndrome: exploring it's hidden depths
Unveiling Alcohol Withdrawal Syndrome: exploring it's hidden depths
 
Cytoskeleton and Cell Inclusions - Dr Muhammad Ali Rabbani - Medicose Academics
Cytoskeleton and Cell Inclusions - Dr Muhammad Ali Rabbani - Medicose AcademicsCytoskeleton and Cell Inclusions - Dr Muhammad Ali Rabbani - Medicose Academics
Cytoskeleton and Cell Inclusions - Dr Muhammad Ali Rabbani - Medicose Academics
 

NBME 28 Q.pdf

  • 1. Tutorial Section: Screen 10 of 10 National Board of Medical Examiners® Tutorial Time Remaining: 14 min 18 sec This concludes the tutorial. Click "Next" to begin your assessment. L J 7 Previous Next Help Pause
  • 2. Exam Section 1: Item 1 of 50 National Board of Medical Examiners^ Comprehensive Basic Science Self-Assessment Time Remaining: 1 hr 14 min 58 sec 9 Mark 1. A 25-year-old man is brought to the emergency department because of severe abdominal pain, nausea, and vomiting for 1 hour. The pain originates in the left flank and radiates to his groin. His putse is 100/min, respirations are 18/min, and blood pressure is 150/100 mm Hg. Physical examination shows tenderness of the left flank and the left lower quadrant of the abdomen. Bowel sounds are mildly hypoactive. Test of the stoof for occult blood is negative. Which of the following best explains these findings? . A ) Colon neoplasm B) Diverticulitis O C) Epididymitis 0 D) Renalinfarction . E) Torsion of the testis F) Ureteral calculus 9 0 0 Next Lab Values Calculator Review Help Pause
  • 3. Exam Section 1: Item 2 of 50 National Board of Medical Examiners^ Comprehensive Basic Science Self-Assessment Time Remaining: 1 hr 13 min 57 sec 9 Mark 2. Which of the following types of sensory information is compromised by lesions of the structure at site X in the photograph shown? A ) Conscious proprioception B) Pain sensation 0 C) Two-point discrimination D ) Unconscious proprioception E: ) Vibration sense & * f* Previous Next Lab Values Calculator Review Help Pause
  • 4. Exam Section 1: Item 3 of 50 National Board of Medical Examiners^ Comprehensive Basic Science Self-Assessment Time Remaining: 1 hr 13 min 51 sec 9 Mark 3. Moving the forearm against resistance from palm-down to palm-up (supination) position requires the use of which of the following muscles? 3 A ) Biceps brachii B) Brachialis O C) Triceps D) Flexor carpi radialis ! J E) Pronator teres 9 0 0 Previous Next Lab Values Calculator Review Help Pause
  • 5. Exam Section 1: Item 4 of 50 National Board of Medical Examiners^ Comprehensive Basic Science Self-Assessment Time Remaining: 1 hr 13 min 42 sec ® Mark 4. A 35-year-old man is admitted to the hospital because of a 5-day history of fever and dyspnea He underwent a bone marrow transplantation G months ago: the procedure was complicated by severe graft-versus-host disease. His temperature is 38- C (100.4T). and respirations are 30/min. Scattered crackles are heard on auscultation of the chest. A chest x-ray shows patchy infiltrates. A transbronchial biopsy specimen shows findings consistent with cytomegalovirus infection. Intravenous administration of ganciclovir is begun. This drug interferes with the function of which of the following enzymes? A ) DNApolymerase 3 B) Integrase o C) Reverse transcriptase 3 0 ) RNA polymerase J E } Thymidine kinase * m O m Previous Next Lab Values Calculator Review Help Pause
  • 6. Exam Section 1: Item 5 of 50 National Board of Medical Examiners^ Comprehensive Basic Science Self-Assessment Time Remaining: 1 hr 13 min 36 sec 9 Mark o 200- T> » £ £ S 100- <D co 0 25 27 1 Cycle (day) M - menstruation 5. The graph shows changes in serum estradiol concentration during a normal menstrual cycle. Which of the following ovarian cells is primarily responsible for the aromatizaiion of androstenedione to estradiol at the time indicated by the arrow? A ) Granulosa ' B) Luteaf . ) G) Stromal D ) Theca externa E) Theca interna O * r - o o o Previous Next Lab Values Calculator Review Help Pause
  • 7. Exam Section 1: Item 6 of 50 National Board of Medical Examiners^ Comprehensive Basic Science Self-Assessment Time Remaining: 1 hr 13 min 32 sec ® Mark 6. A 14-year-old girl is brought to the physician by her parents because of a 1-month history of a rash that appears with sun exposure, Her parents tell the physician that she has been eating fiittle food. Physical examination shows a pruritic rash on the exposed areas of the body. Her serum tryptophan concentration is decreased Urine studies show increased excretion of amino acids: predominantly alanine, isoleucine, leucine, phenylalanine: tryptophan, and valine. Production of which of the following vitamins is most likely impaired in this patient? A ) Niacin 0 B) Vitamin B ,(thiamine) 0 G) Vitamin B2 (riboflavin) 0 0 ) Vitamin B5 (pantothenic add) E) Vitamin C * m O m Previous Next Lab Values Calculator Review Help Pause
  • 8. Exam Section 1 : Item 7 of 50 National Board of Medical Examiners^ Comprehensive Basic Science Self-Assessment Time Remaining: 1 hr 13 min 29 sec ® Mark 7. Aminoglycoside antibiotics are used for their synergistic action against bacteria, in combination with other agents. These antibiotics demonstrate in vitro synergy for several bacterial species when combined with which of the following classes of antibiotics? 3 A ) Fluoroquinolones S) Macrolides O C) Penicillins D) Rifamycrns ! J E) Tetracyclines * m O m Previous Next Lab Values Calculator Review Help Pause
  • 9. Exam Section 1: Item 8 of 50 National Board of Medical Examiners^ Comprehensive Basic Science Self-Assessment Time Remaining: 1 hr 13 min 26 sec ® Mark 8. A 16-year-old boy with moderate intellectual disability is brought to the physician for a routine examination. There is a family history of mild and moderate intellectual disability in his mother and brother, respectively. Physical examination shows a long face; prominent ears, and moderately enlarged testicles. Which of the following best describes the genetic mechanism of this patient's disorder? . A ) Mutation in a mitochondrial gene 8) Presence of an extra sex chromosome . J C) Translocation of a portion of an autosome ' 3 D) Trinucleotide repeat mutation on the X chromosome . E } Trisomy of an autosome O * 9 0 0 Previous Next Lab Values Calculator Review Help Pause
  • 10. Exam Section 1: Item 8 of 50 National Board of Medical Examiners^ Comprehensive Basic Science Self-Assessment MeaseWait ® Mark 9. A 75-year-old woman comes to the physician because of a 3-month history of an enlarging lesion on her face. Physical examination shows a 1, 5-cm, brown-black, mottled, scaly lesion with irregular borders. Microscopic examination of a biopsy specimen of the lesion shows atypical melanocytes spread along the basilar layer of the epidermis. Which of the following is the most likely cause of these findings? A ) Acanthosis nigricans 8) Actinic keratosis C) Compound nevus D) Lentigo maligna E) Seborrheic keratosis O * m O m Previous Next Lab Values Calculator Review Help Pause
  • 11. Exam Section 1: Item 10 ol 50 National Board of Medical Examiners^ Comprehensive Basic Science Self-Assessment Time Remaining: 1 hr 13 min 15 sec 9 Mark 10. A physician wishes to determine the proportion of newborns delivered at a local hospital who had a diagnosis of congenital heart disease within the past year. Which of the following statistical measurements best describes these data? Q A ) Attributable risk S) Incidence O C) Odds ratio D) Prevalence ' J E) Relative risk * m O m Previous Next Lab Values Calculator Review Help Pause
  • 12. Exam Section 1: Item 11 of 50 National Board of Medical Examiners^ Comprehensive Basic Science Self-Assessment Time Remaining: 1 hr 13 min 11 sec ® Mark 11. A 25-year-old woman comes to the physician after her blood pressure was found to be 130/105 mm Hg at a health fair. She takes no medications. There is no family history of hypertension. Her last menstrual period was 1 week ago. Her blood pressure today is 180/110 mm Hg. Bilateral abdominal bruits are heard. Treatment with an angiotensin-converting enzyme (ACE) inhibitor will most likely have which of the following acute effects on this patient's renal function? . A ) Decreased concentrating ability secondary to renal angioedema B) Decreased glomerular filtration rate secondary to dilation of efferent arterioles . 5 C) Decreased renal blood flow secondary to dilation of afferent arterioles ' ! D) increased concentrating ability secondary to a change in permeability of the collecting duct . E) Interstitial nephritis secondary to allergic drug reaction * m O m Previous Next Lab Values Calculator Review Help Pause
  • 13. Exam Section 1: Item 12 ol 50 National Board of Medical Examiners^ Comprehensive Basic Science Self-Assessment Time Remaining: 1 hr 12 min 57 sec 9 Mark 12 A 16-year-ofd boy is brought to the physician because of a 3-month history of shortness of breath while playing sports. He has no shortness of breath at rest. He says7 "Whenever I run around I cough, so i don't want to be on the basketball team anymore. " He takes no medications and has no known allergies. There is a family history of hypertension and asthma. He is 165 cm (5 ft 5 in) tall and weighs 68 kg (150 lb): BMI is 25 kg/m2. His respirations are 12/min, and blood pressure is 115/75 mm Hg. Cardiac examination shows no abnormalities except for a midsystolic click at the apex. The lungs are clear to auscultation of the chest. Which of the following best explains this patient's symptoms? A ) Deconditioning 3 B) Exercise-induced asthma o G) Malingering 3 0 ) Mitral valve prolapse J E } Thyroid disease O m O m Previous Next Lab Values Calculator Review Help Pause
  • 14. Exam Section 1: Item 13 of 50 National Board of Medical Examiners^ Comprehensive Basic Science Self-Assessment Time Remaining: 1 hr 12 min 53 sec 9 Mark 13. A 25-year-ofd man comes to the physician because of a 3-day history of pain and swelling of his right leg. He has no history of major medical illness or recent trauma. Examination of the right lower extremity shows edema and tenderness. Duplex ultrasonography of the right lower extremity shows a thrombus extending into the superficial femoral vein. Further studies show protein C deficiency. Inactivation of which of the foliowing coagulation factors is most likely as a result of this deficiency in this patient? A ) Factors V (proaccelebn) and VIII (antihemophilic factor) 3 B) Factors V (proaccelebn) and IX (plasma thromboplastin component) O C) Factors V (proaccelebn) and XI (plasma thromboplastin antecedent) 3 D ) Factors VIII (antihemophilic factor) and IX (plasma thromboplastin component) Q E) Factors VIII (antihemophilic factor) and XI (plasma thromboplastin antecedent) ( J F) Factors IX (plasma thromboplastin component) and XI (plasma thromboplastin antecedent) * m O m Previous Next Lab Values Calculator Review Help Pause
  • 15. Exam Section 1: Item 14 ol 50 National Board of Medical Examiners^ Comprehensive Basic Science Self-Assessment Time Remaining: 1 hr 12 min 50 sec ® Mark 14. Which of the following is most directly responsible for concentrating testosterone in the lumen of the seminiferous tubules? 3 A ) Androgen-binding protein ^ S) Follicle-stimulating hormone (FSH) O G) FSH/gonadotropin-releasing hormone o Q) Irthibm ' J E) Luteinizing hormone O m O m Previous Next Lab Values Calculator Review Help Pause
  • 16. Exam Section 1: Item 15 ol 50 National Board of Medical Examiners^ Comprehensive Basic Science Self-Assessment Time Remaining: 1 hr 12 min 47 sec 0Mar * 15. A 23-year-old woman has had the lesions shown In her mouth for 3 days. She has had frequent similar episodes over the past 15 years. The lesions are exacerbated by spicy, salty: and acidic food and drinks. They last approximately 1 week and resolve spontaneously. Visits to the dentist seem to trigger the development of the sores. Which of the following is the most likely diagnosis? A ) Aphthous ulcers 3 B) Candidiasis 0 C) Geographic tongue O 0 ) Koplik spots ^ E) Leukoplakia F) Lichen planus ' G) Psoriasis o 9 0 0 Previous Next Lab Values Calculator Review Help Pause
  • 17. Exam Section 1: Item 16 ol 50 National Board of Medical Examiners^ Comprehensive Basic Science Self-Assessment Time Remaining: 1 hr 12 min 41 sec ® Mark 16. A 20-year-old woman comes to the emergency department 30 minutes after slippmg on ice and extending her hand to break her fall. Palpation of the anatomic snuff-box produces pain. A wrist x-ray is most likely to show a fracture of which of the following carpal bones? . A ) Scaphoid 8) Lunate . ) C) Triquetrum D) Pisiform . . E) Trapezium F) Trapezoid . ) G ) Capitate H) Hamate O * m O m Previous Next Lab Values Calculator Review Help Pause
  • 18. Exam Section 1: Item 17 of 50 National Board of Medical Examiners^ Comprehensive Basic Science Self-Assessment Time Remaining: 1 hr 12 min 38 sec ® Mark 17. A 7G-year-ofd man undergoes laparotomy for resection of an abdominal aortic aneurysm. During the procedure, an incidental finding of acquired colonic diverticula is made. The diverticula in this patient are most likely present in which of the following? A ) Ascending coion i S) Cecum Q G) Descending colon D) Sigmoid colon ! J E) Transverse colon * m O m Previous Next Lab Values Calculator Review Help Pause
  • 19. Exam Section 1: Item 13 ol 50 National Board of Medical Examiners^ Comprehensive Basic Science Self-Assessment Time Remaining: 1 hr 12 min 19 sec 9 Mark 18. During a period of 36 hours, an 80-year-old woman has increasingly severe abdominal pain followed by fever chills, tachycardia: hypotension and. finally, shock. Blood cultures grow Escherichia coti. Her condition worsens and. despite supportive therapy and antibiotics; she dies 4 days after the onset of the illness. Which of the following is the most likely cause of the initial hypotension? . A ) Excessive production of nitric oxide B) Generation of hydrogen peroxide o C) Hemorrhage ! D ) Induction of endothelial adhesion molecules . E } Platelet aggregation * m O m Previous Next Lab Values Calculator Review Help Pause
  • 20. Exam Section 1: Item 19 of 50 National Board of Medical Examiners^ Comprehensive Basic Science Self-Assessment Time Remaining: 1 hr 12 min 16 sec ® Mark 19. A 65-year-old woman comes to the physician because of a 3-month history of headache, weakness of her arms, and left flank pain: she also has had a 14-kg (31-Eb) weight loss during this period. Physical examination shows weakness of the proximal upper and lower extremity muscles. There is augmentation of strength with repetitive testing of the deltoid muscles. An MRI of the brain shows a single well-demarcated mass surrounded by edema in the right frontal lobe. A stereotactic biopsy specimen of the lesion shows a malignant; smalt blue cell neoplasm that expresses cytokeratin, chromogranin, and synaptophysin. Which of the following is the most likely diagnosis? A ) Anaplastic ependymoma 3 B) ExtranodaE primary central nervous system lymphoma o C) Glioblastoma multiforme 3 0 ) Primary cerebral neuroblastoma E } Pulmonary small cell carcinoma metastatic to the brain ® 0 0 0 0 Previous Next Lab Values Calculator Review Help Pause
  • 21. Exam Section 1: Item 20 ol 50 National Board of Medical Examiners^ Comprehensive Basic Science Self-Assessment Time Remaining: 1 hr 12 min 12 sec ® Mark 20. Failure of normal differentiation of the endoderm in the embryonic lung bud is most likely to affect the development of which of the following? 3 A ) Capillary patterns 3 S) Cartilage in bronchi O Gj Smooth muscle on the bronchi J D) Surfaetant secretion 3 E) Tracheal rings o o o Previous Next Lab Values Calculator Review Help Pause
  • 22. Exam Section 1: Item 21 ol 50 National Board of Medical Examiners^ Comprehensive Basic Science Self-Assessment Time Remaining: 1 hr 12 min 9 sec ® Mark 21. A 67-year-old woman comes to the physician for a health maintenance examination. Her brother and mother have a history of colon cancer. The physician recommends colonoscopy: but the patient says that she would prefer only for her stool to be tested for biood. The physician explains that testing the stool for occult blood is not appropriate in this case. The physician is most likely concerned about which of the following regarding this test? . A ) Low sensitivity 8) Low specificity . J C) Potential for a false-positive result 3 D ) Uncertain negative predictive value . E) Uncertain positive predictive value * m O m Previous Next Lab Values Calculator Review Help Pause
  • 23. Exam Section 1: Item 22 ol 50 National Board of Medical Examiners^ Comprehensive Basic Science Self-Assessment Time Remaining: 1 hr 12 min 5 sec ® Mark 22 A 5-year-old boy is brought to the emergency department after ingesting 10 oz of a household cleaning solvent He is treated for acute hepatic and renal failure for 1 week and then discharged. During the next montfr regeneration of this boy's mature hepatocytes and renal tubular epithelial cells will be accomplished mostly by which of the following mechanisms? . A ) Activation of stem cells to enter G1 phase of the cell cycle 8) Decreased apoptosis at GrM transition of the cell cycle . J G) Recruitment of cells from G0 into the cell cycle ’ 3 D) Shortened time for progression of cells through the cell cycle . E) Terminal differentiation by celts exiting from the cell cycle O 0 I* 0 0 0 Previous Next Lab Values Calculator Review Help Pause
  • 24. Exam Section 1: Item 23 of 50 National Board of Medical Examiners^ Comprehensive Basic Science Self-Assessment Time Remaining: 1 hr 12 min 4 sec ® Mark 23. A 17-year-old boy is brought to the physician by his mother because she is concerned that his puberty is delayed. The mother states. "He is so short. His father is 6 feet 5 inches talk I don't understand why he has not had his growth spurt." When the mother leaves the room, the patient states: ' I'm fine. I don't know what's the matter with her. She wants me to be tall like my dad." The patient is 175 cm (5 ft 9 in) tali and weighs 70 kg (155 lb): BMI is 23 kg/m2 Sexual development is Tanner stage 4. in addition to reassuring the mother that her son is fine, which of the following is the most appropriate initial statement by the physician to the mother? A ) "Since your son is fine with his height you should try to accept him as he is. " 3 B) "Tell me more about your concerns about your son's height." o C) "Well do some blood tests just to be sure that all your son's hormone levels are okay." 3 O ) "Your son is average for his height and weight." J E) "Your son is not going to be any taller." * m O m Previous Next Lab Values Calculator Review Help Pause
  • 25. Exam Section 1: Item 24 ol 50 National Board of Medical Examiners^ Comprehensive Basic Science Self-Assessment Time Remaining: 1 hr 12 min 2 sec ® Mark 24. A 34-year-old woman is admitted to the hospital for treatment of pulmonary tuberculosis. Infliximab therapy was initiated 6 months ago for severe Crohn disease. This pharmacotherapy most likely inhibited which of the following immunologic functions in this patient? . A ) Activation of nuclear factor K8 to induce expression of interleukin-10 (IL-10) 8) Direct toxicity to the causal organism . J C) Maintenance of granulomas ' 3 D) Recruitment of segmented neutrophils to ingest and kill the bacteria (_y E) Stimulation of B lymphocytes to produce neutralizing antibodies against the causal organism * m O m Previous Next Lab Values Calculator Review Help Pause
  • 26. Exam Section 1: Item 25 ol 50 National Board of Medical Examiners^ Comprehensive Basic Science Self-Assessment Time Remaining: 1 hr 11 min 53 sec ® Mark 25. An 8-year-old boy continues to bleed excessively after tooth extraction. Prothrombin time, bleeding time, and platelet count are within the reference range. Partial thromboplastin time is prolonged but corrects after addition to the assay chamber of plasma from a patient with hemophilia A. Which of the following is the most likely diagnosis? . A ) Acute disseminated intravascular coagulation 8) Pactor V (proaccelerin) deficiency . J C) Factor VII (proconvertin) deficiency D) Hemophilia A . . E) Hemophilia B F) Immune thrombocytopenic purpura . J. G ) von Willebrand disease O * m O m Previous Next Lab Values Calculator Review Help Pause
  • 27. Exam Section 1: Item 26 ol 50 National Board of Medical Examiners^ Comprehensive Basic Science Self-Assessment Time Remaining: 1 hr 11 min 56 sec ® Mark 26. The diagram shows the major factors that determine blood pressure. Which of the following labeled factors is affected most by an aradrenergic antagonist? ^Centralnervous system m® Peripheral resistance farteridfat) Cardiac output x Blood pressure © Stroke volume ^Heort rale J "Contractility j ^Venous return Capacitance vessel tone (v&nutar) Blood volume © O A) O 8) O C) O D) OE) m O m Previous Next Lab Values Calculator Review Help Pause
  • 28. Exam Section 1: Item 27 of 50 National Board of Medical Examiners^ Comprehensive Basic Science Self-Assessment Time Remaining: 1 hr 11 min 54 sec ® Mark r f ..Jl ; < A / 1 / ^ | f 5 iVtt VI II 05 v i uVL V2 VJ R V? v * r * m nVF 4 — V~- I -T- —I V _ ^ H - II ^ . 4ir4 J ^ A vs 27. A previously healthy 21-year-old woman comes to the office because of a 2-month, history of shortness of breath and fatigue. Her most recent menstrual period was 3 months ago. Menses previously had occurred at regular 28-day intervals. She tells the physician that she thinks she may be pregnant. She takes no medications and has not seen a physician for several years. She appears healthy. She is 160 cm (5 ft 3 in) tall and weighs 54 kg (120 ib); BMI is 21 kg/m2 Vital signs are within normal limits. The lungs are dear. Cardiac examination shows a normal S(f a widely split S2 that does not change with respiration, and a grade 3/6 holosystoMc murmur that is loudest at the lower left sternal border and radiates to the upper left sternal border. EGG is shown. The most likely cause of these ffndings is dysfunction of which of the following structures? A ) Atrial septum 3 B) Ductus arteriosus Q C) Interventricular septum D ) Pulmonic valve E) Tricuspid valve * m O m Previous Next Lab Values Calculator Review Help Pause
  • 29. Exam Section 1: Item 28 ol 50 National Board of Medical Examiners^ Comprehensive Basic Science Self-Assessment Time Remaining: 1 hr 11 min 52 sec ® Mark ft *’v -i - H ‘-v : 8 .. T 7- - -v /. y ^ :mrnm r * aw k J % J VJ - b fi* * " . i . J -i » - WWHB ^dS?^ ' ^d> V I £ V-p B a % -rt -kr . r - - w. v- . i *_ . it J : >S- : ^ r’ v - ,.v . V ’ & 11 -J "H H _* IS n T * IT * “ S "V P I i -J. 1 P s i"3 1 r ii p * K i i a i l . i f s t •J~ r b ' - b f a J 9 " P J r* In J n X « ]31 P 9 + 28. A 53-year-old man has had progressive difficulty swallowing for the past 3 months. He has a 10-year history of heartburn with esophageal regurgitation of gastric contents. Tissue obtained on biopsy of the lower third of the esophagus is shown. Which of the following best describes the nature of this lesion? A } Basal zone hyperplasia of submucosal glands ! J B) Intestinal metaplasia of squamous epithelium O G) Malignant transformation of epithelium into squamous carcinoma ' 0 ) Squamous metaplasia of submucosal glands «1 0 r r r»; (IS Lab Values Calculator Review Previous Next Help Pause
  • 30. Exam Section 1: Item 29 of 50 National Board of Medical Examiners^ Comprehensive Basic Science Self-Assessment Time Remaining: 1 hr 11 min 45 sec ® Mark 29. A 50-year-old man comes to the emergency department because of a 2-week history of progressive shortness of breath. His pufse is 90/min, respirations are 26/min: and blood pressure is 120/S0 mm Hg. Physical examination shows no other abnormalities. Laboratory studies show: Arterial Pco2 Arterial Po2 Arterial 02content Mixed venous Po2 Mixed venous 02 content 30 mm Hg 96 mm Hg 12 vol% (N=17%-21%) 36 mm Hg 8 vol% (N=10%- 1 :6%) Which of the following is the most likely explanation for these findings? A ) Anemia B) Drug-induced alveolar hypoventilation C) Residence at a high altitude D ) Severe regional mismatching of alveolar ventilation and pulmonary capillary perfusion E) Voluntary hyperventilation * m O m Previous Next Lab Values Calculator Review Help Pause
  • 31. Exam Section 1: Item 30 ol 50 National Board of Medical Examiners^ Comprehensive Basic Science Self-Assessment Time Remaining: 1 hr 11 min 46 sec ® Mark 30. An investigator is studying the effects of triiodothyronine (T ^ ) and thyroxine (TJin hepatocytes in an experimental animal model. Which of the following best describes the action of these thyroid hormones on this target tissue? 3 A ) Both T3 and T 4 bind to the melanocortin 2 receptor on the cell surface J B) Both T3 and T 4 enter the nucleus ( G) T3 is converted to T4 in the cytosol J D) Thyroid hormone receptors preferentially bind T 4 over T3 O m O m Previous Next Lab Values Calculator Review Help Pause
  • 32. Exam Section 1: Item 31 ol 50 National Board of Medical Examiners^ Comprehensive Basic Science Self-Assessment Time Remaining: 1 hr 11 min 42 sec ® Mark 31. A female newborn delivered at 38 weeks' gestation to a 28-year-old woman; gravida 1, para 1 develops respiratory distress. Pregnancy and delivery were uncomplicated' amniotic fluid was clear, and the placenta was normal. Fetal ultrasonography and MRf at 34 weeks’ gestation showed a congenital diaphragmatic hernia: with evidence of small bowel and stomach herniation into left hemithorax. She is 52 cm (20.4 in) long and weighs 3500 g (7 lb 11 oz). Her temperature is 37.5°C (99.ST), pulse is 138/min. respirations are 50/min, and blood pressure is 70/55 mm Hg. Physical examination shows peripheral cyanosis that improves after administration of oxygen via endotracheal intubation. Breath sounds are decreased on the left. Cardiac examination shows normal heart sounds without murmurs. Which of the following complications of the described pathology is most likely to be life threatening to this newborn? A ) Active pneumonia . B) Alveolar edema 0 C) Amniotic embolism 0 0 ) Inadequate surfactant synthesis E) Pulmonary hypoplasia * m O m Previous Next Lab Values Calculator Review Help Pause
  • 33. Exam Section 1: Item 32 ol 50 National Board of Medical Examiners^ Comprehensive Basic Science Self-Assessment Time Remaining: 1 hr 11 min 35 sec ® Mark 32 An 18-hour-old male newborn Is 61 cm (24 in) long and weighs 5443 g (12 lb). His mother has type 1 diabetes mellitus. His serum glucose concentration is 20 rrig/dL Which of the following fetal conditions immediately prior to birth most likely precipitated the newborn's postnatal hypoglycemia? . A ) Decreased gluconeogenesis 8) Decreased glycogen concentration . J C) Decreased glycogen synthetase activity 3 D ) Decreased serum insuEin concentration . E } Increased serum insulin-like growth factor O * m O m Previous Next Lab Values Calculator Review Help Pause
  • 34. Exam Section 1: Item 33 of 50 National Board of Medical Examiners^ Comprehensive Basic Science Self-Assessment Time Remaining: 1 hr 11 min 31 sec ® Mark 33. The synthesis of the enzymes necessary for the replication of the genome occurs during which of the following phases of the cell cycle? O A) O B) O C) O D) O E) * m O m Previous Next Lab Values Calculator Review Help Pause
  • 35. Exam Section 1: Item 34 ol 50 National Board of Medical Examiners^ Comprehensive Basic Science Self-Assessment Time Remaining: 1 hr 11 min 29 sec ® Mark 34. A 22-year-old woman is brought to the emergency department in a semicomatose condition after collapsing near the end of running a marathon. Her prerace weight was 47 kg (103 lb). She now weighs 50 kg (110 lb). Her pulse is 115/min, respirations are 15/min, and blood pressure is 90/50 mm Hg. Physical examination shows cool, dry skin. She is responsive to painful stimuli. Laboratory studies show: Serum Na+ 116 mEq/L 4.8 mEq/L 89 mEq/L 22 mEq/L 22 mg/dL 101 mg/dL 1 mg/dL K- ci- HCO3 - Urea nitrogen Glucose Creatinine This patient's condition is most likely due to which of the following? o A ) Decreased ADH (vasopressin) 6 } Decreased aldosterone Q C) Excessive fluid intake ! _ D) Inadequate fluid intake O E) Increased aldosterone * m O m Previous Next Lab Values Calculator Review Help Pause
  • 36. Exam Section 1: Item 35 ol 50 National Board of Medical Examiners^ Comprehensive Basic Science Self-Assessment Time Remaining: 1 hr 11 min 26 sec ® Mark 35. A 36-year-old woman with type 2 diabetes mellitus comes to the physician for a follow-up examination. Current medications include a sulfonylurea. She is 173 cm (5 ft 3 in) tall and weighs 95 kg (210 lb): BMI is 32 kg/m2 Physical examination shows acanthosis nigricans. Treatment with metformin is most likely to produce which of the following effects in this patient? . A ) Decrease intestinal carbohydrate digestion B) increase beta-cell insulin secretion . J C) Increase deposition of adipocyte fat ' 3 D) increase hepatic triglyceride synthesis . E } Inhibit hepatic gluconeogenesis O * m O m Previous Next Lab Values Calculator Review Help Pause
  • 37. Exam Section 1: Item 36 ol 50 National Board of Medical Examiners^ Comprehensive Basic Science Self-Assessment Time Remaining: 1 hr 11 min 23 sec ® Mark 36. A 23-year-old man in the emergency department has apnea and pinpoint pupils. Needle tracks are present on his arms. Activation of which of the following opioid receptors in the central nervous system is most likely to be responsible for the apnea? O A) 5 O B) K O C) u O D) a O * m O m Previous Next Lab Values Calculator Review Help Pause
  • 38. Exam Section 1: Item 37 of 50 National Board of Medical Examiners^ Comprehensive Basic Science Self-Assessment Time Remaining: 1 hr 11 min 21 sec ® Mark 37. A 5-year-old girl with premature sexual development is diagnosed with precocious puberty. Pelvic examination shows a mass consistent with an ovarian tumor. Laboratory studies show decreased serum concentrations of gonadotropins and a marked increase in circulating estrogens. The ovarian tumor is most likely derived from which of the following cell types? . A ) Endothelial cells 8) Germinal epithelium . ) G) Granulosa cells 0 D) Stromal fibroblasts E) Thecal cells * m O m Previous Next Lab Values Calculator Review Help Pause
  • 39. Exam Section 1: Item 33 ol 50 National Board of Medical Examiners^ Comprehensive Basic Science Self-Assessment Time Remaining: 1 hr 11 min 19 sec ® Mark 38. A healthy 22-year-old woman undergoes testing to determine whether she is a suitable kidney donor for her 25-year-old brother with end-stage renal disease caused by type 1 diabetes meilitus. Immunologic studies show that she is human leukocyte antigen (HLAJ-DR3/DR6 positive; and her brother is HLA-DR3/DR4 positive. To determine the extent of alioreactivity: a mixed lymphocyte reaction is done using irradiated stimulator cells isolated from the donor and responder cells isolated from the recipient. The T lymphocytes that proliferate in these cultures will most likely react with which of the following HLA types? A) DR3 only O B) OR3 and DR4 o C) DR3 and DR6 o 0) 0R4 only E) DR4 and DR6 O F) 0R6 only O * I* m O m Previous Next Lab Values Calculator Review Help Pause
  • 40. Exam Section 1: Item 39 of 50 National Board of Medical Examiners^ Comprehensive Basic Science Self-Assessment Time Remaining: 1 hr 11 min 17 sec 9 Mark 39. A 67-year-old man with poorty controlled unstable angina is about to undergo coronary angiography with stent placement Prior to the procedure, treatment is initiated with aspirin and a drug that inhibits platelet interaction with fibrinogen. This drug is most likely which of the following? . A ) Abciximab 8) Celecoxib 0 C) Cilostazol _ D) Cloptdogrel . E } Dipyridamole * m O m Previous Next Lab Values Calculator Review Help Pause
  • 41. Exam Section 1: Item 40 ol 50 National Board of Medical Examiners^ Comprehensive Basic Science Self-Assessment Time Remaining: 1 hr 11 min 15 sec ® Mark 40. Lesch-Nyhan syndrome, an X-linked recessive disease; is seen in approximately 1/100.000 males. Which of the following is the expected prevalence of heterozygous females? O A) 1/1000 c B) 1/10,000 o C) 1/50,000 ' 0) 1/200,000 o E) 1/10,000,000 o m O m Previous Next Lab Values Calculator Review Help Pause
  • 42. Exam Section 1: Item 41 ol 50 National Board of Medical Examiners^ Comprehensive Basic Science Self-Assessment Time Remaining: 1 hr 11 min 12 sec ® Mark 41. The specimen shown is from a 65-year-old man. Which of the following is the most likely diagnosis? O A ) Acute leukemia B) Colonic carcinoma 0 C) Hepatic cell carcinoma D ) Leiomyosarcoma E) Liposarcoma o o o Previous Next Lab Values Calculator Review Help Pause
  • 43. Exam Section 1: Item 42 ol 50 National Board of Medical Examiners^ Comprehensive Basic Science Self-Assessment Time Remaining: 1 hr 11 min 10 sec ® Mark 42 A 45-year-old man who is able to bicycle 4 5 minutes a day has switched to a rowing machine. After 5 minutes on the machine, he experiences vertigo, lightheadedness. and fatigue of the left upper extremity. Within a few minutes of stopping the rowing exercise, all symptoms resolve.. Which of the following findings is most likely on physical examination? O A ) Diastolic murmur at the cardiac apex 8) increased jugular venous pressure . J C) Pansystolic murmur at the cardiac apex D) Right carotid bruit . E) Supraclavicular bruit O * m O m Previous Next Lab Values Calculator Review Help Pause
  • 44. Exam Section 1: Item 43 of 50 National Board of Medical Examiners^ Comprehensive Basic Science Self-Assessment Time Remaining: 1 hr 11 min 8 sec 9 Mark 43. A 55-year-old man comes to the physician because of a 2-month history of increasing difficulty swallowing and regurgitation of undigested food. He also has noticed unusual rumbling sounds in his voice that he feels originate in his neck. Physical examination shows halitosis. A videofluoroscopic swallowing study shows a 4-cm, posterior midline pouch protruding between the thyropharyngeus and cricopharyngeus portions of the inferior pharyngeal constrictor muscle. These muscles are most likely innervated by which of the following nerves? A ) Glossopharyngeal nerve 0 B) Hypoglossal nerve 0 G) Motor fibers from the vagus nerve 0 0 ) Parasympathetic fibers from the vagus nerve O & ) Sympathetic fibers from the superior cervical ganglion * m O m Previous Next Lab Values Calculator Review Help Pause
  • 45. Exam Section 1: Item 44 ol 50 National Board of Medical Examiners^ Comprehensive Basic Science Self-Assessment Time Remaining: 1 hr 11 min 6 sec 9 Mark 44. A 5-year-old girl is brought to the physician because of list;essness: fatigue, and dulf pain in the right upper quadrant of the abdomen. Her height and weight are below the 25th percentile. Laboratory findings indicate that the content of her p-globin chain is 15% to 20% of normal. Sequencing of the (3-globin gene shows a point mutation in a sequence 3' to the coding region in which AATAAA is converted to AACAAA. Consequently, the amount of mRNA for (3-globin is decreased to 10% of normal. Which of the following functions in rriRMA synthesis and processing is most fikefy encoded by the sequence AATAAA? A ) Capping with GTP 3 B) Cleavage and pofyadenylation o C) Silencing of the promoter 3 0 ) Splicing of the initial mRNA transcript in the nucleus J E } Transport of the mRNA out of the nucleus * m O m Previous Next Lab Values Calculator Review Help Pause
  • 46. Exam Section 1: Item 45 ol 50 National Board of Medical Examiners^ Comprehensive Basic Science Self-Assessment Time Remaining: 1 hr 11 min 3 sec 9 Mark 45. A 7G-year-old man comes to the physician for a follow-up examination. He has hypertension treated with a p-adrenergic antagonist He lives on a farm in central California and says he has always distilled his own liquor Before retiring f 0 years ago, he worked in a hat factory and subsequently in a textile factory. He has smoked 2 packs of cigarettes daily for the past 55 years. He tells the physician that he has had several episodes of painful swelling of his right great toe. Physical examination shows several lesions consistent with gouty tophi over the elbows bilaterally. Laboratory studies show: Hemoglobin A Serum Glucose Creatinine Uric acid 5.6% 1c 93 mg/dL 3.2 mg/dL 7.9 mg/dL The most likely cause of this patient's condition is which ofthe following? A ) Cigarette smoking ^ B) Drinking home-distilled liquor C) Farming in central California Q D ) Working in a hat factory E } Working in a textile factory o * 9 0 0 Previous Next Lab Values Calculator Review Help Pause
  • 47. Exam Section 1: Item 46 ol 50 National Board of Medical Examiners^ Comprehensive Basic Science Self-Assessment Time Remaining: 1 hr 11 min 1 sec 9 Mark 46. A 22-year-old woman comes to the physician because of a 6-month history of difficulty swallowing. She says that she feels like she is choking on both solids and liquids. She has no pain with swallowing. She has had a 4.5-kg f10-lb) weight loss during this time. There is no history of fever or chills. She is not sexualfy active. She does not smoke cigarettes or use illicit drugs. She is 170 cm (5 1 !7 in) tall and weighs 59 kg (130 lb); EM is 20 kg/m2 Her vital signs are within normal limits. Physical examination shows no abnormalities. An x-ray of the esophagus is shown. Which of the following is the most likely explanation for this patient's symptoms? O A ) Acid reflux into the lower esophagus Q B } Atrophy of the smooth muscle in the esophagus o C ) Inflammatory degeneration of esophageal waif neurons D ) Longitudinal mucosal tear at the esophagogastric junction E) Perforation of the esophageal wall «r * m O m Previous Next Lab Values Calculator Review Help Pause
  • 48. Exam Section 1: Item 47 ol 50 National Board of Medical Examiners^ Comprehensive Basic Science Self-Assessment Time Remaining: 1 hr 10 min 59 sec ® Mark 47. A 4S-year-old woman comes to the office because of a 4-month history of headaches, itchy skim difficulty swallowing, heartburn, chest tightness, pain in her arms and legs; and a burning sensation with urination. She has a history of simitar symptoms since the age of 14 years but previous examinations showed no abnormalities. Her vital signs are within normal limits. Physical examination and laboratory studies today show no abnormalities. Which of the following is the most likely diagnosis? A ) Conversion disorder 8) Factitious disorder . J C) Illness anxiety disorder (hypochondriasis) D) Malingering . E } Somatic symptom disorder O * m O m Previous Next Lab Values Calculator Review Help Pause
  • 49. Exam Section 1: Item 43 ol 50 National Board of Medical Examiners^ Comprehensive Basic Science Self-Assessment Time Remaining: 1 hr 10 min 56 sec ® Mark 48. Based on the graph of ju-aminohippurate (PAM) concentration versus PAH secretion, which of the following is lower at point Y than at point X? A ) Glomerular filtration rate B) PAH clearance Q C) PAH excretion rate ' D ) PAH filtered bad E) Renal blood flow Y 80- “O at c £ X Q OJ & <n l 40 — x < 0 i T T 0 20 40 60 80 100 Plasma p-aminohippurate (PAH) concentration mg/dL o o o Previous Next Lab Values Calculator Review Help Pause
  • 50. Exam Section 1: Item 49 of 50 National Board of Medical Examiners^ Comprehensive Basic Science Self-Assessment Time Remaining: 1 hr 10 min 52 sec ® Mark 49. A 41-year-old woman is evaluated because of increasingly severe headaches for 6 weeks. Her blood pressure is 160/100 mm Hg while standing and supine. A bruit is heard over the left costovertebral angle. Urinalysis shows no abnormalities. An angiogram of the left renal artery shows alternating areas of stenosis and aneurysmal dilatation ("string of beads" sign). Which of the following conditions of the renal artery is the most fikely diagnosis? . A ) Fibromuscular dysplasia 8) Hyaline arteriolosclerosis . J C) Intimal fibroplasia 3 0 ) Periarterial fibroplasia . E } Perimedial hyperplasia O * 9 0 0 Previous Next Lab Values Calculator Review Help Pause
  • 51. Exam Section 1: Item 50 ol 50 National Board of Medical Examiners^ Comprehensive Basic Science Self-Assessment Time Remaining: 1 hr 10 min 51 sec ® Mark 50. A cohort study assessing risk factors for acquisition of infection with a newly identified agent is performed. Only newly diagnosed subjects are eligible, and controls are selected on the basis of age. The results of this study are shown: Infection Present Infection Absent Turtle exposure No turtle exposure 60 20 40 80 Which of the following is the relative risk, for the exposure variable? O A) 0-7 O B) 1.0 O C) 1.7 O P) 2.2 O E) 3.1 * m O m Previous Next Lab Values Calculator Review Help Pause
  • 52. Exam Section 2: Item 1 of 50 National Board of Medical Examiners^ Comprehensive Basic Science Self-Assessment Time Remaining: 1 hr 14 min 55 sec 9 Mark 1. A 30-year-old man with HIV infection has been treated with a combination of antiretroviral drugs, including zidovudine (AZT); for 3 years. Laboratory studies show a marked increase in his plasma HIV viral load during the past 3 months. Viral resistance to zidovudine is suspected. A mutation in which of the following is most likely to explain the resistance to zidovudine in this patient? A ) Integrase 8) Neuraminidase . C) Protease ! D ) RNA-dependent DNA polymerase . E } Thymidine kinase 9 0 0 Next Lab Values Calculator Review Help Pause
  • 53. Exam Section 2: Item 2 of 50 National Board of Medical Examiners^ Comprehensive Basic Science Self-Assessment Time Remaining: 1 hr 14 min 51 sec 9 Mark 2. An investigator is studying [ ^adrenoreceptors in female experimental animals. During the experiment, epinephrine is injected intramuscularly into each animal, and the effects on p^ -adrenoreceptors are then observed. Which of the following physiologic effects is most likely to be observed in these animals? . A ) Increased myocardial contractility 8) internal urethral sphincter contraction O C) Lipolysis 3 D) Pilomotor contraction . . E) Pupillary dilation F) Uterine relaxation * m O m Previous Next Lab Values Calculator Review Help Pause
  • 54. Exam Section 2: Item 3 of 50 National Board of Medical Examiners^ Comprehensive Basic Science Self-Assessment Time Remaining: 1 hr 14 min 43 sec 9 Mark 3. A 16-year-old boy is brought to the emergency department because of a 2-day history of increasingly severe abdominal pain. His temperature is 39X (182:2X1pulse is 86,'min, respirations are 18/min, and blood pressure is 120/60 mm Hg. Abdominal examination shows exquisite tenderness of the right lower quadrant. His leukocyte count is 16: 000/mma. An appendectomy is done; the appendix is swollen with a tan exudate on the serosal surface. Which of the fofiowing best characterizes the leukocytosis in this patient? A ) Basophilia 0 B) Eosinophiiia 0 G) Lymphocytosis 0 0 ) Monocytosis J E } Neutrophilia * m O m Previous Next Lab Values Calculator Review Help Pause
  • 55. Exam Section 2: Item 4 of 50 National Board of Medical Examiners^ Comprehensive Basic Science Self-Assessment Time Remaining: 1 hr 14 min 46 sec ® Mark 4. A 32-year-old primigravid woman delivers a heafthy 3402-g (7-lb 8-oz) male newborn after an uncomplicated cesarean delivery because of a nonreassuring fetal stress test. Two days prior to discharge from the hospital she has persistent numbness of the area surrounding the abdominal incision. The physician assures the patient that sensation will gradually return as the nerves regenerate. Which of the following best describes the rate-limiting step in this patient's return to normal sensation? . A ) Dorsal root ganglion cel! proliferation 8) Fast anterograde axonal transport 0 C) Fibroblast proliferation 0 D ) Retrograde axonal transport . E) Slow anterograde axonal transport * m O m Previous Next Lab Values Calculator Review Help Pause
  • 56. Exam Section 2: Item 5 of 50 National Board of Medical Examiners^ Comprehensive Basic Science Self-Assessment Time Remaining: 1 hr 14 min 44 sec ® Mark 5. An 18-year-old woman comes to the physician because of progressive fever general malaise, and blood in her urine since she began oral antibiotic therapy for a urinary tract infection 5 days ago. She also has a 3-day history of a rash.. Her temperature is 38°C (1DG. 43F), pulse is 75/min, respirations are 12/mrn, and blood pressure is 125/80 mm Hg. Physical examination shows a petechial rash over the chest. back: and upper and lower extremities. Urinalysis shows: Blood Protein Leukocytes Eosinophils 3+ 1+ 150/hpf 30% Which of the following is the most likely diagnosis? o A ) Acute tubular necrosis 6 } Glomerulonephritis Q C) IgA nephropathy D) Interstitial nephritis O E) Papillary necrosis * m O m Previous Next Lab Values Calculator Review Help Pause
  • 57. Exam Section 2: Item 6 of 50 National Board of Medical Examiners^ Comprehensive Basic Science Self-Assessment Time Remaining: 1 hr 14 min 33 sec 9 Mark 6. A 33-year-old woman has had weakness of the right lower two thirds of the face for the past 2 months. Which of the following labeled regions in the normal brain shown is the most likely site of the lesion causing this symptom? Central sulcus E F C / G B r r £$ I > J (h : kM A- 5 r cd- -i I .-- Ll m -wr r m - — . -J 3« » r. j i H . ^ . ip O A) O B) O C) O D) OE) OF) O G) O H) O n O J) 0 0 # 0 0 Previous Next Lab Values Calculator Review Help Pause
  • 58. Exam Section 2: Item 7 of 50 National Board of Medical Examiners^ Comprehensive Basic Science Self-Assessment Time Remaining: 1 hr 14 min 36 sec ® Mark 7. A 4G-year-old woman comes to the physician because of temperatures of 38 to 39X (100.4 to 102.2aF) and malaise for the past 2 days. She has chronic alcohol dependence. There are several spider angiomas on the face, chest and back; she is not jaundiced. The abdomen is protuberant and nontender. There is shifting dullness to percussion. A firm liver edge is ballotable 3 cm below the right costal margin. Routine laboratory studies show miid anemia; mildly increased hepatic transaminase activities: and a decreased serum albumin concentration. Peritoneal aspiration yields serous fluid with 1100 leukocytes/mm3 {80 neutrophils) and 100 erythrocytes/mm3 Which of the following processes best accounts for the patient's febrife ilEness? A ) Acute cholecystitis 3 B) Chronic pancreatitis o C) Exacerbation of autoimmune hepatitis 3 0 ) Pelvic infEammatory disease E } Spontaneous bacterial peritonitis O * 9 0 0 Previous Next Lab Values Calculator Review Help Pause
  • 59. Exam Section 2: Item 8 of 50 National Board of Medical Examiners^ Comprehensive Basic Science Self-Assessment Time Remaining: 1 hr 14 min 34 sec 9 Mark 8. A patient with a 6-month history of heat intolerance. fatigue; episodes of tachycardia: and weight loss has a diffusely enlarged thyroid gland Serum concentrations of triiodothyronine (T3) and thyroxine (TJ are increased; thyroid-stimulating hormone is decreased. Which of the following is the most likely diagnosis? O A ) Autoimmune thyroid hyperplasia 8) Pituitary neoplasm O G) Surreptitious ingestion of T4 D) Thyroid neoplasm O * m O m Previous Next Lab Values Calculator Review Help Pause
  • 60. Exam Section 2: Item 9 of 50 National Board of Medical Examiners^ Comprehensive Basic Science Self-Assessment Time Remaining: 1 hr 14 min 31 sec 9 Mark 9. A 19-year-old man who is a college student is brought to the emergency department because of the sudden onset of right-sided chest pain and difficulty breathing after an accident in which he was thrown from his bicycle. He has difficulty walking and cannot climb stairs because of pain and shortness of breath. He is slightly cyanotic, afebrile: and tachypneic. Which of the following is most suggestive that fractured ribs caused the respiratory problem? A ) Bronchophony B) Expiratory stridor 0 C) Inspiratory stridor 0 D) Subcutaneous crepitus . E) Succussion splash * m O m Previous Next Lab Values Calculator Review Help Pause
  • 61. Exam Section 2: Item 10 of 50 National Board of Medical Examiners^ Comprehensive Basic Science Self-Assessment Time Remaining: 1 hr 14 min 29 sec ® Mark 10. A 62-year-ofd woman develops difficulty breathing. Pulmonary function tests before and after branchediiator therapy show no changes. Predicted and patient values are: Predicted Test Patient FVC (L) FEV1 (L) FEV/FVC Total Eung capacity (L) Residual volume (L) 5.0 4.0 4.0 2.4 0.8 0.6 6.0 7.2 1.6 2.7 Which of the following is the most likely explanation for these findings? Airway Resistance Lung Compliance O A) O B) O C) O 0) OE) T T normal T Normal 1 I T 4 * m O m Previous Next Lab Values Calculator Review Help Pause
  • 62. Exam Section 2: Item 11 of 50 National Board of Medical Examiners^ Comprehensive Basic Science Self-Assessment Time Remaining: 1 hr 14 min 26 sec ® Mark 11. A 3-year-old girl is brought to the physician because of a 2-week history of diarrhea. Her temperature is 37.6CC (99.8aF}: pulse is 70/min, respirations are Wmjn; and blood pressure is 110/70 mm Hg. Physical examination shows generalized [ymphadenopathy. A CT scan of the chest and abdomen shows enlarged lymph nodes in the mesentery and para-aortic region. Examination of a lymph node biopsy specimen shows marked proliferation of histiocytes and numerous segmented neutrophils. Granulomata are absent and special stains show numerous acid-fast bacilli. which are subsequently identified as Mycobacterium avium-intracellulare. Serum studies show normal concentrations of IgA, lgG; IgM, B lymphocytes, T lymphocytes, and CD4? and CD8+- T lymphocytes. This patient most likely has defective function or expression of which of the following proteins? A ) Class I MHG molecules . 8) Interferon-gamma receptor 0 G) interleukin-2 (IL-2) receptor ) 0) Leukocyte function-associated antigen-1 ' E) MADPH oxidase * m O m Previous Next Lab Values Calculator Review Help Pause
  • 63. Exam Section 2: Item 12 of 50 National Board of Medical Examiners^ Comprehensive Basic Science Self-Assessment Time Remaining: 1 hr 14 min 24 sec ® Mark 12 Drug X is used to treat pain associated with rheumatoid arthritis. The drug is a weak acid, with a pKa of 4.4; it is absorbed principally through the stomach. It is determined that Drug X is absorbed efficiently by the body because of the ionization conditions under which it exists at gastric and blood pH. Which of the following sets of physical chemical states of the drug is most fikely? At Gastric pH Ionized Ionized Nonionized At Blood pH ionized O A) O B) O C) O D) nonionized ioniized Nonionized nonionized * m O m Previous Next Lab Values Calculator Review Help Pause
  • 64. Exam Section 2: Item 13 of 50 National Board of Medical Examiners^ Comprehensive Basic Science Self-Assessment Time Remaining: 1 hr 14 min 21 sec ® Mark 13. A previously healthy 10-year-old girl is brought to the physician by her mother because of a 6-week history of headache, nausea, and difficulty walking. An MR] of the brain shows a mass iin the posterior fossa that is found to be an astrocytoma, i his tumor developed from cells that normally serve which of the following functions in the brain? . A ) Formation of myelin sheaths in the central nervous system 8) Phagocytosis of recycled synaptic terminal membrane . C) Production and secretion of cerebrospinal fluid Q D) Termination of action potentials . E) Transport of hormones from the cerebral spinal fluid to capillaries F) Uptake of amino acid neuretransmitters * m O m Previous Next Lab Values Calculator Review Help Pause
  • 65. Exam Section 2: Item 14 of 50 National Board of Medical Examiners^ Comprehensive Basic Science Self-Assessment Time Remaining: 1 hr 14 min 19 sec 9 Mark 14. A 26-year-old woman comes to the physician because of a 1-week history of rectal pain that is made more severe by defecation, and occasional blood on the toilet tissue after a bowel movement. She says that her stools appear normal and that she has not had any trauma. She has a history of chronic constipation. Visual rectal examination shows the findings in the photograph. Which of the following is the most likely diagnosis? A ) Anal fissure 3 B) Bowen carcinoma C) Condyloma acuminatum O D) Perianal abscess E) Prolapsed internai hemorrhoid O * & * C Previous Next Lab Values Calculator Review Help Pause
  • 66. Exam Section 2: Item 15 of 50 National Board of Medical Examiners^ Comprehensive Basic Science Self-Assessment Time Remaining: 1 hr 14 min 16 sec 9 Mark 15. A moderately obese 55-year-old man is brought to the emergency department because of a 10-hour history of severe chest pain. He has a 5-year history of exercise-induced angina. His pulse is 109/min, respirations are 15/min, and blood pressure is 132/92 mm Hg. Physical examination shows diaphoresis. A blood sample obtained 2 hours after admission shows increased serum activity of creatine kinase ME. Which of the following is the most likely cause of this aboratory finding? . A ) Increased GoEgi complex activity 8) increased permeability of the plasma membrane O G) Mitochondria!swelling D) Muclear lysis . E) Proliferation of the endoplasmic reticulum O * 9 0 0 Previous Next Lab Values Calculator Review Help Pause
  • 67. Exam Section 2: Item 16 of 50 National Board of Medical Examiners^ Comprehensive Basic Science Self-Assessment Time Remaining: 1 hr 14 min 12 sec 9 Mark 16. AGG-year-ofd woman comes to the physician because of a 6-month history of pain in her hips and knees. Physical examination shows findings consistent with osteoarthritis; and the physician recommends iibuprofen. The patient refuses and asks about taking glucosamine. Which of the following responses by the physician is most appropriate? . A ) "Glucosamine hasn't been studied well enough for me to recommend it." 8) "Glucosamine's side effects aren't listed. It may be more dangerous than we realize." . G) "Ibuprofen has been proven effective for your condition." ' ! D) "What have you heard about using glucosamine to treat arthritis?" . E) "Why did you come to me if you don’t want to take what I recommend?11 F) "You should really see a naturopathic doctor." * m O m Previous Next Lab Values Calculator Review Help Pause
  • 68. Exam Section 2: Item 17 of 50 National Board of Medical Examiners^ Comprehensive Basic Science Self-Assessment Time Remaining: 1 hr 14 min 10 sec 9 Mark 17. An 88-year-old man who lives alone is brought to the physician by his daughter because she is concerned that he has not been eating a well-balanced diet for 9 months. He is 170 cm (5 ft 7 in) tall and weighs 50 kg (110 fb); BMI is 17 kg/m2 Physical examination shows multiple ecchymoses on the upper and lower extremities. Laboratory studies show: Platelet count Prothrombin time Serum Vitamin B6 (pyridoxine) Vitamin C (ascorbic acid) Folic acid 160,000/mm3 12 sec (INR=1} 9 ng/dL (N=5—30) 0.1 mg/dL (N=0.4-2) 5 ng/dL (N=2—20) The ecchymoses in this patient are most likely due to a disorder of which of the following? A ) Arachidonic add production Q B) Binding of carboxyglutamic acid to phospholipid C C) Carboxylation of factor fl (prothrombin) J D ) Proline hydroxylafion E } Transfer of methyl groups to organic acids * m O m Previous Next Lab Values Calculator Review Help Pause
  • 69. Exam Section 2: Item 13 of 50 National Board of Medical Examiners^ Comprehensive Basic Science Self-Assessment Time Remaining: 1 hr 14 min 3 sec ® Mark 18. A 14-year-ofd boy has persistent leukocytosis and neutrophilia without evidence of a current infection. He has a history of recurrent infections of the skin: upper and lower airways, and perirectal area. Gram-negative and gram-positive rods have been isolated. The number and function of B and T lymphocytes are normal. Production of hypochlorous acid by neutrophils and the nitroblue tetrazolium reduction test are normal;. Neutrophil chemotactic response to the formyl-MetLeuPhe (fMLP) peptide is diminished. Which of the following disorders of neutrophils is the most likely diagnosis? A ) Chronic granulomatous disease 3 B) Cyclic neutropenia o C) Leukocyte adhesion deficiency 3 0 ) Myeloperoxidase deficiency E } Neutrophil-specific granule deficiency O * m O m Previous Next Lab Values Calculator Review Help Pause
  • 70. Exam Section 2: Item 19 of 50 National Board of Medical Examiners^ Comprehensive Basic Science Self-Assessment Time Remaining: 1 hr 14 min 5 sec ® Mark 0-r0 1 x 2 I > 4 ^0 2 ^ 3 6 1 1 n 2 £ 3 3 3 1 2 1 1 1 1 o 4 HI 2 2 3 3 5 2 6 3 1 2 3 1 1 1 IV 2 5 3 6 2 3 2 3 1 4 Affected male # Affected female Unaffected male O Unaffected female 0 Affected male, deceased 0 Unaffected female, deceased 19. An investigator is studying a large family with many members who are affected by a disorder caused by a fully penetrant autosomal dominant inherited gene mutation. A pedigree is shown. Most affected members also have a rare allele at a focus thought to be closely linked to the disease focus. A father (individual 111-3) and his daughter (individual iV-3) have the disorder, but they have the wild-type allele at the linked locus. Which of the following is the most likely cause of these findings? ( J A ) Insertion of a LINE sequence 8 } Random segregation Q G) Recombination D) Single nucleotide polymorphism E) Transduction ® 0 0 0 0 Previous Next Lab Values Calculator Review Help Pause
  • 71. Exam Section 2: Item 20 ol 50 National Board of Medical Examiners^ Comprehensive Basic Science Self-Assessment Time Remaining: 1 hr 14 min 3 sec ® Mark 20. A 45-year-old woman comes to the physician because of a 6-month history of shortness of breath with exeition and a nonproductive cough. She sometimes has difficulty swallowing and often has heartburn, especially if she fies down after a meal. She adds that her fingers have swollen, and she has had to get her wedding ring resized. Her fingers also become white and painful in cold weather or cotd water. Her pulse is 75/min: respirations are 20/min, and blood pressure is 150/100 mm Hg. Physical examination shows tight, smooth facial skin without wrinkles. Additional testing is most likely to show which of the following sets of cardiovascular changes in this patient? Left Ventricular Diastolic Compliance Mean Pulmonary Artery Pressure Coronary Vascular Resistance O A) O B) O C) O D) O E) O F) O G) O H) f f f T T I T * V 4 4 t t v ' l i A. 1 4 V V © 15 fE Lab Values Calculator Review Previous Next Help Pause
  • 72. Exam Section 2: Item 21 ol 50 National Board of Medical Examiners^ Comprehensive Basic Science Self-Assessment Time Remaining: 1 hr 13 min 45 sec 9 Mark 21. A 25-year-old woman comes to the physician for a routine health maintenance examination. She is currently preparing for an 8-km (5-mile) race, and she sprint trains twice weekly. During this training: she runs 200-meter sprints in two groups of 10 sprints separated by a 30-second rest between each sprint in a group: and a 2-minute rest between each group of 10. After the workout her legs feel weak, and her muscles burn and sometimes cramp. Which of the following best explains her symptoms? A ) Decreased activity of the sodium-proton antiporter, resulting in an acidic sarcoplasm 8) increased activity of the sodium-proton antiporter resulting in an acidic sarcoplasm O C) increased oxygen delivery to the muscle: leading to increased metabolism and acid production ' D) Regeneration of NAD+ from NADH; which produces acid * m O m Previous Next Lab Values Calculator Review Help Pause
  • 73. Exam Section 2: Item 22 ol 50 National Board of Medical Examiners^ Comprehensive Basic Science Self-Assessment Time Remaining: 1 hr 13 min 42 sec ® Mark 22 A 14-year-old girl is brought to the physician because of a 1-month history of migraine-like headaches, vomifing: and multiple left-sided focal seizures. She has had hearing foss since the age of 11 years. Her mother and maternal grandmother have high-tone deafness. Physical examination shows Joss of vision in one half of the visual field of the right eye and weakness of the right upper and tower extremities. Serum and cerebrospinal fluid concentrations of lactic acid are increased. This patient most likely has a mutation of which of the fallowing? O A ) Endoplasmic reticulum glycosyItransferase 3 B) Lysosomal a-gfucosidase o C) Mitochondrial tRNALeu 3 0 ) Nuclear proteasome activator J E } Peroxisomal catalase O m O m Previous Next Lab Values Calculator Review Help Pause
  • 74. Exam Section 2: Item 23 of 50 National Board of Medical Examiners^ Comprehensive Basic Science Self-Assessment Time Remaining: 1 hr 13 min 40 sec 9 Mark 23. A case-control study is conducted to determine if obesity is a risk factor for gastroesophageal reflux disease (GERD). A questionnaire is used to select subjects with severe symptoms of GERD and subjects with no symptoms. A BMf is calculated for each subject. The results (in kg/m2) are shown: BMI<25 25<BMK30 30<BMk35 BMI>35 Subjects with GERD symptoms Subjects with no symptoms 303 900 200 50 700 900 200 30 Which of the following represents the odds ratio for GERD symptoms in subjects with BMfs greater than 35 compared with subjects with BMIs less than 25? A) (50 *30) / (300 *700) o B) (50 *700) I (30x300) o cj [50/(30+50)31[300/(300+700)] : D) [50/(50+300)31[30/(30+700)3 . ^ E) [50/(50+200+900+300)3 '[30/(30+200+900+700)3 * m O m Previous Next Lab Values Calculator Review Help Pause
  • 75. Exam Section 2: Item 24 ol 50 National Board of Medical Examiners^ Comprehensive Basic Science Self-Assessment Time Remaining: 1 hr 13 min 33 sec 0Mar * 24. A 14-year-old boy is brought to the physician by his mother because of daily headaches for 2 months. The headaches are described as a bilateral aching in the tempfes. His mother states that he also Tias not been himself' for the past few months. He seems more confused, often forgetting names, dates, and places: and he is clumsy with frequent falls. His school performance also has declined over the past quarter. Physical examination shows a broad-based; ataxic gait. He is alert and oriented to person: place, and time: but he is slow to answer questions. Chronic abuse of which of the following substances is the most likely cause of this patient's condition? A ) Cocaine C B) Ethanol 0 G) Inhaled gfue 0 0 ) Methamphetamines 0 E } PCP (phencyclidine) * m O m Previous Next Lab Values Calculator Review Help Pause
  • 76. Exam Section 2: Item 25 ol 50 National Board of Medical Examiners^ Comprehensive Basic Science Self-Assessment Time Remaining: 1 hr 13 min 36 sec 9 Mark 25. A study is designed to measure the impact of exercise on the incidence of myocardial infarction. Subjects are enrolled in the study and divided into two groups based on their self-reported exercise habits. At the end of the study; subjects who reported exercising have half the incidence of myocardial infarction compared with the subjects who did not exercise. Which of the following best describes this study design? A ) Case-control 8) Case series . C) Cohort o D) Cross-sectional . E } Randomized clinical trial * m O m Previous Next Lab Values Calculator Review Help Pause
  • 77. Exam Section 2: Item 26 ol 50 National Board of Medical Examiners^ Comprehensive Basic Science Self-Assessment Time Remaining: 1 hr 13 min 33 sec 9 Mark 26. An 18-year-old woman is being evaluated for amenorrhea. She has never had a menstrual period. She is 183 cm (6 ft) tall. Breast development and external genitalia are normal. There is no axillary or pubic hair. Which of the following karyotypes is most likely? O A) 45.X O 0) 46,xx O C) 46,XY O D) 46.X,i(Xq) O E) 47,XXX * m O m Previous Next Lab Values Calculator Review Help Pause
  • 78. Exam Section 2: Item 27 of 50 National Board of Medical Examiners^ Comprehensive Basic Science Self-Assessment Time Remaining: 1 hr 13 min 31 sec ® Mark 27. A 50-year-old man comes to the physician because of a 2-month history of pain of his wrists; changes in skin color, and progressive fatigue. His brother has type 2 diabetes mefliitus and cirrhosis. Physical examination shows bronze-colored skin, tenderness of the metacarpophalangeal joints in both hands, and hepatosplenomegaly. Serum studies show: AST AIT Ferritin 100 U/L 110 U/L 1200 ng/mL Total iron-binding capacity 200 pg/dL (N=25G-400) Transferrin saturation 80% (N=20—50} Analysis of a liver biopsy specimen shows a markedly increased iron concentration and cirrhosis. Which of the following is the most likely cause of the fmdings in this patient? A ) Increased erythropoietin action Q B) Increased intestinal iron absorption Q C) Increased oral iron intake ! D ) Decreased erythropoiesis ) E) Decreased iron excretion F) Decreased serum transferrin concentration m O m Previous Next Lab Values Calculator Review Help Pause
  • 79. Exam Section 2: Item 28 ol 50 National Board of Medical Examiners^ Comprehensive Basic Science Self-Assessment Time Remaining: 1 hr 13 min 29 sec ® Mark 28. The presence of argininosuccinate in the urine indicates a defect in the conversion of which of the following? 0 A ) Ammonia to urea J S) Lysine to glutaryl CoA 0 G) Methionine to succinyl CoA J D) Phenylalanine to fumarate ! J E) Tryptophan to indole o o o Previous Next Lab Values Calculator Review Help Pause
  • 80. Exam Section 2: Item 29 of 50 National Board of Medical Examiners^ Comprehensive Basic Science Self-Assessment Time Remaining: 1 hr 13 min 26 sec ® Mark 29. A 55-year-old woman is brought to the emergency department after being injured in a motor vehicle collision. She has an injury of the soft tissue of the face that prevents her from drinking from a glass without spilling the contents. Several days later she stilt has this problem. The photograph shows her attempting to purse her lips to whistle. Which of the following nerves is most likely damaged? A ) Buccal branch of the facial nerve B) Inferior alveolar branch of the mandibular division of the trigeminal nerve O Infraorbital branch of the maxillary division of the trigeminal nerve D ) Mandibular branch of the facial nerve E) Pharyngeal branches of the vagus nerve * m O m Previous Next Lab Values Calculator Review Help Pause
  • 81. Exam Section 2: Item 30 ol 50 National Board of Medical Examiners^ Comprehensive Basic Science Self-Assessment Time Remaining: 1 hr 13 min 24 sec ® Mark 30. A previously healthy 6-month-ofd boy is brought to the physician because of a cough for 1 week. Initially he had a low-grade fever sneezing, congestion, and runny nose. He then developed a dry intermittent cough. The parents now note that with any startle the baby chokes and gasps. He has not had any immunizations. Physical examination shows paroxysms of '‘machine gun" -like coughing with a forced expiratory grunt at the end of coughing. Leukocyte count is 30; 000/mm3 (70% lymphocytes). Neutrophil chemotaxis and oxidative metabolism are defective due to increased activity of which of the following enzymes? A ) Adenyly! cyclase 3 B) Myeloperoxidase o C) NADPH oxidase O 0 ) Phospholipase C J E } Protein kinase C * m O m Previous Next Lab Values Calculator Review Help Pause
  • 82. Exam Section 2: Item 31 of 50 National Board of Medical Examiners^ Comprehensive Basic Science Self-Assessment Time Remaining: 1 hr 13 min 22 sec ® Mark 31. A 22-year-old woman comes to the office because of a 3-day history of nonproductive cough. She also has a 1-week history of fatigue: progressive shortness of breath with exertion and white lying down and swelling of her legs and feet. She delivered a male newborn via uncomplicated vaginal delivery 1 month ago. She has no history of major medical illness and takes no medications. Her temperature is 37.7aC (99.8aF): pulse is 104/min, respirations are 20/miinP and blood pressure is 126/80 mm Hg. Bilateral basilar crackles are heard. There is 1+ edema of the lower extremities bilaterally. Which of the following is the most likely diagnosis? A ) Amniotic fluid embolism 3 B) Cardiomyopathy o C) Major depressive disorder 3 0 ) Pneumonia J E } Pulmonary embolism ' F) Pulmonary fibrosis o * 9 0 0 Previous Next Lab Values Calculator Review Help Pause
  • 83. Exam Section 2: Item 32 ol 50 National Board of Medical Examiners^ Comprehensive Basic Science Self-Assessment Time Remaining: 1 hr 13 min 20 sec ® Mark 32 A 63-year-old man is scheduled to undergo coronary artery bypass grafting with a portion of the great saphenous vein. An incision to remove a portion of the vein should begin in which of the following locations? 3 A ) Along the lateral surface of the leg J S) Along the medial side of the ankle joint (3 G) Along the plantar surface of the foot 0 } Anterior to the knee joint ' J E) Posterior to the hip joint * m O m Previous Next Lab Values Calculator Review Help Pause
  • 84. Exam Section 2: Item 33 of 50 National Board of Medical Examiners^ Comprehensive Basic Science Self-Assessment Time Remaining: 1 hr 13 min 18 sec 9 Mark 33. A 62-year-old man is brought to the emergency department 2 hours after the sudden onset of pain and coolness of his right leg. He is otherwise healthy except for mild hyperthyroidism treated with propylthiouracil. Examination of the tower extremities shows normal skim naiis: and hair growth patterns. Pulses are absent in the right lower extremity and normal on the left. Which of the following is the most likely diagnosis? A ) Cellulitis 8) Deep venous thrombosis O C) Embolic arterial occlusion 3 D ) Lumbar cisc herniation . E } Rhabdomyolysis * m O m Previous Next Lab Values Calculator Review Help Pause
  • 85. Exam Section 2: Item 34 ol 50 National Board of Medical Examiners^ Comprehensive Basic Science Self-Assessment Time Remaining: 1 hr 13 min 16 sec 9 Mark 34. A 32-year-old woman comes to the physician for a follow-up examination after atypical cells were noted on a recent Pap smear. Physical examination shows a 1 x 1-cm area of leukoplakia on the cervix A biopsy specimen of the lesion shows invasive squamous cell carcinoma. Malignant cells from this site will most likely drain first to which of the following lymph nodes in this patient? . A ) Femoral 8) inferior mesenteric O C) Internal iliac D) Lumbar . E) Superficial inguinal * m O m Previous Next Lab Values Calculator Review Help Pause
  • 86. Exam Section 2: Item 35 ol 50 National Board of Medical Examiners^ Comprehensive Basic Science Self-Assessment Time Remaining: 1 hr 13 min 14 sec 9 Mark 35. A 70-year-old man dies in a motor vehicle collision. He had been undergoing evaluation for occult blood in the stool A photograph of a section of the transverse colon obtained at autopsy is shown. Which of the following is the most likely diagnosis? .. A ) Hyperplastic polyp Q B) Inflammatory pseudopolyp C) Juvenile polyp O D ) Peutz-Jeghers syndrome E) Tubular adenoma & * f* Previous Next Lab Values Calculator Review Help Pause
  • 87. Exam Section 2: Item 36 ol 50 National Board of Medical Examiners^ Comprehensive Basic Science Self-Assessment Time Remaining: 1 hr 13 min 12 sec 9 Mark 36. A 61-year-old man has erectile dysfunction due to spinal cord injury at L-2. Sildenafil is likely to markedly correct the dysfunction by acting at which of the following labeled structures in the transverse section of the penis? O A) O B) O C) O Pi 0 0 © 0 0 ns Lab Values Calculator Review Previous Next Help Pause
  • 88. Exam Section 2: Item 37 of 50 National Board of Medical Examiners^ Comprehensive Basic Science Self-Assessment Time Remaining: 1 hr 13 min 10 sec 9 Mark 40-i 37. Which of the following is the mean number of episodes of urinary tract infections for children (n=100} in the sample shown in the graph? O A ) 1 c B) 1.55 o C) 2.07 D ) Cannot be determined from this graph g 30- I J iBUBn o 8 2 °- _Q I 10- « 3 0 1 2 Number of urinary tract infections m O m Previous Next Lab Values Calculator Review Help Pause
  • 89. Exam Section 2: Item 33 ol 50 National Board of Medical Examiners^ Comprehensive Basic Science Self-Assessment Time Remaining: 1 hr 13 min 3 sec ® Mark 38. A 3-year-old boy is brought to the physician because of a 1-month history of pale skin. His parents are of European descent. He has no personal or family history of major medical illness. Physical examination shows pallor. Laboratory studies show: Hemoglobin Hematocrit Mean corpuscular hemoglobin concentration Mean corpuscular volume 3 g.'dL (N=11-15) 24% (N=28-45) 34% Hb/cell (IM=31-36) 90 pm3 (N=77-98) A photomicrograph of a peripheral blood smear is shown. Genetic testing is most likely to show which of the following findings in this patient? A ) Heterozygous mutation in the ankyrin gene B) Heterozygous mutation in the a-globin gene O - P) Heterozygous mutation in the p-giobin gene D ) Homozygous mutation in the ankyrin gene O E) Homozygous mutation in the a-globin gene F) Homozygous mutation in the P-globin gene * m O m Previous Next Lab Values Calculator Review Help Pause
  • 90. Exam Section 2: Item 39 of 50 National Board of Medical Examiners^ Comprehensive Basic Science Self-Assessment Time Remaining: 1 hr 13 min 5 sec ® Mark 39. A 24-year-old woman is brought to the emergency department 15 minutes after the sudden onset of shortness of breath following a bee sting. Her pulse is 130/min, and blood pressure is 70/30 mm Hg. Three hours later, her blood pressure returns to normal following administration of intravenous fluids, corticosteroids. antihistamines: and epinephrine. The next day. she has minimal urine output. Which of the following areas of the kidney is likely to be most affected with this patient's prolonged hypotension? O A ) Glomerular epithelial cells 8) Loop of Henle 0 G) Medullary interstifium D) Mesangial cells . E) Proximal tubules * m O m Previous Next Lab Values Calculator Review Help Pause
  • 91. Exam Section 2: Item 40 ol 50 National Board of Medical Examiners^ Comprehensive Basic Science Self-Assessment Time Remaining: 1 hr 13 min 3 sec ® Mark 40. A 65-year-old man develops urinary incontinence immediately after an operation for prostate cancer. The most likely cause of his condition is damage to the prostatic nerve plexus that resulted in denervation of the internal urethrai sphincter. The function of which of the following types of tissue is most likely impaired as a result of this damage? . A ) Dense irregular connective tissue 8) Dense regular connective tissue . C) Skeletal muscle D) Smooth muscle . E } Transitional epithelium * m O m Previous Next Lab Values Calculator Review Help Pause
  • 92. Exam Section 2: Item 41 ol 50 National Board of Medical Examiners^ Comprehensive Basic Science Self-Assessment Time Remaining: 1 hr 13 min 0 sec ® Mark 41. A 10-year-old boy who has begun chemotherapy for acute myelogenous leukemia awakens at night with fever and severe pain in the ankles. Treatment with over-the-counter analgesics does not resolve the pain. The next morning, he has pain with urination and blood in the urine. An increased serum concentration of which of the following compounds is the most likely cause of this patient's symptoms? A ) Cystine 8) Glycine . ; C) Magnesium V D) Urea . . E) Uric acid * m O m Previous Next Lab Values Calculator Review Help Pause
  • 93. Exam Section 2: Item 42 ol 50 National Board of Medical Examiners^ Comprehensive Basic Science Self-Assessment Time Remaining: 1 hr 12 min 53 sec 9 Mark 42 A 27-year-old woman is brought to the physician by her family because she has been progressively lethargic and unwilling to leave her apartment over the past week. She has been receiving treatment in a mental health center for 10 years but missed her last appointment 13 days ago because of a snowstorm. She is now reluctant to return because she believes the staff is involved in an extraterrestrial plot. Three years ago; she had similar symptoms treated with electroconvulsive therapy. She appears disheveled. She is having auditory hallucinations of several people talking about her. Physical examination shows normal findings. She has poor eye contact, a flat affect and slow speech. She describes an elaborate delusional system about the plot at the mental health center. Thought content is otherwise impoverished. Which of the following is the most likely diagnosis? A ) Bipolar disorder depressed . B) Borderline personality disorder 0 C) Delusional disorder ) D ) Schizophrenia E) Schizotypal personality disorder * m O m Previous Next Lab Values Calculator Review Help Pause
  • 94. Exam Section 2: Item 43 of 50 National Board of Medical Examiners^ Comprehensive Basic Science Self-Assessment Time Remaining: 1 hr 12 min 56 sec ® Mark 43. A viral gene product is found to decrease expression of class i MHC molecules on the surfaces of infected cells. A mutant strain of virus is isolated that has a nonfunctional form of this gene. Which of the following types of cells are likely to contribute more effectively to control of the parental strain of the virus than to control of the mutant virus? 0 A ) CD4+- 7 lymphocytes 8) CD8+- T lymphocytes . ) C) Follicular dendritic cells ' _ D) Natural killer cells . . E } Plasma cells * m O m Previous Next Lab Values Calculator Review Help Pause
  • 95. Exam Section 2: Item 44 ol 50 National Board of Medical Examiners^ Comprehensive Basic Science Self-Assessment Time Remaining: 1 hr 12 min 54 sec ® Mark 44. A previously healthy 40-year-old man is brought to the emergency department 1 hour after the sudden onset of severe pain in his left leg while playing tennis. He is found to have ruptured the left Achilles tendon and undergoes operative repair and long leg cast immobilization. Six months later, the left calf shows a 2-cm decrease in circumference compared with the right calf. Which of the following is the most likely cause of this decrease? . A ) Decreased glycogen synthesis 8) Decreased myosin light chain phosphatase activity . J C) Increased phosphatidyl degradation ' j D) increased protein degradation . E) Mitochondria damage F) Necrosis of muscle fibers * m O m Previous Next Lab Values Calculator Review Help Pause
  • 96. Exam Section 2: Item 45 ol 50 National Board of Medical Examiners^ Comprehensive Basic Science Self-Assessment Time Remaining: 1 hr 12 min 50 sec ® Mark 45. A 5G-year-old woman has recently diagnosed carcinoma of the breast. An x-ray of the chest shows a tumor next to the right side of the heart. An enhanced CT scan with the tumor invading the pericardium is shown. Which of the following structures is most likely involved? A) Coronary sinus B) Greater splanchnic vein C) Right phrenic nerve D ) Right vagus nerve E) Thoracic duct O * 9 0 0 Previous Next Lab Values Calculator Review Help Pause
  • 97. Exam Section 2: Item 46 ol 50 National Board of Medical Examiners^ Comprehensive Basic Science Self-Assessment Time Remaining: 1 hr 12 min 4£ sec ® Mark 46. A 25-year-old woman with a history of rheumatic fever and mitral valve dysfunction comes to the physician because of a 2-week history of fever and fatigue. She underwent a root canal procedure 1 month ago; before which she had taken a single dose of amoxicillin. Her temperature is 38.4*0 (101 29F). A grade 4/6 blowing murmur is heard on auscultation under the left axilla. A photomicrograph of a Gram stain of the organism recovered from a blood culture specimen is shown. On blood agar plates; the organism shows alpha hemolysis. Which of the following is the most likely causal organism? A ) Enterococcus faecalis B) Group A beta-hemolytic streptococci 0 C) Staphylococcus aureus D) Streptococcus mitts E) Streptococcus pneumoniae O * m O m Previous Next Lab Values Calculator Review Help Pause
  • 98. Exam Section 2: Item 47 ol 50 National Board of Medical Examiners^ Comprehensive Basic Science Self-Assessment Time Remaining: 1 hr 12 min 46 sec ® Mark 47. A 24-year-old woman comes to the physician for advice about contraceptive methods. She is recently married and is not interested in having children until her mid 30s. Which of the following contraceptives carries the highest risk for interference with fertility in this patient Eater in life? . A ) Cervical cap with spermicidal jelly 8) Condoms and spermicidal foam o C) JUD o D) Oral contraceptive . . E) Progestin implant O m O m Previous Next Lab Values Calculator Review Help Pause
  • 99. Exam Section 2: Item 43 ol 50 National Board of Medical Examiners^ Comprehensive Basic Science Self-Assessment Time Remaining: 1 hr 12 min 44 sec ® Mark 48. Topical corticosteroid creams and phototherapy are often effective in the treatment of psoriasis. This effectiveness suggests a role for metabolism or nuclear binding of which of the following vitamins in the treatment of psoriatic lesions? A) Niacin ^ S) Vitamin B1 (thiamine) 0 G) Vitamin B2 (riboflavin) J 0 } Vitamin B6 (pyridoxine) 0 E) Vitamin C . ' F) Vitamin D 0 G) Vitamin E H } Vitamin K * m O m Previous Next Lab Values Calculator Review Help Pause
  • 100. Exam Section 2: Item 49 of 50 National Board of Medical Examiners^ Comprehensive Basic Science Self-Assessment Time Remaining: 1 hr 12 min 42 sec ® Mark 49. A 21-year-old woman of Japanese descent comes to the emergency department because of a 3-hour history of facial flushing. Her symptoms began shortly after she drank a glass of champagne for the first time at a wedding reception. Physical examination shows profound erythema over the face. The most likely cause of these findings is a mutation in the gene for which of the following enzymes? . A ) Acetyl-CoAreductase 8) Alcohol catalase . ) C) Alcohol dehydrogenase ' _ D) Alcohol reductase . E) Aldehyde dehydrogenase O * 9 0 0 Previous Next Lab Values Calculator Review Help Pause
  • 101. Exam Section 2: Item 50 ol 50 National Board of Medical Examiners^ Comprehensive Basic Science Self-Assessment Time Remaining: 1 hr 12 min 39 sec ® Mark 50. A 35-year-old woman is brought to the emergency department after she sustains a fracture of the neck of the fibula of her right leg. She was struck by a car while crossing the street. Which of the following findings is most likely on examination of the affected leg? Pain Over Proximal Fibula Pain Over Distal Fibula absent present absent Dorsiflexion Plantar Flexion Achilles Reflex O A) O B) O C) O D) O E) Absent 4/5 1/5 1+ Absent 0/S 4/5 1+ Present 1/5 4/5 2+ Present absent present 5/5 0/5 absent Present 4/5 4/5 2+ * m O m Previous Next Lab Values Calculator Review Help Pause
  • 102. Exam Section 3: Item 1 of 50 National Board of Medical Examiners^ Comprehensive Basic Science Self-Assessment Time Remaining: 1 hr 14 min 57 sec 9 Mark 1. AG5-year-old man comes to the physician because of fever and a worsening cough productive of approximatefy % cup of blood-tinged sputum daily. He has smoked VA packs of cigarettes daily for 45 years. His temperature is 38.4X (101 2aF); respirations are 20/min,and blood pressure is 140/90 mm Hg. Physical examination shows bilateral clubbing of the digits. Pulmonary examination shows egophony. whispered pedoriloquy, and dullness to percussion in the area overlying the 2nd and 3rd ribs on the right anteriorly just inferior to the right clavicle. Which of the following structures is the most likely site of an obstructing carcinoma? A ) Main carina 0 B) Right lower lobe bronchus 0 G) Right main bronchus 0 0 ) Right middle lobe bronchus J E } Right upper lobe bronchus m O m Next Lab Values Calculator Review Help Pause
  • 103. Exam Section 3: Item 2 of 50 National Board of Medical Examiners^ Comprehensive Basic Science Self-Assessment Time Remaining: 1 hr 14 min 54 sec 9 Mark 2. A 43-year-old woman comes to the physician because of a 4-week history of intermittent episodes of severe abdominal pain: especially after eating fatty foods. Physical examination shows no abnormalities. Abdominal ultrasonography shows several 3- to 4-mm gallstones contained within the gallbladder. The patient declines surgery. A drug with which of the following mechanisms of action is most appropriate for this patient? . A ) Decreased gallbladder muscle contractility 8) increased gastrin production . J C) increased metabolism of bite salts o D) inhibition of biliary cholesterol secretion . E } Inhibition of cytochrome P450 activity O m O m Previous Next Lab Values Calculator Review Help Pause
  • 104. Exam Section 3: Item 3 of 50 National Board of Medical Examiners^ Comprehensive Basic Science Self-Assessment Time Remaining: 1 hr 14 min 52 sec ® Mark 3. A 24-year-old woman comes to the physician for an examination prior to employment. Physical examination shows no abnormalities. An ECG shows a sinus rhythm of 70/min; PR interval of 152 msec and QRS complex of 84 msec: there are no ectopic beats. Which of the following best represents the longest conduction time in this patient? A ) Anterior left bundle branch 8) intra-atrial G) Low right atrium to bundle of His D) Proximal bundle of His to ventricular myocardium E) Right bundle branch O * m O m Previous Next Lab Values Calculator Review Help Pause
  • 105. Exam Section 3: Item 4 of 50 National Board of Medical Examiners® Comprehensive Basic Science Self-Assessment Time Remaining: 1 hr 14 min 46 sec ® Mark 4. An 18-year-old woman with mild intellectual disability is brought to the physician because of a 3-day history of decreased ability to see in reduced light. She has a lifelong history of chronic diarrhea. Two years ago; she developed a lack of muscle control of her arms and legs: and generalized weakness. Her 16-year-old brother has had similar symptoms. Ophthalmologic examination shows bilateral retinitis pigmentosa. There is ataxia and loss of deep tendon reflexes. Laboratory studies show erythrocytes with spiny projections and a serum total cholesterol concentration of 40 mg/dL. Which of the following apolipoproteirs is most likely deficient in this patient? O A ) ApoA-l O B) Apo A-l o C} Apo B O D } Apo C O E) Apo E o o & r # Previous Next Lab Values Calculator Review Help Pause
  • 106. Exam Section 3: Item 5 of 50 National Board of Medical Examiners^ Comprehensive Basic Science Self-Assessment Time Remaining: 1 hr 14 min 41 sec ® Mark 5. An investigator is studying obesity in an adolescent population. Five thousand normal-weight patients are followed from the age of 10 years to the age of 15 years. At the conclusion of the study, 1100 patients meet the criteria for obesity. Which of the following best represents the incidence per 1000 patient-years of obesity during the course of this study? O A) 10 O 8) 25 O C) 44 o D) 50 O E) 64 O * m O m Previous Next Lab Values Calculator Review Help Pause
  • 107. Exam Section 3: Item 6 of 50 National Board of Medical Examiners^ Comprehensive Basic Science Self-Assessment Time Remaining: 1 hr 14 min 33 sec 9 Mark t " *y?vrG>H v * .>v- - . -. -v ^ asf i i u A d a£F £ 6. A 2S-year-old woman at 32 weeks' gestation comes to the physician because of a 4-day history of fever and back pain. She says:hat during this time she afso has been crying frequentfy. Her temperature is 38X (100.4°F). Physical examination shows costophrenic angle tenderness. The photomicrograph shown represents her disease. Which of the following mechanisms iis the most likely cause? Q A ) Chorioamnionitis 3 B) Endometritis Q G) Glomerulonephritis O D) Hematogenous infection Q E } Obstructive uropathy F) Pelvic infEammatory disease * _ c* 4 A * i i Hi ^ * _ S 'L r i ~ - Sr I ^ | I - i L .* * J T' a i F V I * »i : *1 * * - •j '*' t* i ** • ' f *> * % * * ¥ V - - k* li * !r ;&3r V 41 . “ P u n -i n ill r *- j rrn* • • 'S- . L * .. . 4 ii 1 , I ' > t ' -r * . i * r / P L U I -t *»* . 0% ^ - l/v » I * . : ! M * A- * h ,vF rf -I 1 *1 * + l* § wf I -- 1 K -; JI 'V -.’ -F „ r'rt H W * s , % » ** V l! twit |V ,1* n J w : 4 m f =. «1 ! © © Previous Next Lab Values Calculator Review Help Pause
  • 108. Exam Section 3: Item 7 of 50 National Board of Medical Examiners^ Comprehensive Basic Science Self-Assessment Time Remaining: 1 hr 14 min 35 sec ® Mark 7. A 54-year-old man with a 1-year history of multiple sclerosis has increasingly painful muscle spasms. An agonist that acts at which of the following receptors is most likely to alleviate the increased extensor tone and clonus? 3 A ) y-Aminobutyricacid ^ S) Histamine-1 (Ht) o G) Histamine-2 (H ^ i D) Muscarinic-1 (M,) 3 E) Muscarinic-2 3 F) Nicotinic E o m O m Previous Next Lab Values Calculator Review Help Pause
  • 109. Exam Section 3: Item 8 of 50 National Board of Medical Examiners^ Comprehensive Basic Science Self-Assessment Time Remaining: 1 hr 14 min 32 sec ® Mark 8. A 27-year-old primigravid woman at 36 weeks' gestation comes to the physician for a prenatal visit. Physical examination shows a uterus consistent in size with a 33-week gestation, so the patient is referred for ultrasonography, which shows normal fetal measurements. The ultrasonography showrn is an image of the male fetal scrotum; the testes are indicated by the arrows. Which of the following is the most likely underlying cause of this finding? A } High attachment of the tunica albuginea . 8) Lack of scrotal attachment of the guberrtaculum ' G) Obstruction of the posterior urethra J 0 ) Patent processus vaginalis E } Twisting of the spermatic cord * m O m Previous Next Lab Values Calculator Review Help Pause
  • 110. Exam Section 3: Item 9 of 50 National Board of Medical Examiners^ Comprehensive Basic Science Self-Assessment Time Remaining: 1 hr 14 min 28 sec ® Mark 9. A 98-year-old woman dies of respiratory failure. At autopsy: gross examination shows a small heart with a brown discoloration. Histologic examination of the cardiac tissue by light microscopy shows fight brown pigment within the cardiac myocytes, particularly in the perinuclear region. This material does not stain positively for iron. Which of the following best describes this pigment? . A ) Anthracotic pigment 8) Free fatty acids . C) Hemosiderin D) Laminin . . E) Lipofuscin O * m O m Previous Next Lab Values Calculator Review Help Pause
  • 111. Exam Section 3: Item 10 ol 50 National Board of Medical Examiners^ Comprehensive Basic Science Self-Assessment Time Remaining: 1 hr 14 min 24 sec ® Mark 10. A 43-year-old woman has had increasing fatigue, intolerance to cold: and dry thickened skin for 7 weeks. The thyroid gland is firm, nodular and diffusely enlarged. Examination of tissue obtained on biopsy of the thyroid gland shows a diffuse infiltration of lymphocytes with occasional lymphoid follicles. Which of the following is the most likely diagnosis? . A ) Chronic autoimmune (Hashimoto) thyroiditis 8) Diffuse toxic goiter (Graves disease) 0 C) Granulomatous thyroiditis 0 D) Papillary carcinoma . E) Toxic multinodular goiter F) Viral thyroiditis * m O m Previous Next Lab Values Calculator Review Help Pause
  • 112. Exam Section 3: Item 11 of 50 National Board of Medical Examiners^ Comprehensive Basic Science Self-Assessment Time Remaining: 1 hr 14 min 20 sec ® Mark 11. A 3-year-old girl has a history of recurrent infections, fn vitro, neutrophils isolated from this patient are capable of phagocytosis and can kill Lactobacillus species but not Staphylococcus aureus. This patient most likely has a defect involving which of the following enzymes? . A ) Catalase 8) Elastase . C) Myeloperoxidase o D) NADPH oxidase . E } Superoxide dismutase O m O m Previous Next Lab Values Calculator Review Help Pause
  • 113. Exam Section 3: Item 12 ol 50 National Board of Medical Examiners^ Comprehensive Basic Science Self-Assessment Time Remaining: 1 hr 14 min 17 sec 9 Mark 12 A 7-year-old girl is brought to the physician because of a 2-week history of painful swelling under her right ami which has become increasingly severe during the past 3 days. Physical examination shows a 2 x 1-cm, tender right axillary lymph node and a small papule on the dorsum of the right hand. A silver-stained biopsy specimen of the papule shows pleomorphic bacilli. Which of the following is the most likely causal organism? O A } Bartonella henselae 8) Brucella melitensis o C) Burkholder!a mallei 0 D) Franciselta tuiarensis . E) Streptobacitlus moniliformis * m O m Previous Next Lab Values Calculator Review Help Pause
  • 114. Exam Section 3: Item 13 of 50 National Board of Medical Examiners^ Comprehensive Basic Science Self-Assessment Time Remaining: 1 hr 14 min 14 sec 9 Mark 13. A newborn born at 3u weeks' gestation dies 3 days later The photograph shown is of a section of brain as seen at autopsy. Which of the following is the most likely underlying disease? A ) Biliary atresia B) Cyanotic congenital heart disease O ®) Hemolytic disease of the newborn D ) Neonatal meningitis O E) Respiratory distress syndrome o o o Previous Next Lab Values Calculator Review Help Pause
  • 115. Exam Section 3: Item 14 of 50 National Board of Medical Examiners^ Comprehensive Basic Science Self-Assessment Time Remaining: 1 hr 14 min 10 sec 9 Mark 14. A 50-year-ofd woman is brought to the emergency department 20 minutes after the sudden onset of pain in her epigastrium that radiates to her back and is associated with nausea. She has a history of cholelithiasis. She appears restless. Her temperature is 37.6X (99.7aF): pulse is 120/min, and btood pressure is 115/60 mm Hg. Abdominal examination shows distention with rebound tenderness and guarding. The pain is partially relieved when the patient bends forward. ACT scan of the abdomen shows fluid surrounding the pancreas. Obstruction of which of the following is the most likely cause of the findings in this patient? A ) Ampulla of Vater 0 B) Cisterna chyli 0 G) Common hepatic duct 0 0 ) Cystic duct J E } Superior mesenteric artery * m O m Previous Next Lab Values Calculator Review Help Pause
  • 116. Exam Section 3: Item 15 ol 50 National Board of Medical Examiners^ Comprehensive Basic Science Self-Assessment Time Remaining: 1 hr 14 min 7 sec ® Mark 15. A 74-year-ofd man comes to the office because of a 3-week history of progressive shortness of breath with exertion. He has had difficulty climbing one flight of stairs. He also has a 20-year history of poorly controlled hypertension, for which he rarely takes his prescribed medication and osteoarthritis treated with naproxen daily. His temperature is 37.1CC (98.8QF); puise is 76/min, respira:ions are 20/min, and blood pressure is 180/98 mm Hg. Pulse oximetry on room air shows an oxygen saturation of 92%. Physical examination shows 10-cm jugular venous distention above the sternal angle. Crackies are heard over both lung bases. An S4 is heard. Echocardiography shows a normal ejection fraction. Which of the following best explains this patient's symptom? ) A ) Decreased myocardiaE oxygen supply ' J B) External compression of the right ventricle 0 C) Impaired left ventricular relaxation ' J D ) Increased turbulent flow across the mitral valve J E } Obstruction of the left ventricular outflow tract O 0 I* 0 0 0 Previous Next Lab Values Calculator Review Help Pause
  • 117. Exam Section 3: Item 16 ol 50 National Board of Medical Examiners^ Comprehensive Basic Science Self-Assessment Time Remaining: 1 hr 14 min 4 sec ® Mark 25i 16. A study is conducted to assess the effect of a new selective estrogen receptor modulator (SERM) on bone fractures. Two thousand women with a history of vertebral fracture and 5000 women with no fracture history are randomly assigned to a placebo or to SERM at a dose of 60 mg or 120 mg. The graph shows the percent of women who had a new fracture during 5 years of foJIow-up (± standard error of the mean). Which of the following is the most accurate conclusion about preventing new fractures based on these data? A ) High-dose SERM treatment is more effective than low-dose SERM treatment B) Low-dose SERM treatment is more effective than high-dose SERM treatment 3 C) Placebo treatment is effective only in women with a history of vertebral fracture D) SERM treatment is effective only in women with a history of vertebral fracture E) SERM treatment is effective only in women with no history of fracture T £ 20 o 15 T T 0) 10 £ T T T 5 - o o 0 No fracture history Fracture history Placebo SERM 60 mg SERM 120 mg O * 9 0 0 Previous Next Lab Values Calculator Review Help Pause
  • 118. Exam Section 3: Item 17 of 50 National Board of Medical Examiners^ Comprehensive Basic Science Self-Assessment Time Remaining: 1 hr 14 min 0 sec ® Mark 17. A 23-year-ofd man cuts his tip inadvertently while shaving. Seconds after the injury the bleeding nearly stops. Which of the following mechanisms is the most likely cause of the early rapid control of blood loss in this man? 3 A ) Activation of antithrombin Hi ^ 8 } Localized secretion of endothelin Q G) Oxygen-stimulated cleavage of thromboplastin D) Polymerization of fibrin «r * m O m Previous Next Lab Values Calculator Review Help Pause
  • 119. Exam Section 3: Item 13 ol 50 National Board of Medical Examiners^ Comprehensive Basic Science Self-Assessment Time Remaining: 1 hr 13 min 57 sec 9 Mark 18. A 21-year-old woman comes to the physician for a routine health maintenance examination. Her 3-year-old sister was recently diagnosed with cystic fibrosis. Prior to any genetic testing; which of the following best approximates this patient's risk of being a heterozygote for cystic fibrosis? O A) 0 O 8) 1/2 O C) 1/4 o D) 2/3 O E} 3/4 O * 9 0 0 Previous Next Lab Values Calculator Review Help Pause
  • 120. Exam Section 3: Item 19 of 50 National Board of Medical Examiners^ Comprehensive Basic Science Self-Assessment Time Remaining: 1 hr 13 min 54 sec 9 Mark 19. A 25-year-old woman with a 3-year history of celiac disease comes to the physician because of a markedly itchy rash on her scalp, arms, buttocks: and legs for 5 weeks. Examination of the skin shows many crusted papules surrounded by excoriations over the upper and lower extremities and a few vesicles on the buttocks. A skin biopsy specimen shows the vesicles to be subepidermal. Direct immunofluorescence of the vesicles shows granular deposits of IgA in the epidermal-dermal junction. Which of the following is the most likely diagnosis? A ) Bullous pemphigoid 3 B) Dermatitis herpetiformis o G) Dyshidrotic eczema 3 D ) Epidermolysis bullosa acquisita J E) Pemphigus * m O m Previous Next Lab Values Calculator Review Help Pause
  • 121. Exam Section 3: Item 20 ol 50 National Board of Medical Examiners^ Comprehensive Basic Science Self-Assessment Time Remaining: 1 hr 13 min 50 sec 9 Mark 20. A 1-year-old boy is brought to the physician for a weli-chiid examination. The mother is concerned because her son's fine, pale hair has not changed color since birth. His eyes are blue. During ophthalmologic examination, the patient turns away from the flashlight and starts crying. Which of the following is the most likely cause of the paie skin color in this patient? . A ) Aberrant migration of neural crest cells 8) Decreased number of epidermal melanocytes . J C) immune destruction of melanocytes ' 3 D) inability to produce melanin . E } Melanin dropout to the dermis O * m O m Previous Next Lab Values Calculator Review Help Pause